You are on page 1of 271

Page|1

NAME: Date:
Professor: Section: Score:
ACCOUNTING FOR SPECIAL TRANSACTIONS
FIRST GRADING EXAMINATION

1. AAA and BBB are partners with capital of P60,000 and P20,000, respectively. Profits and losses
are divided in the ratio of 60:40. AAA and BBB decided to form a new partnership with CCC,
who invested land valued at P15,000 for a 20% capital interest in the new partnership. CCC’s cost
of the land was P12,000 the partnership elected to use the bonus method to record the admission
of CCC into the relationship. CCC’s capital account should be credited for
a. P12,000 c. P16,000
b. P15,000 d. P19,000
60+20+15= 95 x 20%=19

2. AAA and BBB formed partnership in 2009. The partnership agreement provides for annual salary
allowances of P55,000 for AAA and P45,000 for BBB. The partners share profits equally and losses
in a 60:40 ratio. The partnership had earnings of P80,000 for 2009 before any allowance to partners.
What amount of these earnings should be credited to each partner’s capital account?
AAA BBB AAA BBB
a. P40,000 P40,000 c. P 44,000 P 36,000
b. 43,000 37,000 d. 45,000 35,000
B

3. The partnership agreement of AAA and BBB provides that interest at 10% per year is to be credited
to each partner on the basis of weighted-average capital balances. A summary of BBB’s capital
account for the year ended December 31, 2009, is as follows:
Balance, January 1 P 140,000
Additional investment, July 1 40,000
Withdrawal, August 1 15,000

What amount of interest should be credited to BBB’s capital account for 2009?
a. P15,250 c. P16,500
b. P15,375 d. P17,250

4. AAA and BBB are partners who share profits and losses on the ratio of 6:4, respectively. On May
1, 2009, their respective capital accounts were as follows:
AAA P 60,000
BBB 50,000
Page|2

On the date, CCC was admitted as a partner with one-third interest in capital and profits for an
investment of P40,000. The new partnership began with total capital of P150,000. Immediately
after CCC’s admission, AAA’s capital should be
a. P50,000 c. P56,667
b. P54,000 d. P60,000

5. AA and BB formed a partnership in 20x1 and made the following investments and capital
withdrawals during the year:

AA BB
Investments Draws Investments Draws March
1………………P30, 000 P 20, 000
June 1………………………… P10, 000 P10,000
August 1………………20, 000 2,000
December 1……………………. 5, 000

The partnership’s profit and loss agreement provides for salary of which P30,000 was paid to each
partner for 20x1. AA is to receive a bonus of 10% on net income after salaries and bonus. The
partners are also to receive interest of 8% on average annual capital balances affected by both
investments and drawings. Any remaining profits are to be allocated equally among the partners.

Assuming the net income of P60, 000 before salaries and bonus, determine how the income would be
allocated among the partners.

a. AA, P31, 138; BB, P28, 862 c. AA, P30, 633; BB, P29, 376
b. AA, P33, 537; BB, P26, 463 d. AA, P30, 684; BB, P29, 316
D

A B
2,000 1,333
(467) (467)
667 (67)
(33) 60,000
Interest 2,167 800
(2,967)
Salaries 30,000 30,000
(60,000)
(1,483) (1,483)
(2,967)
30,683 29,317

Use the following information to answer the next two questions


Page|3

The following condensed balance sheet is presented for the partnership of AAA and BBB, who share
profits and losses in the ratio of 60:40, respectively:
Cash P 45,000 Accounts payable P 120,000
Other assets 625,000 AAA, capital 348,000
BBB, loan 30,000 BBB, capital
232,000
Total P 700,000 Total P 700,000
The assets and liabilities are fairly valued on the balance sheet. AAA and BBB decide to admit CCC
as a new partner with 20% interest.

6. What amount should CCC contribute in cash or other assets?


a. P110,000 c. P140,000
b. P116,000 d. P145,000

D 348,000 + 232,000 = 580 ÷ 80% x 20% = 145,000

7. Instead of admitting a new partner, AAA and BBB decide to liquidate the partnership. If other
assets are sold for P500,000, what amount of the available cash should be distributed to AAA?
a. P255,000 c. P327,000
b. P273,000 d. P348,000
B
A B 45000
348,000 202,000 550,000 500,000
(75,000) (50,000) (125,000) (120,000)
273,000 152,000 425,000 425,000 Cash

8. The following condensed balance sheet is presented for the partnership of BBB and AAA, who
share profits and losses on the ratio of 60:40, respectively:
Other assets P 450,000
BBB loan
20,000
P 470,000
Accounts payable P 120,000
BBB, capital 195,000
AAA, capital
155,000
Total P 470,000
The partners have decided to liquidate the partnership. If the other assets are sold P385,000, what
amount of the available cash should be distributed to BBB?
a. P136,000 c. P159,000
Page|4

b. P156,000 d. P195,000

A
A B 385,000
175,000
155,000 175,000

(26,000) (39,000) (65,000) (120,000)


136,000 265,000 265,000 Cash
129,000

9. On December 31, 1998, the partners of MNP Partnership decided to liquidate their business.
Immediately before liquidation, the following condensed balance sheet was prepared:

Cash P 50,000 Liabilities P 375,000


Noncash assets 900,000 Nieva, loan 80,000
Perez, loan 25,000
Munoz, capital (50%) 312,500
Nieva, capital (30%) 107,500
____ Perez, capital (20%) 50,000
Total P 950,000 Total P 950,000
The noncash assets were sold for P400,000. Assuming Perez is the only solvent partner, what
amount of additional cash will be invested by Perez? (rounded to the nearest peso) a. P
37,143
b. 25,000
c. 5,250
d. 0

B
Munoz Nieva Perez
50% 30% 20 %
Loan 80,000 25,000
Capital 312,500 107,500 50,000
Totals 312,500 187,500 75,000
Loss on realization (950 - 400) (250,000) (150,000)
(100,000)
62,500 37,500 (25,000)

10. The partners of the M & N Partnership started liquidating their business on July 1, 2004, at which
time the partners were sharing profits and losses 40% to M and 60% to N. The balance sheet of
the partnership appeared as follows:
Page|5

M & N Partnership
Balance Sheet – July 1, 2004

Assets Liabilities & Capital


Cash……………………. P 8,800 Accounts payable………… P32, 400
Receivable……………… 22,400 M, capital………………… P31, 000
Inventory…………...….. 39,400 M, drawing………… 5,400 25, 600
Equipment…..P65, 200 N, capital………………… .P33, 200
Accumulated N, drawing……………………. 200 33, 000 depreciation 30, 80034, 400 N,
loan…………………………………… 14 , 000
Total…………………… P105, 000 Total…………………………………… P105, 000

During the month of July, the partners collected P600 of the receivables with no loss. The partners also
sold during the month the entire inventory on which they realized a total of P32,400.

How much of the cash was paid to M’s capital on July 31, 2004?
a. P -0- c. P5, 400
b. 25, 600 d. 320
D
M N Totals
25,600 47,000 72,600
(7,000) Loss on in ventory
(2,800) (4,200)
Possible l oss on rece ivables
(8,720) (13,080) (21,800)
Possible l oss on PP E
(13,760) (20,640) (34,400)
320 9,080

11. After all noncash assets have been converted into cash in the liquidation of the AA and JJ
partnership, the ledger contains the following account balances:

Debit Credit

Cash……………………………………………… P 34, 000


Accounts payable………………………………………………………… P25, 000
Loan payable to AA……………………………………………………….. 9,000
AA, capital…………………………………………… 8,000
JJ, capital……………………………………………………………… 8,000

Available cash should be distributed; P25,000 to accounts payable and;


Page|6

a. P9, 000 loan payable to AA c. P1,000 to AA and P8, 000 to JJ


b. P4, 500 each to AA and JJ d. P8,000 to AA and P1, 000 to JJ
C
A J
1,000 8,000
1,000 8,000

12. After incurring losses resulting from very unprofitable operation, the Alphabets Partnership
decided to liquidate when the partners’ capital balances were:

A, capital (40%) P 80,000


B, capital (40%) 130,000
C, capital (20%) 96,000
The non-cash assets were sold in installment. Available cash were distributed to partners in every
sale of non-cash assets. After the second sale of non-cash assets, the partners received the same
amount of cash in the distribution. And from the third sale of non-cash assets, cash available for
distribution amounts to P 28,000, and non-cash assets has a book value of P 12,500. Using cash
priority program, what amount did C received in the third installment of cash? a. P 11,600
b. 8,000
c. 5,600
d. 0
C
28 ,000 x 20% = 5,600
A B C Payments
40% 40% 20% A B C
80,000 130,000 96,000 306,000
200,000 325,000 480,000
(155,000) (31,000)
200,000 325,000 325,000
(125,000) (125,000) (50,000) (25,000)

200,000 200,000 200,000 (80,000) (80,000) (40,000)


(306,000) (80,000) (130,000) (96,000)

13. The partnership of AA, BB, and CC was dissolved on June 30, 20x1 and account balances after
non-cash assets were converted into cash on September 1, 2004 are:

Cash……………………………P50, 000 Accounts payable P120, 000


AA, capital (30%) 90, 000
Page|7

BB, capital (30%) (60, 000)


CC, capital (40%) (100, 000)

Personal assets and liabilities of the partners at September 1, 20x1 are:

Personal Personal
Assets Liabilities
AA………………………………………………………….. P80, 000 P90, 000
BB………………………………………………………… 100, 000 61, 000
CC………………………………………………………… 192, 000 80, 000
If CC contributes P70, 000 to the partnership to provide cash to pay the creditors, what amount of
AA’s P90, 000 partnership equity would appear to be recoverable?

a. P90,000 c. P79, 000


b. 81,000 d. None
B
A B C
30% 40% 40 %
90,000 (60,000) (100,000)
70,000 (70,000)
90,000 (60,000) (30,000)

39,000
90,000 (21,000) (30,000)
(9,000) (12,000)

81,000 (42,000)

14. Partners Able, Baker, and Chapman, who share profit and loss equally, have the following
personal assets, personal liabilities, and partnership capital balances:

Able Baker__ Chapman_


Personal assets P 30,000 P 80,000 P 60,000
Personal Liabilities 25,000 50,000 72,000
Capital balances 50,000 (32,000) 70,000

After applying the doctrine of marshaling of assets, the capital balances of Able, Baker, and Chapman,
respectively, would be
a. P 50,000 P(2,000) P 58,000
b. 48,000 0 58,000
Page|8

c. 49,000 0 57,000
d. 34,000 0 54,000
C
A B C
Interest 50,000 (32,000) 70,000 88,000
Personal 30,000
(12,000)
50,000 (2,000) 58,000 108,000
(925.93) 2,000 (1,074.07) (2,000)

49,074 - 56,926

15. A, B and C are partners in a textile distribution business, sharing profits and losses equally. On
December 31, 2004, the partnership capital and the partners’ drawing were as follows:

A B C Total
Capital P100,000 P80,000P300,000 P480,000
Drawing 60,000 40,000 20,000 120,000

The partnership was unable to collect on its trade receivables, and it was forced to liquidate. The
operating profits for 2005 amounted to P72,000, and was all exhausted including the partnership
assets. Unsettled creditors’ claim at December 31, 2005 amounted to P84,000. B and C have
substantial private resources, but A has no available free assets.

The final cash distribution to C was: a.


P 162,000
b. P 108,000
c. P 84,000
d. P 78,000

D
A B C A L C (adjusted)
40,000 40,000 280,000 0 84,000 432,000
24,000 24,000 24,000 72,000 Net loss 516,000
64,000 64,000 304,000
(172,000) (172,000) (172,000) (516,000)
(108,000) (108,000) 132,000
108,000 (54,000) (54,000) (108,000)
- (162,000) 78,000
Page|9

16. A, B and C are partners with capital balance of P 350,000, P 250,000 and P 350,000 and sharing
profits 30%, 20% and 50% respectively. Partners agree to dissolve the business and upon
liquidation, all of the partnership assets are sold and sufficient cash is realized to pay all the claims
except for P50,000. C is personally insolvent, but the other two partners are able to meet any
indebtedness to the firm. On the remaining claim against the partnership, A is to absorb. a. P
40,000

b. P 15,000
c. P 30,000
d. P 25,000

A
A B C A L C
30% 20% 50% - 50,000 950,000
350,000 250,000 350,000 950,000 Net loss 1,000,000
(300,000) (200,000) (500,000) (1,000,000)
50,000 50,000 (150,000)
(90,000) (60,000) 150,000

(40,000) (10,000) -

17. A, B, and C are partners in ABC Partnership and share profits and losses, 5:3:2, respectively. The
partners have agreed to liquidate the partnership. Prior to liquidation, the partnership balance
sheet shows the following book values.

Cash P 25,200
Non-cash 297,600
Notes, payable to C 38,400
Other liabilities 184,800
A, capital 72,000
B, capital (12,000)
C, capital 39,600

Liquidation expenses of P 16,800 are paid. Non-cash assets with a book value of P 240,000 are sold for
P 216,000.

How much cash should C receive? a.


P 74,571
P a g e | 10

b. P 46,458
c. P 39,600
d. P 37,600

C
A (50%) B (30%) C (20%) A L C
72,000 (12,000) 39,600 -
Note 38,400
72,000 (12,000) 78,000 138,000
(8,400) (5,040) (3,360) (16,800) Liquidatio n expenses
(12,000) (7,200) (4,800) (24,000) Loss on s ale
51,600 (24,240) 69,840

(28,800) (17,280) (11,520) (57,600) Possible loss on un sold asset s


22,800 (41,520) 58,320
(29,657) 41,520 (11,863)

(6,857) - 46,457
6,857 (6,857)
(0) - 39,600

18. Partners Bee, Cee, Dee and Gee who share profits 5:3:1:1, respectively, decide to liquidate their
partnership. Capital balances before liquidation are:

Bee P 60,000
Cee 40,000
Dee 30,000
Gee 10,000
The partners agree to the following:
(1) Partnership’s computer equipment with a book value of P12,000 is to be taken over by partner
Bee at a price of P15,000.
(2) Partnership’s liabilities are to be paid off and the balance of cash on hand, P30,000 is to be
divided in a manner that will avoid the need for any possible recovery of cash from a partner.

How much of the P30,000 cash be distributed to Partner Cee? a.


P 10,000
b. P 0
c. P 20,000
d. P 15,000
P a g e | 11

liquidation. The partners had been sharing profits and losses on a 60:40 ratio. The balance sheet
prepared on the day of liquidation began was as follows:

Assets Liabilities and Capital


Cash P 18,000 Accounts payable P 42,000
Receivables 75,000 A, loan 24,000
Inventory 90,000 A, capital 102,000
Other 84,000 B, capital 99,000
Total P267,000 Total P267,000
During June, one-third of the receivables was collected; P45,000 of inventory was sold at an average
of 70% of book value; other assets were sold for P36,000.

How much should A and B receive upon liquidation?


P a g e | 12

A B
a. P32,100 P36,400
b. P 8,100 P27,400
c. P40,200 P41,800
d. P59,100 P54,400
A
Carrying amount of non-cash (75 + 90 + 84) 249
Amount realized from sale (1/3 x 75) + (70% x 45) + 36 (92.5)
Loss on realization (156.5)

Cash 18 + 92.5 = 110.5 – 42 = 68.5 cash available to partners less capital balance 225K = (156.5)
A B
60% 40 %
126,000 99,000 225,000
(93,900) (62,600) (156,500)
32,100 36,400 68,500

20. A, B, and C, who divide profits and losses 50%, 30%, and 20%, respectively, have the following December
31, 20x1 account balances:

A, drawing (Dr.)………………………………………………………… P 12, 000


C, drawing (Cr.)………………………………………………………..… 4,800 Accounts
receivable- A……………………………………………….… 7, 200
Loans payable- B…………………………………………………………... 14, 400
A, capital…………………………………………………………………. 59, 400
B, capital……………………………………………………………………. 44, 400
C, capital………………………………………………………………..… 39, 000

On this data, the partnership’s assets are P211,200 (including cash of P64, 200).The partnership is liquidated
and C receives P33,000 in final settlement. How much is the total loss on realization?

a.P10,800 c. P54,000
b. 31,200 d. 64,200
C
C
20%
43,800
(10,800) (54,000)
33,000

21. A and B share partnership profits and losses in a 7:3 ratio. Their post-closing trial balance on January 31
show before liquidation:

Cash………………………………………P 30, 000


Accounts receivable, net………….……... 380, 000
Inventory……………………………… 260, 000
Furniture, net…………………………… 120, 000
Accounts payable…………………………………………………P165, 000
A, capital…………………………………………………………...350, 000
B, capital………………………………………………………. 275, 000
P a g e | 13

C offered to buy for P760,000 the partnership assets including liabilities but excluding cash and after certain
assets are to be restated at their fair values as follows:

Accounts receivable ……………………………………………….. P350,000


Inventory ……………………………………………………………… 250,000
Furniture ……………………………………………………………… 135,000

How much will A and B receive as final settlement of their partnership interest?

a. P 570, 000 c. P790, 000


b. 760, 000 d. 625, 000
C
A B
70% 30 %
350,000 275,000
115,500 49,500 165,000
465,500 324,500 790,000

22. AAA and BBB partnership’s balance sheet at December 31, 2009, reported the following:
Total Assets P 100,000 Total liabilities 20,000
AAA, capital 40,000
BBB, capital 40,000

On January 2, 2010, AAA and BBB dissolved their partnership and transferred all assets and liabilities
to a newly formed corporation. At the date of incorporation, the fair value of the net assets was P12,000
more than the carrying amount in the partnership’s books, which was assigned to tangible assets. AAA
and BBB were each issued 5,000 shares of the corporation’s P1 par value common stock. Immediately
following incorporation, additional paid-in capital in excess of par should be credited for
a. P68,000 c. P77,000
b. P70,000 d. P82,000

D 80K + 12K = 92K – 10K = 82K

23. When property other than cash is invested in a partnership, at what amount should the noncash
property be credited to the contributing partner’s capital account? a. Fair value at the date of
contribution.
b. Contributing partner’s original cost.
c. Assessed valuation for property tax purposes.
d. Contributing partner’s tax basis.
24. A and B formed a partnership. A contributed cash of ₱500,000 while B contributed land with carrying
amount of ₱400,000 and fair value of ₱800,000 . The land has an unpaid mortgage of ₱200,000 which is
assumed by the partnership. How much is the correct valuation of B’s capital immediately after the
partnership formation? a. 400,000
b. 500,000
c. 600,000
d. 800,000

Solution:
P a g e | 14

Partnershi
A B p
Cash 500,000 - 500,000
Land (at fair value) 800,000 800,000
Total 500,000 800,000 1,300,000

Mortgage payable - 200,000 200,000


A, capital 500,000 500,000
B, capital (800K – 600,000
200K) 600,000
Total 500,000 800,000 1,300,000
25. Mr. A and Ms. B formed a partnership and agreed to divide the initial capital equally even though Mr.
A contributed ₱100,000 and Ms. B contributed ₱84 ,000 in identifiable assets. The partners agree that the
difference in the amount of contribution and the amount of credit to the partner’s capital shall be treated
as compensation for the expertise that the partner will be bringing to the partnership. How much is the
correct valuation of A’s capital immediately after the partnership formation? a. 84,000
b. 92,000
c. 100,000
d. 108,000

Solution:
Cash 184,000
A, Capital (184,000 ÷ 2) 92,000
B, Capital (184,000 ÷ 2) 92,000

26. A and B formed a partnership. The following are their contributions:

A B
Cash 500,000 -
Accounts receivable 100,000 -
Building 700,000
Total 600,000 700,000

A, capital 600,000
B, capital 700,000
Total 600,000 700,000
Additional information:
• The accounts receivable includes a ₱20 ,000 account that is deemed uncollectible.
• The building is under-depreciated by ₱50,000.
• The building has an unpaid mortgage ₱100,000, but this is not assumed by the partnership. Partner B
promised to pay for the mortgage himself.

How much is the correct valuation of A’s capital immediately after the partnership formation? a.
460,000
b. 580,000
c. 650,000
d. 720,000
P a g e | 15

Solution:
Partnershi
A B p
Cash 500,000 - 500,000
Accounts receivable
(100K – 20K) 80,000 - 80,000
Building (700K – 650,000
50K) 650,000
Total 580,000 650,000 1,230,000

A, capital 580,000 580,000


B, capital 650,000 650,000
Total 580,000 650,000 1,230,000
27. Mr. A and Ms. B formed a partnership and agreed to divide the initial capital equally even though Mr.
A contributed ₱100,000 and Ms. B contributed ₱84 ,000 in identifiable assets. The partners agree that the
difference in the amount of contribution and the amount of credit to the partner’s capital shall be treated
as cash settlement between the partners. The compound entry to record the partners’ contributions
includes a credit to B’s capital account in the amount of a. 84,000
b. 92,000
c. 100,000
d. 108,000

Solution:
Cash 184,000
A, Capital (184,000 ÷ 2) 92,000
B, Capital (184,000 ÷ 2) 92,000

The cash settlement among the partners is not recorded in the partnership’s books because this is not a
transaction of the partnership but rather a transaction among the partners themselves.

28. If the partnership agreement does not specify how income is to be allocated, profits and loss should be
allocated a. Equally.
b. In proportion to the weighted average of capital invested during the period.
c. Equitably so that partners are compensated for the time and effort expended on behalf of the
partnership.
d. In accordance with their capital contributions.

29. A and B share in partnership profits and losses on a 40:60 ratio. During the year, A’s capital account has
a net increase of ₱50,000. Partner A made contributions of ₱10 ,000 and capital withdrawals of ₱60 ,000
during the year. How much was the share of B in the partnership profit for the year? a. 100,000
b. 150,000
c. 200,000
d. 180,000

Solution:

Step 1:

beg.
P a g e | 16

A, Capital
-

60,000 10,000
?
50,000
Withdrawal
s Additional investment
Share in profit
end.

Step 2:

A, Capital
beg. -
60,000 10,000 Withdrawals Additional investment
Share in profit 100,000 (squeeze)
50,000
end.

Step 3: 100,000 ÷ 40% = 250,000 partnership profit Step 4:


B’s share: 250,000 x 60% = 150,000

30. The partnership agreement of A, B and C stipulates the following:


• Partners A and C shall receive annual salaries of ₱12,000 and ₱8 ,000, respectively.
• A bonus of 10% of profit after salaries but before deduction of bonus shall be given to Partner A, the
managing partner.
• Each partner shall receive 10% interest on average capital investments.
• Any remaining profit or loss shall be shared as follows: 40% to A and 30% each to B and C.

The average capital investments of partners during the year are as follows:
A ₱100,000
B 60,000
C 120,000

The partnership earns profit of ₱100,000.

How much is the share of Partner C in the partnership profit?


a. 47,600
b. 32,200
c. 19,200
d. 33,200

Solution:
A B C Total
Amount being allocated 100,000
Allocation:
1. Salaries 12,000 8,000 20,000
2. Bonus (100K - 20K) x 10% 8,000 8,000
3. Interest on cap.
P a g e | 17

(100K x 10%);(60K x 10%);(120K x 10%) 10,000 6,000 12,000 28,000


4. Allocation of remainder:
(100K - 20K - 8K - 28K) = 44K;
(44K x 40%); (44K x 30%); (44K x 30%) 17,600 13,200 13,200 44,000
As allocated 47,600 19,200 33,200 100,000
31. The partnership agreement of A and B provides that interest at 10% per year is to be credited to each
partner on the basis of weighted-average capital balances. A summary of B’s capital account for the year
ended December 31, 20x1 is as follows:

Balance, Jan. 1, 20x1 252,000 Additional investment, July


1 72,000
Withdrawal, August 1 (27,000) Balance, Dec. 31,
20x1 297,000

How much is the interest on B’s weighted average capital?


a. 27,675
b. 33,633
c. 37,214
d. 23,322

Solution:
Balance, Jan. 1, 20x1 252,000 12/12 252,000
Additional investment, July 1 72,000 6/12 36,000
Withdrawal, August 1 (27,000) 5/12 (11,250)
Weighted average capital 276,750
Multiply by: 10%

Interest 27,675

32. Red and White formed a partnership in 2003. The partnership agreement provides for annual salary
allowances of ₱55,000 for Red and ₱45 ,000 for White. The partners share profits equally and losses in a
60/40 ratio. The partnership had earnings of ₱80 ,000 for 2003 before any allowance to partners. What
amount of these earnings should be credited to each partner’s capital account?
Red
White
a. 40,000 40,000
b. 43,000 37,000
c. 44,000 36,000
d. 45,000 35,000
Solution:
Red White Total
Amount being allocated 80,000
Allocation:
1. Salaries 55,000 45,000 100,000
2. Allocation of remaining profit
(80K profit – 100K salaries) = -20K
P a g e | 18

(-20 x 60%); (-20K x 40%) (12,000) (8,000) (20,000)


As allocated 43,000 37,000 80,000
33. Fox, Greg, and Howe are partners with average capital balances during 2002 of ₱120,000 , ₱60,000, and
₱40,000, respectively. Partners receive 10% interest on their average capital balances. After deducting
salaries of ₱30,000 to Fox and ₱20 ,000 to Howe, the residual profit or loss is divided equally. In 2003 the
partnership sustained a ₱33 ,000 loss before interest and salaries to partners. By what amount should
Fox’s capital account change? a. 7 ,000 increase.
b. 11 ,000 decrease.
c. 35 ,000 decrease.
d. 42 ,000 increase.

Solution:

Fox Greg Howe Total

Amount being allocated (33,000)


Allocation:
1. Salaries 30,000 - 20,000 50,000
2. Interest on capital 12,000 6,000 4,000 22,000
3. Allocation of balance
(-33K – 50K - 22K) = -105K / 3 (35,000) (35,000) (35,000) (105,000)
As allocated 7,000 (29,000) (11,000) (33,000)
34. The partnership agreement of Axel, Berg & Cobb provides for the year-end allocation of net income in
the following order:
• First, Axel is to receive 10% of net income up to ₱100,000 and 20% over ₱100,000.
• Second, Berg and Cobb each are to receive 5% of the remaining income over ₱150,000.
• The balance of income is to be allocated equally among the three partners.

The partnership’s 2003 net income was ₱250,000 before any allocations to partners. What amount should be
allocated to Axel?
a. 101,000
b. 103,000
c. 108,000
d. 110,000

Solution:
Axel Berg Cobb Total
Amount being allocated 250,000
Allocation:
1. Bonus to A
First 100K (100K x 10%) 10,000 10,000
Over 100K [(250K - 100K) x 20%] 30,000 30,000
2. Bonus to Berg and Cobb
(250K - 10K - 30K - 150K) x 5% 3,000 3,000 6,000
3. Allocation of bal. (204K / 3) 68,000 68,000 68,000 204,000
As allocated 108,000 71,000 71,000 250,000
35. The partnership agreement of Reid and Simm provides that interest at 10% per year is to be credited to
each partner on the basis of weighted-average capital balances. A summary of Simm’s capital account
for the year ended December 31, 2003, is as follows:
P a g e | 19

Balance, January 1 140,000


Additional investment, July
1 40,000
Withdrawal, August 1 (15,000) Balance,
December 31 165,000

What amount of interest should be credited to Simm’s capital account for 2003?
a. 15,250
b. 15,375
c. 16,500
d. 17,250

B [140K + (40K x 6/12) – (15K x 5/12) = 153.75K x 10% = 15,375

36. Blau and Rubi are partners who share profits and losses in the ratio of 6:4, respectively. On May 1 ,
2003, their respective capital accounts were as follows :

Blau 60,000
Rubi 50,000

On that date, Lind was admitted as a partner with a one-third interest in capital and profits for an investment
of ₱40,000. The new partnership began with total capital of ₱150,000 . Immediately after
Lind’s admission, Blau’s capital should be
a. 50,000
b. 54,000
c. 56,667
d. 60,000

Solution:
Total capital after admission 150,000
Multiply by: Interest of Lind 1/3
Capital credit to Lind 50,000
Contribution of Lind (40,000)
Bonus to Lind 10,000
Multiply by: Old P/L ratio of Blau 60 %

Deduction to Blau's capital 6,000

Interest of Blau before admission of Lind 60,000


Deduction to Blau's capital (6,000)

Adjusted capital of Blau after admission 54,000

37. Kern and Pate are partners with capital balances of ₱60,000 and ₱20 ,000, respectively. Profits and
losses are divided in the ratio of 60:40. Kern and Pate decided to form a new partnership with Grant,
P a g e | 20

who invested land valued at ₱15 ,000 for a 20% capital interest in the new partnership. Grant’s cost of
the land was ₱12 ,000. The partnership elected to use the bonus method to record the admission of
Grant into the partnership. Grant’s capital account should be credited for a. 12,000
b. 15,000
c. 16,000
d. 19,000

Solution: (60K + 20K + 15K) = 95K total capital after admission x 20% = 19,000
Use the following information for the next two questions:
On June 30, 2003, the condensed balance sheet for the partnership of Eddy, Fox, and Grimm, together with
their respective profit and loss sharing percentages were as follows:

320,00 Assets, net


of liabilities
0

Eddy, capital (50%) 160,000


Fox, capital (30%) 96,000
Grimm, capital (20%)

38. Eddy decided to retire from the partnership and by mutual agreement is to be paid ₱180,000 out of
partnership funds for his interest. No goodwill is to be recorded. After Eddy’s retirement, what are the
capital balances of the other partners?
Fox Grimm a. 84,000 56,000
b. 102,000 68,000
c. 108,000 72,000
d. 120,000 80,000

Solution:
Payment to Eddy 180,000
Capital balance of Eddy 160,000

Excess payment to Eddy 20,000

Capital balances before retirement Fox Grimm


96,000 64,000
Share in excess payment to Eddy
(12,000) (8,000)
Capital balances after retirement
84,000 56,000

39. Assume instead that Eddy remains in the partnership and that Hamm is admitted as a new partner
with a 25% interest in the capital of the new partnership for a cash payment of ₱140,000. The bonus
method shall be used to record the admission of Hamm. Immediately after admission of Hamm,
Eddy’s capital account balance should be a. 280,000
b. 172,500
c. 160,000
d. 140,000
P a g e | 21

Solution:
Eddy, capital 160,000
Fox, capital 96,000
Grimm, capital 64,000
Investment of Hamm 140,000
Total partnership capital after admission 460,000
Multiply by: Interest of Hamm 25 %
Capital credit to Hamm 115,000
Investment of Hamm 140,000

Bonus to old partners (25,000)

Eddy, capital (before admission) 160,000


Share in bonus to old partners (25K x 50%) 12,500
Eddy, capital (after admission) 172,500

The next two items are based on the following information:


The following condensed balance sheet is presented for the partnership of Alfa and Beda, who share profits
and losses in the ratio of 60:40, respectively:
Cash 45,000
Other assets 625,000
Beda, loan 30,000
700,000

Accounts payable 120,000


Alfa, capital 348,000
Beda, capital 232,000
700,000

40. The assets and liabilities are fairly valued on the balance sheet. Alfa and Beda decide to admit Capp as
a new partner with 20% interest. No goodwill or bonus is to be recorded. What amount should Capp
contribute in cash or other assets? a. 110,000
b. 116,000
c. 140,000
d. 145,000

D (348K + 232K) = 580K ÷ 80% = 725K capital after admission x 20% = 145,000

41. Instead of admitting a new partner, Alfa and Beda decide to liquidate the partnership. If the other assets
are sold for ₱500,000 , what amount of the available cash should be distributed to Alfa?
a. 255,000
b. 273,000
c. 327,000
d. 348,000
P a g e | 22

Solution:
The total loss on the sale is computed as follows:
Sale of other assets 500,000
Carrying amount of other assets (625,000)

Total loss on sale (125,000)

The partial settlement to partners is computed as follows:


Alpha Beda Totals
Capital balances before liquidation 348,000 232,000 580,000
Receivable from Beda (20,000) (20,000)
Total 348,000 212,000 560,000
Allocation of loss
[125K x (60% & 40%)] (75,000) (50,000) (125,000)
Amounts received by the partners 273,000 162,000 435,000
42. The statement of financial position of the partnership of A, B and C shows the following information:

Cash 22,400
Other assets 212,000
Total assets 234,400

Liabilities 38,400
A, capital (50%) 76,000
B, capital (25%) 64,000
C, capital (25%) 56,000
Total liabilities 234,400 and
equity

The partners realized ₱56 ,000 from the first installment sale of non-cash assets with total carrying amount
of ₱120,000 . How much did B receive from the partial liquidation? a. 25,000
b. 24,000
c. 16,000
d. 0

Solution:
A (50%) B (25%) C (25%) Totals
Cap. bal. before liquidation 76,000 64,000 56,000 196,000
Allocation of loss (78,000) (39,000) (39,000) (156,000)
Total (2,000) 25,000 17,000 40,000
Allocation of deficiency 2,000 (1,000) (1,000) -
Total - 24,000 16,000
43. The statement of financial position of the partnership of A, B and C shows the following information:

Cash 40,000
Other assets 720,000
Total assets 760,000

Liabilities 300,000
P a g e | 23

760,000
B, loan 64,000
C, loan 20,000
A, capital (50%) 250,000
B, capital (30%) 86,000
C, capital (20%) 40,000
Total liabilities and equity

The non-cash assets are sold for ₱320,000 . Partner C is the only solvent partner. In the settlement of the
partners’ claims, how much additional contribution is required of Partner C? a. 50,000
b. 30,000
c. 20,000
d. None

Solution:
Net proceeds 320,000
Carrying amount of all other
assets (720,000)
(400,000
Loss )

A (50%) B (30%) C (20%) Totals


Cap. bal. before liquidation 250,000 86,000 40,000 376,000
Payable to partners 64,000 20,000 84,000
Total 250,000 150,000 60,000 460,000
Allocation of loss (200,000) (120,000) (80,000) (400,000)
Total 50,000 30,000 (20,000) 520,000
Additional contribution 20,000 20,000
Total 50,000 30,000 - 540,000
“It is the Lord who goes before you. He will be with you; he will not fail you or forsake you. Do not fear or be
dismayed.”– ( Deuteronomy 31:8)

END –
P a g e | 24

INTEGRATED REVIEW 2: Advanced Financial Accounting and Reporting (AFAR)


#1 | Partnership Accounting: Nature, Formation, Operations, Dissolutions, Liquidation

1. The JPB partnership reported net income of P160,000 for the year ended December 31,
20x4. According to the partnership agreement, partnership profits and losses are to be
distributed as follows:

J P B
Salaries P 50,000 P60,000 P30,000
Bonus on net income 10% 5% 10%
Remainder (if positive) 60% 30% 10%
Remainder (if negative) 30% 40% 30%

How should partnership net income for 20x4 be allocated to J, P, and B?


J P B
A. P96,000 P48,000 P16,000 B. P58,000
P64,000 P38,000
C. P60,000 P60,000 P40,000
D. P66,000 P68,000 P46,000

(RESA, May 2018)


2. DO is admitted into the partnership of RE and MI by investing cash equivalent to ¼ of
their capital. Which of the following is true after the admission of DO?
A. Assets of the partnership will increase
B. Total partners’ equity remain the same
C. RE and MI capital decreased by ¼
D. Assets of the partnership will remain the same

(RESA, May 2018) For numbers 3-4, refer to the problem below:
The following condensed balance sheet is presented at February 18, 2018 for the partnership of Dana
and Janis, who share profits and losses in ratio of 60:40, respectfully.
Cash P150,000 Accounts payable P120,000
Non-cash assets 300,000 Dana, Capital 195,000
Dana, Loan 20,000 Janis, Capital 155,000

The non-cash assets realized P250,000 in actual liquidation


3. How much would Dana receive if cash is distributed to the partners just before the start
of actual liquidation?
A. P 5,000
B. P 18,000
C. P 30,000
D. P 0
4. How much cash would Janis receive upon final liquidation, assuming no prior cash
distribution had been made to the partners.
A. P 135,000 B. P
145,000
C. P 100,000
D. P 0
(PRTC, May 2018)
P a g e | 25

5. PP contributed P24,000 and CC contributed P48,000 to form a partnership, and they


agreed to share profits in the ratio of their original capital contributions. During the first
year of operations, they made a profit of P16,290; PP withdrew P5,050 and CC P8,000.
At the start of the following year, they agreed to admit GG into the partnership. He was
to receive a one-fourth interest in the capital and profits upon payment of P30,000 to PP
and CC, whose capital accounts were to be reduced by transfers to GG’s capital account
of amounts sufficient to bring them back to their original capital ratio.

How should the P30,000 paid by GG be divided between PP and CC?


A. PP, P 9,825; CC, P 20,175
B. PP, P 15,000; CC, P 15,000
C. PP, P 10,000; CC, P 20,000
D. PP, P 9,300; CC, P 20,700
(Dayag, 2015)

6. Scott, Joe, and Ed are liquidating their partnership. At the date the liquidation begins
Scott, Joe, and Ed have capital account balances of P162,000, P192,500, and P215,000,
respectively and the partners share profits and losses 40%, 35%, and 25%, respectively.
In addition, the partnership has a P36,000 Notes Payable to Scott and a P20,000 Notes
Receivable from Ed. When the liquidation begins, what is the loss absorption power with
respect to Joe?
A. P192, 500
B. P 67,375
C. P550,000
D. P770,000
(Dayag, 2015)
7. Which of the following is not considered a legitimate expense of a partnership?
A. Interest paid to partners based on the amount of invested capital
B. Depreciation on assets contributed to the partnership by partners
C. Salaries for management hired to run the business
D. Supplies used in the partners’ offices
(Punzalan, 2014) 8. In the AA-BB partnership, AA and BB had a capital ratio of 3:1 and
a profit and loss ratio of 2:1 respectively. The bonus method was used to record CC’s
admittance as a new partner. What ratio would be used to allocate, to AA and BB, the
excess of CC’s contribution over the amount credited to CC’s capital account? A. AA
and BB’s new relative ratio.
B. AA and BB’s new relative profit and loss ratio.
C. AA and BB’s old capital ratio.
D. AA and BB’s old profit and loss ratio.

(Dayag 2013) 9. The following is the priority sequence in which liquidation proceeds
will be distributed for a partnership:
A. Partnership drawings, partnership liabilities, partnership loans, partnership capital
balances
B. Partnership liabilities, partnership loans, partnership capital balances.
C. Partnership liabilities, partnership loans, partnership drawings, partnership capital
balances.
D. Partnership liabilities, partnership capital balances, partnership loans

(Punzalan, 2014)
10. Partnership capital and drawings accounts are similar to the corporate
A. Paid in capital, retained earnings, and dividends accounts.
B. Retained earnings accounts
C. Paid in capital and retained earnings accounts
D. Preferred and common stock accounts.
P a g e | 26

(Punzalan, 2014)

11. An advantage of the partnership as a form of business organization


would be A. Partners do not pay income taxes on their share in
partnership income.
B. A partnership is bound by the act of the partners.
C. A partnership is created by mere agreements of the partners.
D. A partnership may be terminated by the death or withdrawal of a partner.
(Punzalan, 2016)

12. In the liquidation of a partnership it is necessary to (1) distribute cash to


the partners; (2) sell non-cash assets; (3) allocate any gain or loss on
realization to the partners; and (4) pay liabilities. These steps should be
performed in the following order
A. 2,3,4,1 B.
2,3,1,4 C.
3,2,1,4
D. 3,2,4,1
(Punzalan, 2016)
13. It is the change in the relation of the partners caused by any partner ceasing to be
associated in the carrying on of the business.
A. Dissolution
B. Liquidation
C. Incorporation
D. Break-up
(Millan, 2017)
14. On January 1, 2016, Atta and Boy agreed to form a partnership contributing their
respective assets and equities subject to adjustment. On that date, the following were
provided:

Atta Boy
Cash 28,000 62,000
Accounts receivable 200,000 600,000
Inventories 120,000 200,000
Land 600,000
Building 500,000
Furniture and Fixtures 50,000 35,000
Intangible assets 2,000 3,000
Accounts Payable 180,000 250,000
Other liabilities 200,000 350,000
Capital 620,000 800,000

The ff adjustments were agreed upon:


A. Accounts receivable of P 20,000 and P 40,000 are uncollectible in A’s and B’s respective
books.
B. Inventories of P 6,000 and P 7,000 are worthless in A’s and B’s respective books
C. Intangible assets are to be written off in both books.

What will be the capital balances of the partners after adjustments?

Atta Boy
A. 592,000 750,00
B. 600,000 700,00
P a g e | 27

C. 592,000 756,300
D. 600,000 750,000
(Punzalan, 2016)

15. Partner Ae first contributed P50,000 of capital into existing partnership on March 1,
2016. On June 1, 2016, said partner contributed another P20,000. On September 1,
2016, he withdrew P15,000 from the partnership. Withdrawal in excess of P10,000
are charged to partner’s capital accounts. What is the annual weighted average
capital balance of Partner Ae?
A. 3
2
,
5
0
0
B. 5
1
,
6
6
7
C
.
6
0
,
0
0
0
D
.
4
8
,
3
3
3
(Punzalan, 2016)

16. Maxwell is trying to decide whether to accept a salary of P 40,000 or salary of P


25,000 plus a bonus of 10% of net income after salaries and bonus as a means of
allocating profit among partners. Salaries traceable to the other partners are
estimated to be P 100,000. What amount of income would be necessary so that
Maxwell would consider choices to be equal
A. 165,000 B.
290,000 C.
265,000
D. 305,000
(Punzalan, 2016)

For numbers 17 and 18 refer to the problem below:


On June 30, 2016, the condensed balance sheet for the partnership of Eddy, Fox, and Grimm
together with their respective profit and loss sharing percentage, were as follows:

Assets, net of liabilities P 320,000


P a g e | 28

Eddy, Capital (50%) P 160,000


Fox, Capital (30%) 96,000
Grimm, Capital (20%) 64,000
Total Capital P 320,000

17. Eddy decided to retire from the partnership by mutual agreement is to be paid P
180,000 out of partnership funds for his interest. Total goodwill implicit in the
agreement is to be recorded. After Eddy’s retirement, what will be capital balances
of the other partners?
Fox Grimm
A. 84,000 56,000
B. 102,000 68,000
C. 108,000 72,000
D. 120,000 80,000
(Punzalan, 2016)

18. Assume instead that Eddy remains in the partnership and that Hamm is admitted as
a new partner with 25% interest in the capital of the new partnership for a cash
payment P140,000. Total goodwill implicit in the transaction is to be recorded.
Immediately after admission of Hamm, Eddy’s capital account balance should be

A. 280,000 B.
210,000 C.
160,000
D. 140,000
(Punzalan, 2016) For numbers 9 and 10 refer to the problem below:
The ABC Partnership has assets with book value of P240,000 and a market value of
P195,000, outside liabilities of P70,000, loans payable to Partner Able of P20,000, and
capital balances for Partners Able, Baker and Chapman of P70,000, P30,000 and P50,000,
respectively. The partners share profits and losses equally.

19. How would the first P100,000 of available assets be distributed?


A. P70,000 to outside liabilities, P20,000 to able and balances equally among partners
B. P70,000 to outside liabilities, and P30,000 to Able
C. P70,000 to outside liabilities, P25,000 to Able, and P5,000 to Chapman
D. P40,000 to Able, P20,000 to Chapman, and the balance equally among partners
(Punzalan, 2017)

20. If all outside creditors and loans to partners had been paid. How would the balance
of the assets be distributed assuming Chapman had already received assets with a
value of P30,000?
A. Each of the partners would receive P30,000
B. Each of the partners would receive P40,000
C. Able: P70,000; Baker: P30,000; Chapman: P20,000
D. Able: P55,000; Baker: P15,000; Chapman: P5,000
(Punzalan, 2017)

21. If a partner’s capital balance is credited for an amount greater than or less than the
fair value of his net contribution, the excess or deficiency is called a
A. Bonus
B. Goodwill
C. Discount
D. Premium
(Millan, 2016)
P a g e | 29

22. Before allocation of loss, which of the following items are allocated first?
A. Salaries
B. Bonuses to partners
C. Interest on the capital of an industrial partner
D. All of these
(Millan, 2016)

23. After the admission of a new partner, the total partnership capital increased by the
fair value of the new partner’s net contributions to the partnership. The admission
was accounted for
A. Under the goodwill method
B. Under the bonus method
C. As a purchase of interest
D. As an investment in the partnership
(Millan, 2016)

24. On May 1, 2016, Cobb and Mott formed a partnership and agreed to share profits
and losses in the ratio of 3:7, respectively. Cobb contributed a parcel of land that
cost him P10,000. Mott contributed P40,000 cash. The land was sold for P18,000 on
May 1, 2016, immediately after formation of the partnership. What amount should
be recorded in
Cobbs’s capital account on formation of the partnership?
A. 18,000 B.
17,400 C.
15,000 D.
10,000
(Punzalan, 2018)

For numbers 5 to 6:
The Grey and Redd Partnership was formed on January 2, 2016. Under the partnership
agreement, each partner has an equal initial capital balance. Partnership net income or loss is
allocated 60% to Grey and 40% to Redd. To form the partnership, Grey initially contributed
assets costing P30,000 with a fair value of P60,000 on January 2, 2016, and Redd contributed
P20,000 cash. Drawings by the partners during 2016 totaled P3,000 by Grey and P9,000 by
Redd. The partnership net income in 2016 was P25,000.
(Punzalan 2018)

25. Under the goodwill method, what is Redd’s initial capital balance in the partnership?
A. 20,000 B.
25,000 C.
40,000
D. 60,000

26. Under the bonus method, what is the amount of bonus?


A. 20,000 bonus to Grey
B. 20,000 bonus to Redd
C. 40,000 bonus to Grey
D. 40,000 bonus to Redd

27. If a partnership has net income of P44,000 and Partner X is to be allocated bonus of
10% of income after the bonus, what is the amount of bonus Partner X will receive?
A. 3,000 B.
3,300 C.
4,000
P a g e | 30

D. 4,400
(Punzalan, 2018)

28. A partnership has the following accounting amounts:


Sales P 700,000
Cost of goods sold 400,000
Operating expenses 100,000
Salary allocations to partners 130,000
Interest paid to banks 20,000
Partners' drawings 80,000
What is the partnership net income (loss)?
A. 200,000
B. 180,000
C. 50,000
D. (30,000)
(Punzalan, 2018)
Solution:

29. Ranken purchases 50% of Lark’s capital interest in the K and L partnership for
P22,000. If the capital balances of Kim and Lark are P40,000 and P30,000,
respectively, Ranken’s capital balance following the purchase is
A. 22,000 B.
35,000 C.
20,000
D. 15,000
(Punzalan, 2018)

30. The following condensed balance sheet is presented for the partnership of Smith and
Jones, who share profits and losses in the ratio of 60:40, respectively:
Other assets P 450,000
Smith, loan 20,000
P 470,000

Accounts payable P120,000


Smith, capital 195,000
Jones, capital 155,000
P 470,000

The partners decided to liquidate the partnership. If the other assets are sold for P385,000, what
amount of the available cash should be distributed to Smith?
A. 136,000 B.
156,000 C.
159,000 D.
195,000
(Punzalan, 2018)

31. Flat and Iron partnership agreement provides for Flat to receive 20% bonus on profits
before bonus. Remaining profits and losses are divided between Flat and Iron in the
ratio 2:3, respectiviely. Ehich partner has greater advantage when the partnership has
a profit or when it has a loss
A. Profit: Flat; Loss:Iron
B. Profit:Flat; Loss: Flat
P a g e | 31

C. Profit: Iron; Loss: Flat


D. Profit: Iron; Loss: Iron
(Punzalan, 2018)

32. During 2016, Young and Zinc maintained average capital balances in their
partnership of 160000 and 100000, respectively. The partners receive 10% interest
on average capital balances and residual profit or loss is divided equally. Partnership
profit before interest was 4000. By what amount should Zinc’s capital account
change for the year?
A. 11000 decrease
B. 2000 increase
C. 1000 decrease
D. 12000 increase
(Punzalan, 2018)

Mitz, Marc and Mart are partners sharing profits in the ratio of 5:3:2, respectively. As of
December 31, 2016, their capital balances were 95,000 for Mitz, 80000 for Marc & 60000
for Mart. On Jan 1, 2017, the partners admitted Vince as a new partner and according to
their agreement, Vince will contribute 80000 in cash to the partnership and also pay 10000
for 15% of Marc’s share. Vince will be given a 20% share in profits, while the original
partners’ share will be approximately the same as before. After the admission of Vince,
the total capital will be 330000 and Vince’s Capital will be 70000

33. The total amount of goodwill to the old partners, upon the admission of Vince would
be:
A. 7
0
0
0
B. 1
5
0
0
0
C
.
2
2
0
0
0
D. 37000

34. . The balance of Marc’s Capital, after admission of Vince would be:
A.
72600 B.
74600 C.
79100
D.
81100
(Punzalan, 2018)
P a g e | 32

35. As of Dec 31, the books of AME Partnership showed capital balances of A - 40,000;
M
25,000; E-5,000. The partners’ profit or loss ratio is 3:2:1, respectively. The partners
decided to dissolve and liquidate. They sold all the non-cash assets for 37,000 cash. After
settlement of all liabilities amounting to 12,000, they still have 28,000 cash left for
distribution. The loss on realization for distribution is

A. 40,000 B.
42,000 C.
44,000
D. 45,000
(Punzalan, 2018)
36. In installment liquidation, which of the following statements is correct regarding the
partial settlement of the partners’ claims?

A. The claims of the partners and outside creditors are partially settled in proportion
B. No distribution is made to the partners until after all non cash assets are realized
C. The carrying amount of unsold non cash assets is treated as loss
D. Estimates of future liquidation costs do not affect the distribution to the partners
(Milan, 2016)
37. Under the entity theory, a partnership is

A. Viewed as having its own existence apart from the partners


B. Viewed through the eyes of the partnera
C. A separate legal and tax entity
D. Unable to enter into contracts in its own name
(FT&C)

38. Which of the following statements is true concerning the treatment of salaries in
partnership accounting?

A. The salary of a partner is treated in the same manner as salaries of corporate


employees
B. Partner salaries are equal to the annual partner draw
C. Partner salaries may be used to allocate profits and losses; they are not
considered expenses of the partnership
D. Partners salaries are directly closed to the capital account
(FT&C)

39. Which of the following is true?

A. A stipulation that excludes one or more partners from any share in the profits
or losses is valid
B. The income summary account is credited in the entry to record the distribution
of profits
C. In the absence of any agreement, salary allowances to partners shall be
provided when the operations yield losses
D. Salary and interest allowances are reported in the statement of comprehensive
income as salaries and interest expense
P a g e | 33

40. Partners C & K share profits and losses equally after each has been credited in all
circumstances with annual salary allowances of 15,000 & 12,000, respectively.
Under this arrangement, C will benefit by 3,000 more than K in which of the
following:

1. Only if the partnership has earnings of 27,000 or more for the year
2. Only If the partnership does not incur a loss for the year
3. In all earnings or loss situation
4. Only if the partnership has earnings of at least 3000 for the year
(FT&C 11e)
41. On June 30, 2015, the balance sheet of Western Marketing, a partnership, is
summarized as follows:
Sundry assets…………………………………………………………….P150,000
West, Capital…………………………………………………………….…90,000
Tern, Capital………………………………………………………………. 60,000

Wes and Tern share profit and losses at a 60:40 ratio, respectively. They agreed to take in
Cuba as a new partner, who purchases 1/8 interest of West and Tern for P25,000. What is
the amount of Cuba’s capital to be taken up in the partnership books if the book value
method is used?
A. P12,500 B.
P18,750 C.
P25,000 D.
P31,250
(Dayag, 2015)

42. In the AD partnership, Allen’s capital is P140,000 and Daniel’s is P40,000 and they
share income in a 3:1 ratio, respectively. They decide to admit David to the
partnership.

Allen and Daniel agree that some of the inventory is obsolete. The inventory account is
decreased before David is admitted. David invests P40,000 for a one-fifth interest. What
is the amount of inventory written down?

A. P
4,
0
0
0
B. P
1
0,
0
0
0
C
.
P
1
5,
0
0
0
D. P20,000
(Dayag, 2015)
P a g e | 34

43. In the AD partnership, Allen’s capital is P140,000 and Daniel’s is P40,000 and they
share income in a 3:1 ratio, respectively. They decide to admit David to the
partnership. David directly purchases one-fifth interest by paying Allen P34,000 and
Daniel P10,000. The land account is increased before David is admitted. By what
amount is the land account increased?

A. P40,000 B.
P36,000 C.
P20,000
D. P10,000
(Dayag, 2015)

44. RR and XX formed a partnership and agreed to divide initial capital equally, even
though RR contributed P25,000 and XX contributed P21,000 in identifiable assets.
Under the bonus method approach to adjust the capital accounts, XX’s unidentifiable
assets should be debited for:

A. P11,500
B. P4,000
C. P2,000
D. 0
(Dayag, 2015)

45. Partner A first contributed P50,000 of capital into an existing partnership on March
1, 2015. On June 1, 2015, the partner contributed another P20,000. On September 1,
2015, the partner withdrew P15,000 from the partnership. Withdrawals in excess of
P10,000 are charged to the partner’s capital account. The annual weighted-average
capital balance is

A. P62,000 B.
P51,667 C.
P60,000
D. P48,333
(Dayag, 2015)

46. For financial accounting purposes, assets of an individual partner contributed to a


partnership are recorded by the partner at

A. Historical cost
B. Book value
C. Fair market value
D. Lower of cost or market
(Dayag, 2015)

47. Which of the following interest component calculation bases is the least susceptible
to manipulation when allocating profits and losses to partners?

A. Beginning capital account balance


B. Average of beginning and ending capital balance
C. Weighted average capital account balance
D. Ending capital balance
(Dayag, 2015)
P a g e | 35

48. In a partnership, interest on capital investment is accounted for as a(n)

A. Return on investment
B. Expense
C. Allocation of net income
D. Reduction of capital
(Dayag, 2015)

49. What is the underlying purpose of the interest on capital balances component of
allocating partnership profits and losses?

A. Compensate partners who contribute economic resources to the partnership


B. Reward labor and expertise contributions
C. Reward for special responsibilities taken
D. None of the above
(Dayag, 2015)
50. What is the underlying purpose of the salary component of allocating partnership
profits and losses?

A. Compensate partners who contribute economic resources to the partnership


B. Reward labor and expertise contributions
C. Reward for special responsibilities taken
D. None of the above
(Dayag, 2015)

51. A, B, and C are partners in an accounting firm. Their capital account balances at year-
end were A P90,000; B P110,000; C P50,000. They share profits and losses on a 4:4:2
ratio, after the following special terms:
• Partner C is to receive a bonus of 10% of net income after the bonus.
• Interests of 10% shall be paid on that portion of a partner’s capital in excess of P100,000
• Salaries of P10,000 and P12,000 shall be paid to partners A & C respectively.
Assuming a net income of P44,000 for the year, the total profit share of Partner C was:
A. P7,800
B. P16,800
C. P19,400
D. P19,800
(Dayag, 2015)

52. X and Y are in partnership, sharing profits equally and preparing their accounts to 31
December each year. On 1 July 2015, Z joined in the partnership, and from that date
profits are shared X 40%, Y 40%, and Z 20%.

In the year ended 31 December 2015, profits were:


6 months to 31 June 2015 P200,000
6 months to 31 December 2015 P300,000

It was agreed that X and Y only should bear equally the expense for a bad debt of P40,000 written-
off in the six months to 31 December 2015 in arriving at the P300,000 profit.

Which of the following correctly states X’s profit share for the year?
A. P216,000 B. P200,000
C. P220,000
D. P224,000
(Dayag, 2015)
P a g e | 36

53. Pol and Loc are partners with capitals of P200,000 and P100,000 and sharing profits
and losses 3:1 respectively. They agree to admit Chic as partner. Chic invests P125,000
for a 25% interest in the firm. Parties agree that the total firm capital after Chic’s
admission is to be P425,000.

The capital balances of the partners after Chic’s admission are:


A. Pol, P214,062.50; Loc, P104,687.50; and Chic, P106,250
B. Pol, P200,000.00; Loc, P100,000.00; and Chic, P125,000
C. Pol, P239,062.50; Loc, P 79,687.50; and Chic, P125,000
D. Pol, P250,000.00; Loc, P125,000.00; and Chic, P100,000
(Guerrero, 2013)

54. Michelle, an active partner in the Michelle-Esme partnership receives an annual bonus
of 25% of the partnership income after deducting the bonus. For the year ended,
December 31, 2013, partnership income before the bonus amounted to P240,000. The
bonus of Michelle for the year 2013 is:
A. P45,000
B. P48,000
C. P80,000
D. P60,000
(Guerrero, 2013)

55. Rita, Sisa, and Tina are partners with the capital balances on June30, 2013 of P60,000,
P60,000 and P40,000, respectively. Profits and losses are shared equally. Tina
withdraws from the partnership. The partners agree that Tina is to take certain furniture
at their secondhand value of P2,400 and cash for the balance of her interest. The
furniture is carried on the books as fully depreciated.

The amount of cash to be paid to Tina and the capital balances of the remaining partners after
the retirement of Tina are:
Cash Rita, capital Sisa, capital
A. P40,000 P60,000 P60,000
B. P37,000 P61,200 P61,200
C. P38,400 P60,800 P60,800
D. P42,800 P58,800 P58,800
(Guerrero, 2013)

For question 55-56 refer to the problem below


The AA, BB, CC Partnership was formed on January 2. 2019. The original cash investments were as
follows:
AA P 48,000
BB P 72,000
CC P108,000
According to the general partnership contract, the partners were to be remunerated as follows:
a. Salaries of P72,000 for AA, P6,000 for BB, and P6,800 for CC.
b. Interest at 12% on the average capital account balances during the year.
c. Remainder divided 40% to AA, 30% to BB, and 30% for CC.
Income before partners’ salaries for the year ended December 31, 2019, was P46,040. AA
invested an additional P12,000 in the partnership on July 1; CC withdrew P18,000 from
the partnership on October 1, and, as authorized by the partnership contract, AA, BB, and
CC each withdrew P375 monthly against their shares of net income for the year.

56. Determine the share of partner AA in the net income:


P a g e | 37

A. P18,416 B. P17,616
C. P13,080
D. P5,880
(ReSA, 2018)

57. Determine the capital balance of partner CC on December 31, 2019:


A. P108,770 B. P104,270
C. P100,112
D. P99,312
(ReSA, 2018)

58. If the salaries to partners are to be recognized as operating expenses by the partnership,
what amount is the share of partner BB in the net income?
A. P18,416
B. P14,190
C. P8,190 D. P7,812
(ReSA, 2018)

(For question 59 – 60)


DD and EE was organized and began operations of March 1, 2019. On that date, DD
invested P75,000 and EE invested land and building with current fair value of P40,000
and P50,000, respectively. EE also invested P30,000 in the partnership on November 1,
2019 because of its shortage of cash. The partnership contract includes the following
remuneration plan:
DD EE
Annual Salary P9,000 P12,000
Annual interest on average capital account balance 10% 10%
Remainder 60% 40%

The annual salary was to be withdrawn by each partner in 12 monthly installments. During
fiscal year ended, February 28, 2020, DD and EE had net sales of P25,000, cost of goods
sold of P140,000 and total operating expenses of P50,000 (excluding partners’ salaries
and interest on average capital account balances). Each partner made monthly cash
drawings in accordance with partnership contract.

59. Determine the share of partner DD in the net income:


A. P29,400 B. P33,000 C. P36,000 D. P23,400
(ReSA, 2018)

60. The capital balance of each partner on March 1, 2020 should be:
A. DD P95,400; EE P138,600
B. DD P66,000; EE P82,000
C. DD P108,000; EE P147,000
D. DD P99,000; EE P135,000
(ReSA, 2018)

61. Which of the following is not considered a legitimate expense of a partnership?


a. Supplies used in the partners’ office
b. Depreciation on assets contributed to the partnership by partners
c. Salaries for management hired to run the business
d. Interest paid to partners based on the amount of invested capital

Millan Textbook (2016)


P a g e | 38

62. If the partnership agreement does not specify how income is to be allocated, profits and
loss should be allocated A. Equally.
B. In proportion to the weighted average of capital invested during the period.
C. Equitably so that partners are compensated for the time and effort expended on behalf
of the partnership.
D. In accordance with their capital contributions.

(Millan, 2016)
63. When Mill retired from the partnership of Mill, Yale, and Lear, the final settlement of
Mill’s interest exceeded Mill’s capital balance. Under the bonus method, the excess
A. Was recorded as goodwill.
B. Was recorded as an expense.
C. Reduced the capital balances of Yale and Lear.
D. Had no effect on the capital balances of Yale and Lear.

(Millan, 2016)

64. State the correct order of the claims on the personal assets of a partner,
I. The partner’s separate creditors
II. To the other partner’s by way of contribution
III. The partnership creditors
A. I, III, II B. I, II,
III C. III, II, I
D. II, I, III

(Millan, 2016)

65. It is the change in the relation of the partners caused by any partner ceasing to be
associated in the carrying on of the business.
A. Dissolution
B. Liquidation
C. Incorporation
D. Break-up

(Millan, 2016)

66. MM, NN, and OO are partners with capital balances on December 31, 2012 of
P300,000, P300,000 and P200,000, respectively. Profits are shared equally. OO
wishes to withdraw and it is agreed that OO is to take certain equipment with
second-hand value of P50,000 and a note for the balance of OO’s interest. The
equipment are carried on the books at P65,000. Brand new equipment may cost
P80,000. Compute for: (1) OO’s acquisition of the second-hand equipment will
result to reduction in capital; (2) the value of the note that will OO get from the
partnership’s liquidation.
A. (1) P 15,000 each for MM and NN, (2) P150,000
B. (1) P5,000 each for MM, NN and OO, (2) P145,000
C. (1) P5,000 each for MM, NN and OO, (2) P195,000
D. (1) P7,500 each for MM and NN, (2) P145,000
(Dayag, 2013)
67. The partnership agreement of XX, YY and ZZ provides for the year-end allocation
of net income in the following order:
• First, XX is to receive 10% of net income up to P200,000 and 20% over P200,000.
• Second, YY and ZZ each are to receive 5% of the remaining income over
P300,000.
• The balance of income is to be allocated equally among the three partners.

The partnership’s 2011 net income was P500,000 before any allocations to partners. What amount
should be allocated to XX?
P a g e | 39

A. P202,000 B.
P216,000 C.
P206,000
D. P220,000

(Dayag, 2013)

68. RR and XX formed a partnership and agreed to divide initial capital equally, even
though RR contributed P25,000 and XX contributed P21,000 in identifiable assets.
Under the bonus approach to adjust the capital accounts. XX’s unidentifiable
assets should be debited for:
A. P
1
1
,
5
0
0
B. P
4
,
0
0
0
C
.
P
2
,
0
0
0
D. P 0

(Dayag, 2013)

69. As of December 31, 2012, the books of Ton Partnership showed capital balances
of: T,
P40,000; O, P25,000; N, P5,000. The partner’s profit and loss ratio was 3:2:1,
respectively. The partners decided to liquidate and they sold all non-cash assets for
P37,000. After settlement of all liabilities amounting P12,000, they still have cash of
P28,000 left for distribution. Assuming that any capital debit balance is uncollectible, the
share of T in the distribution of the P28,000 cash would be:
A. P17,800 B.
P18,000 C.
P19,000
D. P17,000

(Dayag, 2013)

70. CC, PP and AA, accountants, agree to form a partnership and to share profits in
the ratio of 5:3:2. They also agreed that AA is to be allowed a salary of P28,000,
and that PP is to
be guaranteed P21,000 as his share of the profits. During the first year of operation,
income from fees are P180,000, while expenses total P96,000. What amount of net income
should be credited to each partner’s capital account?
A. CC, P28,000, PP, P16,800, AA, P11,200 B. CC,
P25,000, PP, P21,000, AA, P38,000 C. CC, P24,000,
PP, P22,000, AA, P38,000
P a g e | 40

D. CC, P25,000, PP, P21,000, AA, P39,000

(Dayag, 2013)

71. Allen retired from the partnership of Allen, Beck and Chale. Allen’s cash
settlement from the partnership was based on new goodwill determined at the date
of retirement plus the carrying amount of the other net assets. As a consequence
of the settlement, the capital accounts of Beck and Chale were decreased. In
accounting for Allen’s withdrawal, the partnership could have used the:

BONUS METHOD GOODWILL METHOD


A. No Yes
B. No No
C. Yes Yes
D. Yes No
(Milan, 2016)

72. Which of the following has the least priority of payment in case of partnership
liquidation?

A. Priority claims such as artisans. Government, liquidation expenses


B. Secured creditors to the extent of covered by the proceeds from the sale of pledged
assets.
C. Unsecured credit to the extent covered by proceeds from sale of unpledged (or free)
assets.
D. The partners’ capital balances.

(Milan 2016)

73. State the proper order of liquidation

I Outside creditors

II Owners’ interests

III Inside creditors


A. I, III, II
B. I, II, III
C. III, II, I
D. II, I, III

(Milan, 2016)

74. According to the Philippine Civil Code, if only the shares of each partner in the
profits has been agreed upon, the share of each in the losses shall be

A. equally
B. equally, but the industrial partner shall not share in the loss

C. the same as the sharing in profits


D. the same as the sharing in profits. However, the industrial partner shall not share in
the loss.

(Milan, 2016)
P a g e | 41

75. Which of the following is not considered a legitimate expense of a partnership?

A. Supplies used in the partners’ offices.

B. Depreciation on assets contributed to the partnership by partners.

C. Salaries for management hired to run the business.

D. Interest paid to partners based on the amount of invested capital.


(Milan, 2016)

76. CC, PP and AA, accountants, agree to form a partnership and to share profits in
the ratio of 5:3:2. They also agreed that AA is to be allowed a salary P28,000 and
that PP is to be guaranteed P21,000 as his share of the profits. During the first year
of operation, income from fees are P180, 000, while expenses total P96,000. What
amount of net income should be credited to each partner’s capital account?

A. CC, P28,000 PP, P16,800 AA, P11,200


B. CC, P25,000 PP, P21,000 AA, P38,000
C. CC, P24,000 PP, P22,000 AA, P38,000
D. CC, P25,000 PP, 21, 000 AA, P39, 000

(Dayag,
2015)

77. The same information in Number 32, except the partnership had a loss of P125,
624 after the interest and salaries to partners, by what amount should BB’s capital
account change increase(decrease)?

A. P (115,443) C. P (41,875)
B. 23,865 D. (18,010)
(Dayag,
2015)

78. PP, QQ and RR, partners to a firm, have capital balances of P11, 200, P13, 000
and P5, 800, respectively, and share profits in the ratio of 4:2:1. Prepare a schedule
showing how available cash will be given to the partners as it becomes available.
Who among the partners shall be paid first with an available cash of P1, 400?
A. QQ B. No One C. RR D. PP

79. The August, Albert and Gerry partnership became insolvent on January 1, 2015,
and the partnership is being liquidate as as practicable. In this respect the following
information for the partners has been marshaled.

Capital Balances Personal Assets Personal Liabilities


August P 70,000 P 80,000 P 40,000
P a g e | 42

Albert - 60,000 30,000 50,000


Gerry - 30,000 70,000 30,000

Assume that residual profits and losses are shared equally among the three partners.
Based on this information, calculate the maximum amount that August can expect to
receive from the partnership liquidation is:

A. P20, 000 C. P70,000


B. 40,000 D. 110,000

(Dayag, 2015)

80. RR and XX formed a partnership and agreed to divide initial capital equally, even
though RR contributed P25,000 and XX contributed P21,000 in identifiable assets.
Under the bonus approach to adjust the capital accounts. XX’s unidentifiable
assets should be debited for:

A. 11,500 C.
2,000 D. 0
B. 4,000

(Dayag, 2015)

81. Partnership capital and drawing accounts are similar to the corporate

A. Paid-in capital, retained earnings, and dividend accounts.


B. Retained earnings account.
C. Paid-in capital and retained earnings accounts.
D. Preferred and common stock accounts.
(Gleim)

82. The partnership agreement is an express contract among the partners (the owners
of the business). Such an agreement generally does not include

A. A limitation on a partner’s liability to creditors.


B. The rights and duties of the partners.
C. The allocation of income between the partners.
D. The rights and duties of the partners in the event of partnership dissolution.
(Gleim)

83. A partnership records a partner’s investment of assets in the business at

A. The market value of the assets invested.


B. A special value set by the partners.
C. The partner’s book value of the assets invested.
D. Any of the above, depending upon the partnership agreement.
(RPCPA 0598)
P a g e | 43

84. Assume that C has a P50,000 equity in the partnership of “A, B, and C.” Partner
C arranges to sell his entire interest to D for P80,000 Cash. Partners A and B agree
to the admission of D. At what amount will the equity of the incoming partner, D,
be shown in the balance sheet?

A. at P50,000.
B. at P50,000 and the P30,000 will be divided equally among the original partners.
C. at P80,000
D. at P80,000 and the P30,000 will represent Goodwill which will be apportioned between
E. the existing equities of A and B.
(RPCPA 107)

Albion and Blaze share profits and losses equally. Albion and Blaze receive salary
allowances of $20,000 and $30,000, respectively, and both partners receive 10% interest
on their average capital balances. Average capital balances are calculated at the beginning
of each month balance regardless of when additional capital contributions or permanent
withdrawals are made subsequently within the month. Partners’ drawings are not used in
determining the average capital balances. Total net income for 2006 is $120,000.

Albion Blaze
January 1 capital balances $ 100,000 $ 120,000
Yearly drawings ($1,500 a month) 18,000 18,000
Permanent withdrawals of capital:
June 3 ( 12,000 )
May 2 ( 15,000 )
Additional investments of capital:
July 3 40,000
October 2 50,000

85. What is the weighted-average capital for Albion and Blaze in 2006?

A. $100,000 and
$120,000. B.
$105,333 and
$126,667. C.
$110,667 and
$119,583.
D. $126,667 and $105,333.
(Beams, 2009)
86. If the average capital for Albion and Blaze from the above information is
$112,000 and $119,000, respectively, what will be the total amount of profit
allocated after the salary and interest distributions are completed?

A. $70,000. B.
$73,100. C.
$75,000.
D. $80,000.
(Beams, 2009)

87. If the average capital balances for Albion and Blaze are $100,000 and
$120,000, what will the final profit allocations for Albion and Blaze in 2006?
P a g e | 44

A. $50,000 and
$70,000. B.
$54,000 and
$66,000. C.
$70,000 and
$50,000.
D. $75,000 and $45,000.
(Beams, 2009) For number 88 to 89 refer to the problem below:
Bloom and Carnes share profits and losses in a ratio of 2:3, respectively. Bloom and
Carnes receive salary allowances of $10,000 and $20,000, also respectively, and both
partners receive 10% interest based upon the balance in their capital accounts on January
1. Partners’ drawings are not used in determining the average capital balances. Total net
income for 2006 is $60,000. If net income after deducting the interest and salary
allocations is greater than $20,000, Carnes receives a bonus of 5% of the original amount
of net income.

Bloom Carnes
January 1 capital balances $ 200,000 $ 300,000
Yearly drawings ($1,500 a month) 18,000 18,000

88. What are the total amounts for the allocation of interest, salary, and bonus, and,
how much over-allocation is present?

A. $60,000 and $0.


B. $80,000 and $20,000.
C. $83,000 and $0.
D. $83,000 and $23,000.
(Beams, 2009)

89. The XYZ partnership provides a 10% bonus to Partner Y that is based upon
partnership income, after deduction of the bonus. If the partnership's income is
$121,000, how much is Partner Y's bonus allocation?

A. $11,000. B.
$11,450. C. $11,650.
D. $12,100.
(Beams, 2009)

Lara, Ives, and Jack are in the process of liquidating their partnership. Since it may take
several months to convert the other assets into cash, the partners agree to distribute all
available cash immediately, except for $10,000 that is set aside for contingent expenses.
The balance sheet and residual profit and loss sharing percentages are as follows:

Cash $ 400,000 Accounts payable $ 200,000


Other assets 200,000 Hara, capital (40%) 135,000
Ives, capital (30%) 216,000
Jack, capital (30%) 49,000

Total assets $ 600,000 Total liab./equity $ 600,000


P a g e | 45

90. How much cash should Ives receive in the first distribution?

A. $146,000. B. $147,000.
C. $153,000.
D. $156,000
(Beams, 2009)

91. Which of the following transactions shall not affect the capital balance of a
partner?
A. Share of a partner in the partnership’s net loss
B. Receipt of bonus by a partner from another partner based on the agreement
C. Advances made by the partnership to a partner
D. Additional investment by a partner to the partnership

CPAR Testbank

92. Which of the following will not result to the dissolution of a partnership?
E. Insolvency of the partnership
F. Admission of a new partner in an existing partnership
G. Assignment of an existing partner’s interest to a third person
H. Retirement of a partner

CPAR Testbank

93. He refers to a partner who contributed not only money and property but also
industry to the newly formed partnership.
A. Industrial partner
B. Nominal partner
C. Capitalist-industrial partner
D. Capitalist partner
CPAR 2017 Pre-Board

94. It refers to a type of partnership wherein all partners are liable to the creditors pro-
rata up to the extent of personal or separate assets after the partnership’s assets are
exhausted.
A. General partnership
B. Partnership by estoppel
C. Limited partnership
D. Particular partnership
CPAR 2017 Pre-Board

95. Which of the following statements concerning the formation of partnership


business is correct?
A. Philippine Financial Reporting Standards (PFRS) allows recognition of goodwill
arising from the formation of partnership.
B. The juridical personality of the partnership arises from the issuance of certification of
registration.
C. The parties may become partners only upon contribution of money or property but
not of industry or service.
P a g e | 46

D. The capital to be credited to each partner upon formation may not be the amount
actually contributed by each partner.
CPAR 2017 Pre-Board

96. The partners, C and D, share profits 3:2. However, C is to receive a yearly
bonus of 20% of the profits, in addition to his profit share. The partnership
made a net income for the year of P960,000 before the bonus. Assuming C’s
bonus is computed on profit after deducting said bonus, how much profit
share will D receive?
A. P307,200 B.
P320,000 C.
P640,000
D. P160,000
CPAR 2017 Pre-Board

97. A, B, and C are partners and share profits and losses as follows: Salaries of
P40,000 to A; P30,000 to B; and none to C. If net income exceeds salaries,
then a bonus is allocated to A. The bonus is 5 percent of net income after
deducting salaries and the bonus. Residual profits or residual losses are
allocated 10 percent to A, 20 percent to B, and 70 percent to C. If net income
before salaries and bonus is P140,000, how much is the share of A?
A. P50,150 B.
P43,333 C.
P46,667
D. P50,000
CPAR 2017 Pre-Board

98. Carson and Lamb establish a partnership to operate and used-furniture


business under the name of C & L Furniture. Carson contributes furniture
that cost P60,000 and has a fair value of P90,000. Lamb contributes P30,000
cash and delivery equipment that cost P40,000 and has a fair value of
P30,000. The partners agree to share profits and losses 60% to Carson and
40% to Lamb. Calculate the peso amount of inequality that will result if the
initial noncash contributions of the partners are recorded at cost rather than
fair market value.
A. P30,000 B.
P10,000 C.
P20,000
D. P18,000
CPAR 2017 Pre-Board Items 99 and 100 are based on the following:
Lucy and Annie were partner sharing profits and losses equally. Ochie was admitted as a
partner by contributing cash of P60,000 for one-third interest in the firm. They agreed to
set the total capital at P210,000 after Ochie’s admission. Prior to Ochie’s admission, the
old partner’s capital accounts were Lucy, P48,000, and Annie, P96,000.

99. The capital balance of Annie after Ochie’s admission was


A. P92,667 B. P94,000 C. P91,000
D. P96,000
100. Assuming that Ochie will share one-fourth interest on the partnership assets
the capital balance of Annie after Ochie’s admission is
A. P96,000
P a g e | 47

B. 99,750
C. P99,000
D. P102,750
CPAR 2017 Pre-Board

101. A partnership agreement calls for allocation of profits and losses by salary
allocations, a bonus allocation, interest on capital, with any remainder to be
allocated by preset ratios. If a partnership has a loss to allocate, generally
which of the following procedures would be applied?

A. Any loss would be allocated equally to all partners.


B. Any salary allocation criteria would not be used.
C. The bonus criteria would not be used.
D. The loss would be allocated using the profit and loss ratios, only.
(Fisher, 2008)

102. Della Reise was admitted to a partnership. She contributed $25,000 cash plus
equipment she purchased for $50,000 and which had accumulated
depreciation for tax purposes of $20,000. The fair value of the equipment
was $35,000. She also assumed 1/3 of partnership debt of $15,000. Her
beginning capital balance was $48,000. For tax purposesher partnership
interest should be initially valued at

A. $60,000 B.
$48,000 C.
$55,000 D.
$65,000
(Fisher, 2008)

103. Under the bonus method, when a new partner is admitted to the partnership,
the total capital of the new partnership is equal to:

A. the book value of the previous partnership + the fair market value of the consideration
paid to the existing partnership by the incoming partner
B. the book value of the previous partnership + any necessary asset write ups from book
value to market value + the fair market value of the consideration paid to the existing
partnership by the incoming partner
C. the book value of the previous partnership - any asset write downs from book to market
value + the fair market value of the consideration paid to the existing partnership by the
incoming partner
D. the fair market value of the new partnership as implied by the value of the incoming
partner's consideration in exchange for an ownership percentage in the new partnership
(Fisher, 2008)

104. Assume that a partnership had assets with a book value of $240,000 and a
market value of $195,000, outside liabilities of $70,000, loans payable to
partner Able of $20,000, and capital balances for partners Able, Baker, and
Chapman of $70,000, $30,000, and $50,000. How would the first $100,000
of available assets be distributed assuming profits and losses are allocated
equally?

A. $70,000 to outside liabilities, $20,000 to Able, and the balance equally among the partners
B. $70,000 to outside liabilities and $30,000 to Able
C. $70,000 to outside liabilities, $25,000 to Able, and $5,000 to Chapman
P a g e | 48

D. $40,000 to Able, $20,000 to Chapman, and the balance equally among the partners
(Fisher, 2008)

105. Partners Dalton, Edwards, and Finley have capital balances of $40,000,
90,000 and $30,000, respectively, immediately prior to liquidation. Total
remaining assets have a book value of $160,000, the liabilities having been
paid. Among these remaining assets is a machine with a fair value of
$35,000. The partners split profits and losses equally. Edwards covets the
machine and is willing to accept it for $35,000 in lieu of cash. The other
partners have no designs on specific assets, only cash in liquidation. How
much cash, in addition to the machine, would be first distributed to Edwards,
before any of the other partners received anything?

A. $15,000
B. $50,000
C. $166,667
D. $300,000
(Fisher, 2008)

106. Partners Thomas, Adams and Jones have capital balances of $24,000,
$45,000, and $90,000 respectively. They split profits in the ratio of 3:3:4,
respectively. Under a predistribution plan, one of the partners will get the
following total amount in liquidation before any other partners get anything:

A. $22,500 B.
$30,000 C.
$40,000 D.
$75,000
(Fisher, 2008)

107. Which of the following statements are true when comparing corporations
and partnerships?

A. Partnership entities provide for taxes at the same rates used by corporations.
B. In theory, partnerships are more able to attract capital.
C. Like corporations, partnerships have an infinite life.
D. Unlike shareholders, general partners may have liability beyond their capital balances.

108. Which of the following characteristics of a partnership most likely explains


why a public accounting firm is organized as a partnership from a public
policy viewpoint?

A. A partnership is not a taxable entity.


B. A partnership is characterized by unlimited liability.
C. A partnership is characterized by a fiduciary relationship among the partners.
D. Salaries to the partners are not considered a component of net income.
(Fisher, 2008)

109. The partnership agreement is an express contract among the partners (the
owners of the business). Such an agreement generally does not include:

A. A limitation on a partner’s liability to creditors.


B. The rights and duties of the partners.
C. The allocation of income between the partners.
P a g e | 49

D. The rights and duties of the partners in the event of partnership dissolution.

(Punzalan, 2016)

110. A partnership records a partner’s investment of assets in the business at

A. The market value of the assets invested.


B. A special value set by the partners.
C. The partner’s book value of the assets invested.
D. Any of the above, depending upon the partnership agreement.
(Punzalan, 2016)

111. When property other than cash is invested in a partnership, at what amount
should the noncash property be credited to the contributing partner’s capital
account?

A. Fair value at the date of recognition.


B. Contributing partner’s original cost.
C. Assessed valuation for property tax purposes. D. Contributing partner’s tax basis.

(Punzalan, 2016)

112. X, Y, Z are capitalist partners and D an industrialist partner. The partnership


reported a net loss of P200,000. How much is the share of D?

A. 0
B. 10,000
C. 25,000
D. 100,000
(Punzalan, 2016)

113. Assume that C has a P715,000 equity in the partnership of “A, B, and C.”
Partner C arranges to sell his entire interest to D for P80,000 Cash. Partners
A and B agree to the admission of D.At what amount will the equity of the
incoming partner, D, be shown in the balance sheet?

A. at P715,000.
B. at P50,000 and the P30,000 will be divided equally among the original partners.
C. at P80,000
D. at P80,000 and the P30,000 will represent Goodwill which will be apportioned
between
E. the existing equities of A and B.
(Punzalan, 2016)

114. Partner Morgan is personally insolvent, owing P600,000. Personal assets


will only bring P200,000 when liquidated. At the same time, Morgan has a
credit balance in the partnership of P120,000. The capital amounts of the
other partners total a credit balance of P250,000. Under the doctrine of
marshaling of assets, how much the personal creditors of Morgan can
collect?
P a g e | 50

A. 120,000 B.
200,000
C. 320,000
D. 570,000
(Punzalan, 2016)

For Numbers 115 to 116 refer to the problem below


As of December 31, the books of AME Partnership showed capital balances of A-P40,000; M-
P25,000; And E-P5,000. The partners’ profit and loss ratio were 3:2:1, respectively. The
partners decided to dissolve and liquidate. They sold all the non-cash assets for P37,000
cash. After settlement of all liabilities amounting to P12,000, they still have P28,000 cash
left for distribution.

115. The loss on the realization of the non-cash assets was

A. P40,000 B. P42,000
C. P44,000
D. P45,000

(Punzalan, 2016)

116. Assuming that any partner’s capital debit balance is uncollectible, the share
of A in the P28,000 cash for distribution would be

A. P19,000 B. P18,000
C. P17,800
D. P40,000

(Punzalan, 2016)

117. The following balance sheet is presented for the partnership A, B and C, who
share profits and losses in the respectively ratio of 5:3:2

Assets Liabilities and Capital


Cash Php 120,000 Liabilities Php 280,000
Other Assets 1,080,000 A, Capital 560,000
B, Capital 320,000
C, Capital 40,000

Total Php 1,200,000 Total Php 1,200,000

Assume the three partners decided to liquidate the partnership. If the other assets are sold for
P800,000, how should the available cash be distributed to each partner?
A B C
A. 280,000 320,000 40,000
B. 324,000 236,000 16,000
C. 410,000 230,000 0
D. 412,000 228,000 0

(Punzalan, 2016)
P a g e | 51

118. Partners Almond, Barney and Colors have capital balances of P20,000, P50,000,
and P90,000, respectively. They split profits in the ratio of 2:4:4, respectively.
Under a safe cash distribution plan, one of the partners will get the following
total amount in liquidation before any other partners get anything
A. 0
B. 15,000
C. 40,000
D. 180,000

(Punzalan, 2016)

119. Methods exist for the division of partnership profits and losses

A. Equally
B. Arbitrary ratio
C. Capital contribution ratio
D. All of the above
(Dayag, 2015)
120. For a partner to withdraw or retire from the partnership, the total interest of a
partner should be properly determined which includes:

A. Share in the profit or loss of the partnership


B. Adjustments in assets and liabilities to reflect fair market values
C. Loans to and from partnership
D. All of the above
(Dayag, 2015)
121. On December 1, 2015, EE and FF formed a partnership, agreeing to share for
profits and losses in the ratio of 2:3, respectively. EE invested a parcel of land
that cost him P25,000. FF invested P30,000 cash. The land was sold for P50,000
on the same date, thress hours after formation of the partnership. How much
should be the capital balance of EE right after formation?

A. P25,000 B. P30,000 C. P60,000


D. P50,000
(Dayag, 2015)

122. MM, NN, and OO are partners with capital balances on December 31, 2015 of
P300,000, P300,000 and P200,000, respectively. Profits are shared equally. OO
wishes to withdraw and it is agreed that OO is to take certain equipment with
second-hand value of P50,000 and a note for the balance of OO’s interest. The
equipment are carried on the books at P65,000. Brand new equipment may cost
P80,000. Compute for: (1) OO’s acquisition of the second-hand equipment will
result to reduction in capital; (2) the value of the note that will OO get from the
partnership’s liquidation.

A. (1) P15,000 each for MM and NN, (2) P150,000


B. (1) P5,000 each for MM, NN and OO, (2) P145,000
C. (1) P5,000 each for MM, NN and OO, (2) P195,000
D. (1) P7,500 each for MM and NN, (2) P145,000
(Dayag, 2015)

123. JJ and KK are partners who share profits and losses in the ratio of 60%: 40%,
respectively. JJ’s salary is P60,000 and P30,000 for KK. The partners are also
paid interest on their average capital balances. In 2015, JJ received P30,000 of
interest and KK, P12,000. The profit and loss allocation is determined after
P a g e | 52

deductions for the salary and interest payments. If KK’s share in the residual
income (income after deducting salaries and interest) was P60,000 in 2015, what
was the total partnership income?
A. P192,000 B.
P345,000 C. P282,000
D. P387,000
(Dayag, 2015)

124. CC, PP, and AA, accountants, agree to form a partnership and to share profits in
the ratio of 5:3:2. They also agreed that AA is to be allowed a salary of P28,000,
and that PP is to be guaranteed P21,000 as his share of the profits. During the
first year of operation, income from fees are P180,000, while expenses total
P96,000. What amount of net income should be credited to each partner’s capital
account?

A. CC, P28,000, PP, P16,800, AA, P11,200 B. CC,


P25,000, PP, P21,000, AA, P38,000 C. CC, P24,000,
PP, P22,000, AA, P38,000
D. CC, P25,000, PP, P21,000, AA, P39,000
(Dayag, 2015)

125.The following condensed balance sheet is presented for the partnership of AA, BB, and
CC, who share profits and losses in the ratio of 4:3:3, respectively:

Cash P160,000
Other Assets 320,000
Total P480,000

Liabilities P180,000
AA, capital 48,000
BB, capital 216,000
CC, capital 36,000
Total P480,000

The partners agreed to dissolve the partnership after selling the other assets for P200,000. Upon
dissolution of the partnership, AA should have received

A. P
0
B. P
4
8,
0
0
0
C
.
P
7
2,
0
0
0
D. P84,000
(Dayag, 2015)
P a g e | 53

126. Larry, Marsha, and Natalie are partners in accompany that is being liquidated. They
share profits and losses 55 percent, 20 percent, and 25 percent, respectively. When the
liquidation begins, they have capital account balances of P108,000, P62,000 and P56,000,
respectively. The partnership just sold equipment with a historical cost and accumulated
depreciation of P25,000 and P18,000, respectively for P10,000. What is the balance in
Marsha’s capital account after the transaction is completed?

A. P62,000 B.
P61,400 C. P62,600
D. P65,000
(Dayag, 2015)

127.Partner A first contributed P50,000 of capital into an existing partnership on March 1, 2015.
On June 1, 2015, the partner contributed another P20,000. Withdrawals in excess of
P10,000 are charged to the partner’s capital account. The annual weighted-average capital balance
is

A. P62,000 B.
P51,667 C. P60,000
D. P48,333

(Dayag, 2015) 128. (1) All assets contributed to the partnership are recorded by the
partner at their agreed values.
(2) All liabilities that the partnership assumes are recorded at their net present values.

A. Only the first statement is correct


B. Only the second statement is correct
C. Both statements are correct
D. Both statements are incorrect
(Dayag, 2015)

129. If a partnership has only non-cash assets, all liabilities have been properly
disbursed, and no additional liquidation expenses are expected, the maximum
potential loss to the partnership in the liquidation process is:

A. The fair market value of the non-cash assets


B. The book value of the non-cash assets
C. The estimated proceeds from the sale of the assets less the book value of the non-cash
assets.
D. None of the above.
(RESA Pre-Board July 2017)

130. In partnership,

A. Management consists of the board of directors


B. Profits are always divided equally among partners
C. Dissolution results when a partner leaves the partnership
D. No partner is liable for more than a proportion of the company’s debt
(RESA Pre-Board April 2016)

131. Mr. MAC is admitted into the partnership of Do and Nald by investing cash
equivalent to ¼ of their capital. Which of the following is true after the
admission of
P a g e | 54

Mr. MAC?

A. Assets of the partnership will increase


B. Total partner’s equity remain the same
C. Do and Nald capital decreased by ¼.
D. Assets of the partnership will remain the same
(RESA Pre-Board July 2017)

132. Under the bonus method, when a new partner is admitted to the partnership, the
total capital of the new partnership is equal to:

A. The book value of the previous partnership plus the fair market value of the
consideration paid to the existing partnership by the incoming partner.
B. The book value of the previous partnership plus any necessary asset write-ups
from book value to market value plus the fair market value of the consideration
paid to the existing partnership by the incoming partner.
C. The book value of the previous partnership minus any asset write downs from
book to market value plus the fair market value of the consideration paid to the
existing partnership by the incoming partner.
D. The fair market value of the new partnership as implied by the value of the
incoming partner’s consideration in exchange for an ownership percentage in
the new partnership.
(RESA Pre-Board July 2017)

133. Which of the following statements is correct regarding a partner’s debit capital
balances?

A. The partner should make contributions to reduce the debit balance to whatever extent
possible.
B. If contributions are not possible, the other partners with credit capital balances will be
allocated a portions of the debit balance based on their proportionate profit-and-
losssharing percentages.
C. Partners who absorb another’s debit capital balance have a legal claim against the
deficient partner.
D. All of these statements are correct.
(RESA Pre-Board July 2017)

134. Following is the balance sheet of the ABCD Partnership at March 31, 2018,
when the partnership is to be liquidated:

Cash P 6, 000 Liabilities P 12, 400


Other Assets 126, 000 A, Loan
12, 000 B, Loan 14, 400
D, Loan 9, 600 A, Capital – 25% 16, 200

B, Capital – 25% 12, 000


C, Capital – 25% 37, 700
D, Capital – 25% 17, 700

During the month of April 2018, assets having a book value of P 18, 000 are sold at a loss
of P 2, 400. Liquidation expenses of P 600 are paid as well as P 7, 200 of the liabilities.
Of the liabilities shown in the balance sheet, P 240 represents salary payable to D and P
160 represents salary payable to C.
P a g e | 55

On April 30, 2018 cash to be distributed to A, B, C and D as follows:

A B C D
A. P 0 P 0 P 0 P 9, 000
B. P 1, 950 P 1, 950 P 1, 950 P 1, 950
C. P 0 P 0 P 0 P 1, 950
D. P 0 P 0 P 9, 000 P 0
(RESA Pre-Board July 2017)

135. Cheryl is the manager of a local store. She is also a partner in the company and she
receives a bonus as part of the profit and loss allocation. Cheryl’s bonus is based on
the increase in revenues recorded during the period. The bonus arrangement is that
Cheryl receives 1 percent of net income for every full percentage point growth for
revenues in excess of a 5 percent revenue growth. During the most recent period,
revenues grew from P500, 000 to P540, 000 and net income grew from P 98, 000 to P
120, 000. How much bonus does Cheryl receive for this period?

A. P 1, 100
B. P 3, 600
C. P 2, 000
D. P 6, 000
(RESA Pre-Board July 2017)

Use the following information for questions 8 and 9:

Cleary, Wasser, and Nolan formed a partnership on January 1, 20x4, with investments of
P 100, 000, P 150, 000, and P 200, 000, respectively. For division of income, they agreed
to (1) interest of 10% of the beginning capital balance each year, (2) annual compensation
of P 10, 000 to Wasser and (3) sharing the remainder of the income or loss in a ratio of
20% for Cleary and 40% each for Wasser and Nolan. Net income was P 150, 000 in 20x4
and P 180, 000 in 20x5. Each partner withdrew P 1, 000 for personal use every month
during 20x4 and 20x5.

136. What was Wasser’s share of income for 20x4?

A. P 63, 000 B. P 53, 000 C. P 58, 000 D. P 29, 000 E. P 51, 000
(RESA Pre-Board April 2016)

137. What was Wasser’s capital balance at the end of 20x5?


A. P 201, 000 B. P 263,
520 C. P 264, 540 D. P
304, 040 E. P 313, 780
(RESA Pre-Board April 2016)

138.
XX, YY, and ZZ, a partnership formed on January 1, 2018 had the following initial
investment:
XX ………………………………………P 170, 000
YY ………………………………………. 255, 000
ZZ ………………………………………. 382, 500

The partnership agreement states that the profits and losses are to be shared equally by the partners
after consideration is made for the following:
- Salaries allowed to partners: P102, 000 for XX, P81, 600 for YY, and P61, 200 for ZZ.
P a g e | 56

- Average partners’ capital balances during the year shall be allowed 10%.

Additional information:
- On June 30, 2018, XX invested an additional P102, 000.
- ZZ withdrew P119, 000 from the partnership on September 30, 2018.
- Share the remaining partnership profit was P 8, 500 for each partner.

The total partnership capital on December 31, 2018 was:


A. P 688, 500
B. P 1, 141, 550
C. P 816, 000
D. d. P 1, 143, 675
(RESA Pre-Board July 2017)

139. At the time of partnership liquidation, which credits shall be settled first?

A. Those amount owing to third persons.


B. Those amount owing to partners other than capital contribution and share in profit.
C. Those amount owing to partners with respect to capital contribution.
D. Those amount owing to partners with respect to share in profit.
(CPAR Reviewer, 2017)
140. How should the net profit or net loss of the partnership be divided among the
partners, whether capitalist or industrial?

A. In accordance with their capital contribution ratio.


B. In accordance with just and equitable sharing taking into account the circumstances of the
partnership.
C. Equally
D. In accordance with the partnership agreement.
(CPAR Reviewer, 2017)
141. At the date of partnership formation of a partnership, the amount credited
to A’s capital is less than the fair value of the property contributed. Which is the
most valid reason?

A. The property contributed by A is impaired.


B. The property contributed by A has been subjected to positive asset revaluation.
C. Bonus has been given by partner A to the other partners.
D. Goodwill arising from partnership formation has been recognized.
(CPAR Reviewer, 2017)

142. When a new partner is admitted to an existing partnership through the


purchase of a portion of existing interest of an incumbent partner, which statement
is correct?

A. The total capital of the old and new partnership will be the same.
B. The partnership will recognize gain or loss on the difference between the amount paid and
capital transferred.
C. Goodwill may be recognized by virtue of the admission.
D. There will be increase in the total assets of the partnership equivalent to the amount paid by
the newly admitted partner.
(CPAR Reviewer, 2017)
143. At the time of retirement, a retiring partner receives more than the amount of his
capital contribution while the remaining partners capital increase after the retirement.
Which of the following is most valid reason?
P a g e | 57

A. Goodwill during retirement is recognized.


B. Asset revaluation is recognized.
C. Bonus is given by retiring partner to remaining partners.
D. Bonus is given by the remaining partners to retiring partner.
(CPAR Reviewer, 2017)

144. On January 1, 2017, Toni, Abbie and JM entered into articles of co-partnership
for the operation of TAJ computer shop. Toni contributed investment property
with assessed value of P1,700,000 subject to mortgage payable of P500,000 to
be assumed by the partnership. Abbie contributed computer equipment with
cost of P600,000 with accumulated depreciation of P200,000. The fair market
value of the computer equipment is P300,000.

On January 2, 2017, the partnership was able to sell the investment property for
P2,000,000. How much cash shall be contributed by JM if the articles of co-partnership
provide that Toni will have 60% interest in the partnership?

A. 500,000 B.
700,000 C.
800,000
D. 600,000

(CPAR Reviewer, 2017)

145. On January 1, 2017, Yazzi, Angel and Nadine organized YAN partnership by
investing P5M, 2M and P3M for capital interest ratio of 4:5:1 respectively.
Nadine has been appointed as managing partner. During year 2017, YAN
partnership reported net income of P3,000,000. Their profit/loss distribution
and drawing agreement are presented below:

• 20% interest on beginning capital


• P10,000, P20,000 and P50,000 monthly salary, respectively
• 25% bonus of net income after interest and salary to managing partner
• The remainder will be divided equally among the partners.
• The partners must withdraw at the end of the year 50% of their share in net income for the
period.

What is the capital balance of Nadine on December 31, 2017?

A.
1,410,000 B.
3,410,000 C.
1,610,000
D. 3,610,000
(CPAR Reviewer, 2017)
146. On December 31, 2017, the capital balance of partners Cristy, Paula and Ara
of CPA Partnership are P1M, P3M and P6M, respectively with profit or loss
agreement ratio of 4:1:5. On January 1, 2018, Cristy decided to retire and
received P400,000 from the partnership.

If the assets of the partnership are not properly valued at the time of retirement, how much
is the capital balance of Paula after the retirement of Cristy?
P a g e | 58

A. 2,9
00,
000
B. 2,8
50,
000
C.
3,1
00,
000
D. 3,150,000
(CPAR Reviewer, 2017) Numbers 147 and 148

On December 31, 2017, the Statement of Financial Position of DEF with profit or loss ratio
of 4:1:5 is presented below:

Cash 2M Liability to third person 4M


Noncash asset 8M D, capital 3.5M
E, capital 1.5M
F, capital 1M

On January 31, 2018, DEF partnership has been subjected to installment liquidation. As of January
31, 2018, the following data concerning liquidation are provided:

 Noncash asset with book value of P6M has been sold at a loss of P2M.
 Liquidation expense amounting to P400,000 has been incurred for the month of January.
 P600,000 cash has been withheld for future liquidation expense.
 P3M liability has been paid.

147. What is F’s share in the maximum possible loss on January 31, 2018?

A. 1,300,000 B.
1,000,000 C.
1,500,000
D. 500,000

148. What is the amount received by E on January 31, 2018?

A. 300,000 B.
700,000 C.
1,000,000
D. 0

149. On December 1, 2011, EE and FF formed a partnership, agreeing to share for


profits and losses in the ratio of 2:3, respectively. EE invested a parcel of land
that cost him P25,000. FF invested P30,000 cash. The land was sold for
P50,000 on the same date, three hours after formation of the partnership.
How much should be the capital balance of EE right after formation?

A. P
2
5
P a g e | 59

,
0
0
0
B. 3
0
,
0
0
0
C
.
2
0
,
0
0
0
D. 50,000

150. MM, NN, and OO are partners with capital balances on December 31 2011 of
P300,000, P300,000 and P200,000, respectively. Profits are shared equally.
OO wishes to withdraw and it is agreed that OO is to take certain equipment
with second-hand value of P50,000 and a note for the balance of OO's interest.
The equipment are carried on the books at P65,000. Brand new equipment may
cost P80,000. Compute for: (1) OO's acquisition of the second-hand equipment
will result to reduction in capital; (2) the value of the note that will OO get
from the partnership's liquidation.

A. (1) P15.000 each for MM and NN,(2) P150,000.


B. (1) P5,000 each for MM, NN and OO,(2) P145,000
C. (1) P5,000 each for MM. NN and OO,(2) P195,000
D. (1) P7,500 each for MM and NN,(2) P145,000.

151. JJ and KK are partners who share profits and losses in the ratio of 60%: 40%
respectively. JJ's salary is P60,000 and P30,000 for KK. The partners are also
paid interest on their averdge capital balances. In 2011, JJ received P30.000 of
interest and KK, P12,000. The profit and loss allocation is determined after
deductions for the salary and interest payments. If KK's share in the residual
income (income after deducting salaries and interest) was P60,000 in 2011,

What was the total partnership income?

A. P192,000
B. 345,000
C. P282,000
D. 387,000
P a g e | 60

152. Lancelot is trying to decide whether to accept a salary of P40.000 or a salary


of P25.000 plus a bonus of 10% of net income after salary and bonus as a
means of allocating profit among the partners. Salaries traceable to the other
partners are estimated to be P100,000. What amount of income would be
necessary so that Lancelot would consider the choices to be equal

A. P165,000
B 290,000
C. P265,000
D. 305,000

153. Merlin, a partner in the Camelot Partnership, has a 30% participation in


partnership profits and losses. Merlin's capital account has a net decrease of
P.200.000 during the calendar year 2011. During 2011, Merlin withdrew
P2.600,000 (charged against his capital account) and contributed property
valued at P500,000 to the partnership. What was the net income of the
Camelot Partnership for year 2011?

A. P3,000,000
B. 4,666,667
C. P 7,000,000
D. 11,000,000

154. The partnership agreement of XX, YY & ZZ


provides for the allocation of net income in the following order:

First, XX is to receive 10% of net income up to P200.000 and 20% over P200,000. Second, YY
and ZZ each are to receive 5% of the remain income over P300,000

The balance of income is to be allocated equally among the three partners. The
partnership's 2011 net income was P500,000 before any
allocations to partners. What amount should be allocated to XX?

A. P202,000
B. 216,000
C. P206,000
D. 220,000

155. On April 30, 2011, XX, YY and ZZ formed a partnership by combining their
separate business proprietorships. XX contributed cash of P75,000. YY
contributed property with a P54,000 carrying amount, a P60,000 original cost,
and P120,000 fair value. The partnership accepted responsibility for the
P52,500 mortgage attached to the property. ZZ contributed equipment with a
P45,000 carrying amount, a P112,500 original cost, and P82,500 fair value.
The partnership agreement specifies that profits and losses are to be shared
equally but is silent regarding capital contributions. Which partner has the
largest April 30, 2011, capital balance?

A. XX
P a g e | 61

B. YY
C. ZZ
D. All capital account balances are equal

156. The Partnership has the following accounting amounts:

(1) Sales P70,000


(2) Cost of Goods Sold P40,000
(3) Operating Expenses P10,000
(4) Salary allocations to partners P13,000
(5) Interest paid to banks P2,000 (6) Partners' withdrawals P8,000 The partnership net income
(loss) is:

A. P20,000
B. 18,000
C. P 5,000

157. The capital accounts of the partnership of NN,


vv, and JJ on lune are presented below with their
respective profif and loss ratios: (P139,200 1/2 , 208,800 1/3 , 96,000 1/6) On
June 1, 2011, LL is admitted to the partnership when LL purchased, for
P132,000, a proportionate interest from NN and JJ
in the net assets and profits of the partnership.
As a result of a transaction LL acquired
a fifth interest in the net assets and profits of the firm. What is the combined
gain realized by NN and JJ upon.the sale of a
portion of their interest in the partnership to LL?

A. P 0
B. 43,200
C P62,400
D. 82,000

158. On January 31, 2011, partners of Lon, Mac & Nan, LLP, had the following
loan and capital account balances (after closing entries for January):

Loan receivable from Lon 20,000 dr


Loan payable to Nan 60,000 cr
Lon, capital 30,000 dr
Mac, capital 120,000 dr
Nan, capital 70.000cr

The partnership's income sharing ratio was Lon, 50%; Mac, 20%, and Nan, 30%. On
January 31, 2011, Ole was admitted to the partnership for a 20% interest in total capital
of the partnership in exchange for an investment of P40,000 cash. Prior to Ole's admission,
the existing partners agreed to increase the carrying amount of the partnership's
inventories to current fair value, a P60,000 increase. The capital account to be credited to
Ole:

A. P60,000
B. P40,000
C. P52,000
D. P46,000
P a g e | 62

159. Which of the following transactions will not affect the total equity of the
partnership?

A. Recognition of impairment loss in case of admission of a new partner B. Withdrawal of


a partner
C. Admission of a new partner by purchase of existing partner’s interest
below its book value
D. Retirement of an existing partner with payment of above the book value of
such interest
(CPAR Final Pre-board Examination May 2017)

160. A, B, and C are partners with average capital balances during 2017 of P472,500,
P238,650 and P162,350; respectively. The partners receive 10% interest on their
average capital balances; after deducting salaries of P122,325 to A and P82,625 to
C, the residual profit or loss is divided equally.

In 2017, the partnership had net loss of P125,624 before interest and salaries to partners. What
amount should A and C capital change respectively?

A. P40,844 decrease and P31,237 decrease


B. P30,267 increase and P40,448 decrease
C. P29,476 increase and P17,536 increase
D. P28,358 increase and P32,458 increase
(CPAR Final Pre-board Examination May 2017)

161. A partner was admitted in an existing partnership through investment of cash


equivalent to ¼ of the new capitalization. If the capital balance of the old partners
increases, what is the most valid reason under Philippine GAAP?

A. Asset revaluation of existing partnership’s assets


B. Impairment loss of existing partnership’s assets
C. Recognition of goodwill of existing partnership
D. Receipt of bonus from the new partner
(CPAR Final Pre-board Examination May 2017)

162. Which of the following statements concerning the formation of partnership


business is correct?

A. PFRS allows recognition of goodwill arising from the formation of partnership


B. The juridical personality of the partnership arises from the issuance of certification of
registration
C. The parties may become partners only upon contribution of money or property but not
of industry or service
D. The capital to be credited to each partner upon formation may not be the amount actually
contributed by each partner
(CPAR Final Pre-board Examination May 2017)

163. Regina, Jessica and Nataly entered into a contract of partnership with a total capital
contribution of P5,000. The parties failed to register its articles of co-partnership
with the Securities and Exchange Commission. Which of the following statements
is correct?
P a g e | 63

A. The contract of partnership is void because the law provides that when the capital
contribution is at least P3,000 it must be registered with Securities and Exchange
Commission
B. The contract of partnership will bind third persons
C. The contract of partnership remains to be valid
D. The partnership does not obtain juridical personality for failure to register with
Securities and Exchange Commission.
(CPAR Final Pre-board Examination May 2017)
164. In the absence of agreement as to distribution of profit, how shall the partnership
profit be distributed to the partners?

A. The industrial partner shall receive a share equivalent to the least share of
a capitalist partner while the capitalist partners shall share based on capital
contribution ratio.
B. The industrial partner shall receive a just and equitable share and the
remainder shall be distributed to the capitalist partners on the basis of
capital contribution ratio
C. The profit shall be distributed on the basis of loss contribution ratio which
may have been agreed upon by the partners
D. The profit shall be distributed equally to all partners including the
industrial partner

(CPAR Final Pre-board Examination May 2017)

165. On July 1, 2016, Anne, Bianca and Carla formed a business partnership to be
operated as an advertising agency. Anne contributed P10M cash while Bianca shall
have a capital credit of P6M upon receipt of bonus of P1M from Anne based on
the provision in Articles of CoPartnership. The terms of the agreement provide
that Anne and Bianca shall have a combined 40% capital interest in the newly
formed partnership. What is the capital contribution made by Carla to the
partnership?

A. P24,000,000
B. P22,500,000
C. P25,000,000
D. P32,000,000
(CPAR Final Pre-board Examination May 2017)

166. On January 1, 2017, Angel, Bea and Colleen formed ABC & Co., a general
professional partnership for the exercise of their common profession. Angel
contributed a building with a cost of P5M and accumulated depreciation of P4M.
Based on the city assessor’s records, the building has an assessed value of P2M.
The building has an annotated mortgage payable amount to P500,000 to be
assumed by the partnership.
On the other hand, Bea contributed 10,000 shares of stocks with par value of P200/share
and prevailing quoted price of P300/share. On January 2, 2017, the building contributed
by Angel was sold for P5.5M. If Colleen wants to have 20% capital interest in the newly
formed partnership, how much cash shall be contributed by her?

A. P875,000
B. P1,125,000 C. P2,125,000
D. P2,000,000
(CPAR Final Pre-board Examination May 2017)
P a g e | 64

167. On January 1, 2014, AB and QR agreed to form a partnership. The following are
their assets and liabilities:
Accounts AB QR
Cash 136,000 76,000
Accounts Receivable 88,000 48,000
Inventories 304,000 364,000
Machinery 480,000 440,000
Accounts Payable 216,000 144,000
Notes Payable 140,000 60,000

AB decided to pay off his notes payable from his personal assets. It was also agreed that
QR inventories were overstated by P24,000 and AB machinery was over depreciated by
P20,000. QR is to invest/withdraw cash in order to receive a capital credit that is 20%
more than AB’s total net investment in the partnership.

How much cash will be presented in the partnership’s statement of financial position?

A. 486,400
B. 546,300
C. 250,400
D. 640,300
(CPAR Final Pre-board Examination May 2017)

168. On December 1, 2014, MG and AN are combining their separate businesses to


form a partnership. Cash and noncash assets are to be contributed. The noncash
assets to be contributed and the liabilities to be assumed are as follows:

MG AN
Book value Fair value Book value Fair value
Accounts Receivable 250,000 262,500 200,000 195,000
Inventory 400,000 450,000 200,000 207,500
PPE 1,000,000 912,500 862,500 822,500
Accounts Payable 150,000 150,000 112,500 112,500

MG and AN are to invest equal amount of cash such that the contribution of MG would be
10% more than the investment of AN. What is the amount of cash presented on the
partnership’s statement of Financial Position on December 1, 2014?

A. 5,025,000 B. 5,500,000 C. 5,750,000


D. 4,950,000
(CPAR Final Pre-board Examination May 2017)

169. After the admission of a new partner, the total partnership capital increased by the
fair value of the new partner’s net contributions to the partnership. The admission
was accounted
for
A. Under the goodwill method
B. Under the bonus method
C. As a purchase of interest
D. As an investment in the partnership
(Millan, 2016)
P a g e | 65

170. If a new partner acquires a partnership interest directly from the partners rather
than from the partnership itself,
A. No entry is required.
B. The partnership should be revalued.
C. The existing partners’ capital accounts should be reduced and the new partner’s account
increased.
D. The partnership has undergone a quasi-reorganization.
(Punzalan, 2015)

171. Which of the following is not a characteristic of a partnership?


A. Limited liability
B. Limited life
C. Mutual agency
D. Ease of formation
(Punzalan, 2015)

172. Abel and Carr formed a partnership and agreed to divide initial capital equally,
even though Abel contributed P100,000 and Carr contributed P84,000 in
identifiable assets. Under the bonus approach to adjust the capital accounts, Carr’s
unidentifiable assets should be debited for
A. 46,000
B. 16,000
C. 8,000
D. 0
(Punzalan, 2015)

173. Alder, Benson and Carl are capitalist partners and Denver, an industrial partner.
The partnership reported a net loss of P100, 000. How much is the share of Denver
in the reported net loss?
A. 0
B. 10,000
C. 25,000
D. 100,000
(Punzalan, 2015)

174. XYZ Partnership provided for the following in their distribution of profits and
losses: First: X to receive 10 % of net income up to P100,000 and 20% of the
amount in excess thereof.
Then: Y and Z are each to receive 5% of the remaining income in excess of P150,000 after X’s
share.
Finally: The balance is to be distributed equally to the three partners.

If the partnership earned a net income of P250,000, what is the total share of Partner X?

A. 100,000 B. 108,000
C. 110,000
D. 130,000
(Punzalan, 2015)

175. After incurring losses resulting from very unprofitable operations, the Goh Kong
Wei
Partnership decided to liquidate when the partners’ capital balances were:

Goh, Capital (40%) P80,000


P a g e | 66

Kong, Capital (40%) 130,000


Wei, Capital (20%) 96,000

The noncash assets were sold in installment. Available cash were distributed to partners
in every sale of noncash assets. After the second sale of noncash assets, the partners
received the same amount of cash in the distribution. And from the third sale of noncash
assets, cash available for distribution amounts to P28,000, and unsold noncash assets has
a book value of P12,500. Using cash priority program, what amount did Wei receive in
the third installment of cash?

A. 11,600
B. 8,000
C. 5,600
D. 0
(Punzalan, 2015)

176. The condensed balance sheet of Adams & Gray, a partnership, at December 31,
2014, follows:

Current assets P250,000


Equipment (net) 30,000
Total assets P280,000

Liabilities P20,000
Adams, Capital 160,000
Gray, Capital 100,000
Total liabilities and capital P280,000

On December 31, 2014, the fair values of the assets and liabilities were appraised at
P240,000 and P20,000, respectively, by an independent appraiser. On January 2, 2015,
the partnership was incorporated and 1,000 shares of P5 par value common stock were
issued. Immediately after the incorporation, what amount should the new corporation
report as additional paid in capital?
A. 270,000 B.
260,000
C. 215,000
D. 0
(Punzalan, 2015)

177. Partner Morgan is personally insolvent, owing P600,000. Personal assets will only bring
P200,000 when liquidated. At the same time, Morgan has a credit balance of P120,000. The
capital amounts of the other partners total a balance of P250,000. Under the doctrine of
marshalling of assets, how much the personal creditors of Morgan can collect?
A. 120,000 B.
200,000 C.
320,000 D.
570,000
(Punzalan, 2015)

178. The partnership agreement of Reid and Simm provides that 10% per year is to be credited to
each partner on the basis of weighted-average capital balances. A summary of Simm’s
capital account for the year-ended December 31, 2014, is as follows:

Balance, January 1 P140, 000


P a g e | 67

Additional Investment, July 1 40, 000


Withdrawal, August 1 (15, 000)
Balance, December 31 165, 000

What amount of interest should be credited on Simm’s capital account for 2014?
A. 15,250 B.
15,375 C.
16,500 D.
17,250
(Punzalan, 2015)

179. The fact that salaries paid to partners are not a component of partnership income is indicative
of
A. A departure from generally accepted accounting principles
B. Being characteristic of the entity theory
C. Being characteristic of the proprietary theory
D. Why partnerships are characterized by unlimited liability
(Punzalan, 2016)

180. The doctrine of marshalling of assets

A. Is applicable only if the partnership is insolvent


B. Allows partners to first contribute personal assets to unsatisfied partnership
creditors C. Is applicable if either the partnership is insolvent or individual
partners are insolvent
D. Amount owed to personal creditors and to partnership for debit capital balances are shared
proportionately from the personal assets of the partners
(Punzalan, 2016)

181. If goodwill is traceable to the incoming partner, the new partner's capital balance equals

A. the fair market value of consideration paid by the incoming partner


B. the book value of the older partnership divided by the existing partners' ownership
percentage in the new partnership minus the book value of the old partnership.
C. incoming partner's ownership percentage multiplied by the capital of the new partnership
D. none of the above.

(Guerrero, 2014)

AY and AN are partners who have the agreement to share profit and loss in the following manner:

AY AN
Annual salaries 261,000 259,000
Interest on average balances 5% 10%
Bonus (based on net income after salaries and interest) 10%
Remainder 50% 50%

During the year ended December 31, 2014, the partnership generated a profit of P575,000
before any deductions. AY’s and AN’s average capital balances for the year are P600,000
and P300,000, respectively. Income is distributed to the partners only as far as it is
available.

182. How much is the total share of AN in the net income for the year ended 2014?
P a g e | 68

A. P286,500 C. P288,500
B. P287,500 D. P295,665
(RESA, 2014)

On January 1, 2014, L, M, and N formed a partnership with capital contributions of


P625,000; P750,000; and P937,500, respectively. The partners agreed that profit and loss
would be allocated as follows: P75,000 salary to each partner, 3% interest on initial capital
contributions, the remainder divided in the ratio 2:4:4, respectively to L, M, and N. The
partnership generated income amounting to P375,000 for the year 2014. During 2014, the
following partnership errors were discovered before the distribution of profit:

• In 2014, a purchase of piece of equipment costing P50,000 was expensed. The equipment
has an estimated life of ten years with equal service potential each year.
• On December 31, 2014, ending inventory was understated by P50,000.
On January 1, 2015, N decided to retire from the partnership.

183. If the balance of the capital of L after retirement amounts to P770,000, how much is the
settlement to N for his retirement?

A. P1,120,000 C. P1,085,000
B. P1,062,500 D. P1,110,875
(RESA, 2014)

184. If the balance of the capital of M after retirement amounts to P890,000, how much is the
settlement to N for his retirement?

A. P1,127,500
B. P1,090,500
C. P1,231,500
D. P1,152,500
(RESA, 2014)

On December 1, 2014, MG and AN are combining their separate businesses to form a


partnership. Cash and noncash assets are to be contributed. The noncash assets to be
contributed and the liabilities to be assumed are as follows:
MG AN
Book value Fair value Book value Fair value
Accounts Receivable 250,000 262,500 200,000 195,000
Inventory 400,000 450,000 200,000 207,500
PPE 1,000,000 912,500 862,500 822,500
Accounts Payable 150,000 150,000 112,500 112,500

MG and AN are to invest equal amount of cash such that the contribution of MG would be 10%
more than the investment of AN.

185. What is the amount of cash presented on the partnership’s statement of Financial Position
on December 1, 2014?
(RESA,2014)
A. P2,762,500
B. P2,512,500
C. P5,525,000
D. P5,025,000
P a g e | 69

On December 1, 2014, MV and CD agreed to invest equal amounts and share profits
equally to form a partnership. MV invested P3,120,000 cash and a piece of equipment.
CD invested some assets which are shown on the next page:

Book value

Accounts Receivable 400,000

Inventory 1,120,000

Machineries, net 2,240,000

Intangibles, net 920,000

The assets invested by CD are not properly valued, P32,000 of the accounts receivable are
proven uncollectible. Inventories are to be written down to P1,040,000. Included in the
machineries is an obsolete apparatus acquired for P384,000 with an accumulated
depreciation balance of P336,000. Part of the intangibles is a patent with a carrying value
of P56,000 which was sued upon by a competitor. CD unsuccessfully defended the case
and the final decision of the court was released on November 29, 2014.

186. What is the fair value of the equipment invested by MV?

A. P1,400,000 C. P1,344,000

B. P968,000 D. P1,560,000

(RESA, 2014)

The partnership of CD, AY, and GP decided to liquidate their partnership on May 31,
2013. Before liquidating and sharing of net income, their capital balances are as follows:
CD (30%) P875,000, AY (30%) P630,000, and GP (40%) P770,000. Net income from
January 1 to May 31 is P420,000. Liabilities of the partnership amounted to P735,000 and
its total assets include cash amounting to P245,000. Unsettled liabilities are P385,000. CD
invested additional cash enough to settle their partnership’s indebtedness. AY is
personally solvent, GP is personally insolvent, and CD becomes insolvent after investing
the cash needed by the partnership.

187. How much were the partnership’s non-cash sold for?


A. P157,500 C. P105,000
B. P3,080,000 D. P525,000

188. How much will CD receive as a result of their liquidation?


A. P385,000
B. 0
C. P315,000
D. P462,000

(RESA, 2014)

189. On April 30, 2016, Al, Ben, and Ces formed a partnership by combining their separate
business proprietorships. Al contributed cash of P50,000. Ben contributed property with a
P36,000 carrying amount, a P40,000 original cost, and P80,000 fair value. The partnership
P a g e | 70

accepted responsibility for the P35,000 mortgage attached to the property. Ces contributed
equipment with a P30,000 carrying amount, a P75,000 original cost, and P55,000 fair value.
The partnership agreement specifies that profits and losses are to be shared equally but is
silent regarding capital contributions. Which partner has the largest capital account balance
at April 30, 2016?
A. Al
B. Ben
C. Ces
D. All capital balances are equal
(Punzalan, 2015)

190. A partnership records a partner’s investment of assets in the business at A. The market
value of the assets invested.
B. A special value set by the partners.
C. The partner’s book value of the assets invested.
D. Any of the above, depending upon the partnership agreement.

(RPCPA 0598) 191. In the Adel-Brick partnership, Adel and Brick had a capital ratio of
3:1 and a profit and loss ratio of 2:1, respectively. The bonus method was used to record
Colter’s admittance as a new partner. What ratio would be used to allocate, to Adel and
Brick, the excess of Colter’s contribution over the amount credited to Colter’s capital
account?

A. Adel and Brick’s new relative capital ratio.


B. Adel and Brick’s new relative profit and loss ratio.
C. Adel and Brick’s old capital ratio.
D. Adel and Brick’s old profit and loss ratio.
(AICPA 0r92 T-35)
192. The final cash distribution to the partners in a partnership in liquidation should be
made in accordance with

A. Balances of the partners’ capital accounts.


B. Partners’ profit and loss sharing ratio.
C. Ratio of capital contributions made by the partners.
D. Ratio of capital contributions less withdrawals made by the partners.
(RPCPA 1081,0586)
193. K, L, and M are partners with average capital balance during 2011 of P472,500,
P238,650, and P162,350, respectively. The partners receive 10% interest on their
average capital balances; after deducting salaries of P122,325 to K and P82,625 to M,
the residual profits or loss is divided equally.

In 2011, the partnership had a net loss of P125,624 before the interest and salaries to partners.

By what amount should K’s and M’s capital account change?


K’s Capital Account M’s Capital Account
A. P40,844 decrease P31,235 decrease
B. P28,358 increase P32,458 increase
C. P29,476 increase P17,536 increase
D. P30,267 increase P40,448 decrease
(Guerrero, 2013)
P a g e | 71

194. Prior to partnership liquidation, a schedule of possible losses is frequently prepared


to determine the amount of cash that may be safely distributed to the partners. The
schedule of possible losses

A. Consists of each partner’s capital account plus loan balance, divided by that partner’s
profit-and-loss sharing ratio.
B. Shows the successive losses necessary to eliminate the capital accounts of partners
(assuming no contribution of personal assets by partners).
C. Indicates the distribution of successive amounts of available cash to each partner.
D. Assumes contribution of personal assets by partners unless there is a substantial
presumption of personal insolvency by the partners.
(Gleim)
195. The following condensed balance sheet is presented for the partnership of Axel,
Barr, and Cain, who share profits and losses in the ratio of 4:3:3, respectively:
Cash P100,000
Other assets 300,000
Total 400,000

Liabilities P150,000
Axel, Capital 40,000
Barr, Capital 180,000
Cain, Capital 30,000
Total 400,000

The partners agreed to dissolve the partnership after selling the other asset for
P200,000. Upon dissolution of the partnership, Axel should have received

A. 0
B. 40,000 C. 60,000
D. 70,000
(Punzalan, 2015)
196. The following blance sheet is presented for the partnership of A, B, and C, who
share profits and losses in the respectively ratio of 5:3:2.
Assets Liabilities and Capital
Cash P 120,000 Liabilities P280,000
Other Assets 1,080,000 A, Capital 560,000
B, Capital 320,000
C, Capital 40,000
Total P1,200,000 Total P1,200,000

Assume that the three partners decided to liquidate the partnership. If the other assets are sold
for P800,000, how should the available cash be distributed to each partner?
A B C
A. 280,000 320,000 40,000
B. 324,000 236,000 16,000
C. 410,000 230,000 0
D. 412,000 228,000 0
(Punzalan, 2015)

197. Red, White, and Blue form a partnership on May 1, 2011. They agree that Red will
contribute office equipment with a toal fair value of P40,000; White will contribute
delivery equipment with a fair value of P80,000; and Blue will contribute cash. If Blue
wants a on third interest in the capital and profits, he should contribute cash of:
A. P40,000
P a g e | 72

B. P120,000
C. P60,000
D. P180,000
(Guerrero, 2013)

198. AK and BK decided to form a partnership on October 1, 2014. Their Statement of


Financial Position on this date were:
AK Bk
Cash 65,625.00 164,062.50
Accounts Receivable 1,487,500.00 896,875.00
Merchandise Inventory 875,000.00 885,937.50
Equipment 656,250.00 1,268,750.00
Total 3,084,375.00 3,215,625.00

Accounts Payable 459,375.00 1,159,375.00


AK, Capital 2,625,000.00
BK, Capital 2,056,250.00
Total 3,084,375.00 3,215,625.00

They agreed the following adjustments shall be made:

• Equipment of AK is underdepreciated by P87,500 and that BK is overdepreciated by


P131,250.
• Allowance for doubtful accounts is to be set up amounting to P297,500 for AK and
P196,875 for BK.
• Inventories of P21,875 and P15,312.50 are worthless in the books of AK and BK
respectively.
• The partnership agreement provides for a profit and loss ratio of 70% to AK and 30%
to BK. Assuming the use of transfer of capital method, how much is the agreed capital
of AK to bring the capital balances proportionate to their profit and loss ratio.
On January 1, 2014, AB and QR agreed to form a partnership. The following are their assets and
liabilities:
Accounts AB QR
Cash 136,000 76,000
Accounts Receivable 88,000 48,000
Inventories 304,000 364,000
Machinery 480,000 440,000
Accounts Payable 216,000 144,000
Notes Payable 140,000 60,000

AB decided to pay off his notes payable from his personal assets. It was also agreed that
QR inventories were overstated by P24,000 and AB machinery was over depreciated by
P20,000. QR is to invest/withdraw cash in order to receive a capital credit that is 20%
more than AB’s total net investment in the partnership.

How much cash will be presented in the partnership’s statement of financial position?
A.
2,935,406.25 B.
2,218,125.00 C.
1,975,312.50
D. 1,258,031.25
P a g e | 73

CPAR Pre-Boards October 2017

199. On December 1, 2014, MV and CD agreed to invest equal amounts and share profits
equally to form a partnership. MV invested P3,120,000 cash and a piece of
equipment. CD invested some assets which are shown on the next page:

Book value
Accounts Receivable 400,000
Inventory 1,120,000
Machineries, net 2,240,000
Intangibles, net 920,000

The assets invested by CD are not properly valued, P32,000 of the accounts receivable
are proven uncollectible. Inventories are to be written down to P1,040,000. Included in
the machineries is an obsolete apparatus acquired for P384,000 with an accumulated
depreciation balance of P336,000. Part of the intangibles is a patent with a carrying
value of P56,000 which was sued upon by a competitor. CD unsuccessfully defended
the case and the final decision of the court was released on November 29, 2014.
A. 274,000 B.
212,000 C.
486,000
D. 374,000

CPAR Pre-Boards October 2017

200. On December 1, 2014, MG and AN are combining their separate businesses to form
a partnership. Cash and noncash assets are to be contributed. The noncash assets to
be contributed and the liabilities to be assumed are as follows:

MG AN
Book value Fair value Book value Fair value
Accounts Receivable 250,000 262,500 200,000 195,000
Inventory 400,000 450,000 200,000 207,500
PPE 1,000,000 912,500 862,500 822,500
Accounts Payable 150,000 150,000 112,500 112,500

MG and AN are to invest equal amount of cash such that the contribution of MG would be
10% more than the investment of AN.
What is the amount of cash presented on the partnership’s statement of Financial Position on
December 1, 2014?

A.
1,344,000 B.
1,244,000 C.
3,120,000
D. 2,180,000

CPAR Pre-Boards October 2017


201. CC Partnership began operations on June 1, 2014. On that date, CY and CR have
capital credits of P175,000 and P240,000, respectively. The partnership has the
following profitsharing plan:
P a g e | 74

a.) 10% interest on partners’ capital balances at the end of the year
b.) P60,000 and P75,000 annual salaries for CY and CR, respectively.
c.) Remaining profit will be divided to CY and CR on a 3:2 ratio, respectively.

During the year, CY invested P150,000 worth of merchandise and withdrew P40,000
cash, while CR invested P120,000 cash. The partnership earned a profit of P266,375
during the year.

How much is CY’s capital balance at the end of 2014?


A. 5,025,000 B.
2,512,000 C.
3,215,000 D.
1,223,750
CPAR Pre-Boards October 2017
202. CC Partnership began operations on June 1, 2014. On that date, CY and CR have
capital credits of P175,000 and P240,000, respectively. The partnership has the
following profitsharing plan:

a) 10% interest on partners’ capital balances at the end of


the year
b) P60,000 and P75,000 annual salaries for CY and CR,
respectively.
c) Remaining profit will be divided to CY and CR on a
3:2 ratio,
respectively.

During the year, CY invested P150,000 worth of merchandise and withdrew P40,000 cash,
while CR invested P120,000 cash. The partnership earned a profit of P266,375 during the
year.

How much is CY’s capital balance at the end of 2014?


A. 266,375 B.
426,625 C.
285,000
D. 150,000
CPAR Pre-Boards October 2017

203. Cherryhill and Hace had been partners for several years, and they decided to admit
Quincy to the partnership. The accountant for the partnership believed that the
dissolved partnership and the newly formed partnership were two separate entities.
What method would the accountant have used for recording the admission of Quincy
to the partnership?

A) the bonus method.


B) the equity method.
C) the goodwill method.
D) the proportionate method.
E) the cost method.
CPAR Pre-Boards October 2017

204. When the hybrid method is used to record the withdrawal of a partner, the
partnership
P a g e | 75

A) revalues assets and liabilities and records goodwill to the continuing partner but not to the
withdrawing partner.
B) revalues liabilities but not assets, and no goodwill is recorded.
C) can recognize goodwill but does not revalue assets and liabilities.
D) revalues assets but not liabilities, and records goodwill to the continuing partner but not to the
withdrawing partner.
E) revalues assets and liabilities but does not record goodwill.
CPAR Pre-Boards October 2017
205. The disadvantages of the partnership form of business organization, compared to
corporations, include
A) the legal requirements for formation.
B) unlimited liability for the partners.
C) the requirement for the partnership to pay income taxes.
D) the extent of governmental regulation.
E) the complexity of operations.
CPAR Pre-Boards October 2017
206. The advantages of the partnership form of business organization, compared to
corporations, include

A) single taxation.
B) ease of raising capital.
C) mutual agency.
D) Limited liability.
E) difficulty of formation.
CPAR Pre-Boards October 2017
207. The dissolution of a partnership occurs

A) only when the partnership sells its assets and permanently closes its books.
B) only when a partner leaves the partnership.
C) at the end of each year, when income is allocated to the partners.
D) only when a new partner is admitted to the partnership.
E) when there is any change in the individuals who make up the partnership.
CPAR Pre-Boards October 2017

208. If a partner’s capital balance is credited for an amount greater than or less than the
fair value of his net contribution, the excess or deficiency is called a

A. Bonus
B. Goodwill
C. Discount
D. Premium
(Millan, 2016) 209. If the partnership agreement does not specify how income is to be
allocated, profits and loss should be allocated

A. Equally
B. In proportion to the weighted average of capital invested during the period
C. Equitably so that partners are compensated for the time and effort expended on behalf
of the partnership
P a g e | 76

D. In accordance with their capital contributions


(Millan, 2016) 210. When property other than cash is invested in a partnership, at what
amount should the noncash property be credited to the contributing partner’s capital
account?

A. Fair value at the date of contribution


B. Contributing partner’s original cost
C. Assessed valuation for property tax purposes
D. Contributing partner’s tax basis
(Millan, 2016)
211. The admission of a new partner effected through purchase of interest in the partnership
is

A. Recorded in the partnership books as a debit to cash or other asset and


credit to the incoming partner’s capital account

B. Recorded in the partnership books as a transfer within equity


C. Recorded in the partnership books as a transfer from equity to liability
D. Not recorded in its entirety

(Millan, 2016)

212. State the proper order of liquidation


I. Outside creditors
II. Owners’ interests
III. Inside creditors
A. I, III, II
B. I, II, III
C. III, II, I
D. II, I, III

(Millan, 2016)

213. Lancelot is trying to decide whether to accept a salary of P40,000 or a salary of P25,000
plus a bonus of 10% of net income after salary and bonus as a means of allocating
profit among
the partners. Salaries traceable to the other partners are estimated to be P100,000. What
amount of income would be necessary so that Lancelot would consider the choices to be
equal?

A. P 165,000
B. P 290,000
C. P 265,000
P a g e | 77

D. P 305,000
(Dayag, 2015)
214. MM, NN, OO are partners with capital balances on December 31, 2015 of P 300,000,
P 300,000 and P 200,000, respectively. Profits are shared equally. OO wishes to
withdraw and it is agreed that OO is to take certain equipment with second-hand value
of P 50,000 and a note for the balance of OO’s interest. The equipment are carried on
the books at P65,000. Brand new equipment may cost P 80,000. Compute for: (1) OO’s
acquisition of the second-hand equipment that will result to reduction in capital; (2)
the value of the note that will OO get from the partnership’s liquidation.
A. (1) P15,000 each for MM and NN, (2) P150,000
B. (1) P5,000 each for MM, NN and OO, (2) P145,000
C. (1) 5,000 each for MM, NN and OO, (2) P195,000
D. (1) P7,500 each for MM and NN, (2) P145,000
(Dayag, 2015) 215. RR and XX formed a partnership and agreed to divide initial capital
equally, even though RR contributed P25,000 and XX contributed P21,000 in identifiable
assets. Under the bonus approach to adjust the capital accounts. XX’s unidentifiable assets
should be debited for:

A. P 11,500
B. P 4,000
C. P 2,000
D. P 0

(Dayag, 2015)

216. A. Smith, a partner in an accounting firm, decided to withdraw from the


partnership, Smith’s share of the partnership profits and losses was 20%. Upon
withdrawing from the partnership he was paid P88,800 in final settlement for his
interest. The total of the partner’s capital accounts before recognition of
partnership goodwill prior to Smith’s withdrawal was P252,000. After his
withdrawal the remaining partners’ capital accounts, excluding their share of
goodwill, totaled P192,000. The total goodwill of the firm was:

A. P 144,000
B. P 168,000
C. P 192,000
D. P 300,000

(Dayag, 2015)

217. The following condensed balance sheet is presented for the partnership of AA, BB,
and CC, who share profits and losses in the ratio of 4:3:3, respectively:

Cash P 160,000
Other assets 320,000
Total P 480,000
P a g e | 78

Liabilities P 180,000
AA, Capital 48,000
BB, Capital 216,000
CC, Capital 36,000
Total P 480,000
The partners agreed to dissolve the partnership after selling the other assets for P200,000.
Upon dissolution of the partnership, AA should have received

A. P 0
B. P 48,000
C. P 72,000
D. P 84,000

(Dayag, 2015)

218. When property other than cash is invested in a partnership, at what amount should
the noncash property be credited to the contributing partner’s capital account?

A. Fair Value at the date of recognition


B. Contributing partner’s original cost
C. Assessed valuation for property tax purposes
D. Contributing partner’s tax basis
(AICPA 0594 F-35)

219. A partnership records a partner’s investment of assets in the business at

A. The market value of the assets invested


B. A special value set by the partners
C. The partner’s book value of the assets invested
D. Any of the above, depending upon the partnership agreement
(RPCPA 0598)

220. In a partnership liquidation, the final cash distribution to the partners should be
made in accordance with the

A. Partners’ profit and loss sharing ratio


B. Balances of the partners’ capital accounts
C. Ratio of capital contributions made by the partners
D. Ratio of capital contributions less withdrawals made by the partners
(RPCPA 1079)
P a g e | 79

221. As a result of the retirement of a partner in an existing partnership, the capital


balance of the remaining partners increases. If the assets of the partnership before
retirement are properly valued, which of the following statements is true?

A. The retiring partner received less than his capital balance before retirement
B. There is partnership net loss prior to the retirement of the said partner
C. The remaining partner gives bonus to the retiring partner
D. There is impairment of existing assets recognized prior to retirement
(CPAR PREBOARD WEEK, 2017)

222. If the partnership agreement does not specify how income is to be allocated, profits
should be allocated

A. Equally
B. In proportion to the weighted-average of capital invested during the period
C. Equitably so that partners are compensated for the time and effort expended on
behalf of the partnership
D. In accordance with an established ratio
(Gleim)

223. On June 30, 2016, a partnership was formed by Mendoza and Lopez. Mendoza
contributed cash. Lopez, previously a sole proprietor contributed non-cash assets
including a realty subject to a mortgage which was assumed by the partnership.
Lopez’s capital account at June 30,2016 should be recorded at

A. The fair value of the property on June 30, 2016


B. Lopez’s carrying amount of the property on June 30, 2016
C. The fair value of the property on June 30, 2016 less the mortgage payable
D. Lopez’s carrying amount of the property on June 30, 2016 less the mortgage payable
(BAYSA & LUPISAN, 2016)

For numbers 224 to 225 refer to the problem below:

Diaz and Esteban entered into a partnership on February 1, 2016 by investing the
following assets:

Diaz Esteban

Cash P 15,000

Merchandise Inventory P 45,000

Land 15,000

Building 65,000

Furniture and Fixtures 100,000


P a g e | 80

The agreement between Diaz and Esteban provides that profits and losses are to be
divided into 40% and 60% to Diaz and Esteban respectively. The partnership is to assume
the P30,000 mortgage loan on the building.

224. If Esteban is to receive a capital credit equal to his profit and loss ratio, how
much cash must he invest?

A. 77,500
B. 97,500
C. 127500
D. 172,500
(BAYSA & LUPISAN, 2016)

225. Assuming Esteban invests P50,000 cash and each partner is to be credited for
the full amount of the net assets invested, the total capital of the partnership
is

A. 210,000
B. 250,000
C. 260,000
D. 290,000
(BAYSA & LUPISAN, 2016)

226. Assuming the partnership agreement provides that the partners should
initially have an equal interest in the partnership capital, what is Esteban’s
capital upon partnership formation?
A. 95,000
B. 105,000
C. 115,000
D. 125,000
(BAYSA & LUPISAN, 2016)

227. Canlas, a partner in the 3C Partnership, has a 30% participation in partnership


profits and lossess. Canlas’ capital account had a net decrease of P120,000
during the calendar year 2016. During 2016, Canlas withdrew P260,000
(charged against his capital account) and contributed property valued at
P50,000 to the partnership. What was the profit of 3C Partnership?

A. 300,000
B. 466,667
C. 700,000
D. 1,100,000
(BAYSA & LUPISAN, 2016)
P a g e | 81

228. Profit is the difference between


A. assets and liabilities
B. the incoming cash and outgoing cash
C. the assets purchased with cash contributed by the owner and the cash spent to operate
the business

D. the assets received for goods and services and the amounts used to provide the goods
and services

(Warren 9th ed.)

229. Which of the items below is not a business organization form?

A. Entrepreneurship C. Partnership
B. Proprietorship D. Corporation
(Warren 9th ed.)

230. An entity that is organized in which ownership is divided into shares of stock
is a

A. Proprietorship C. Partnership
B. Corporation D. Governmental Unit

(Warren 9th ed.)

231. Financial reports are used by

A. Management C. Inventors
B. Creditors D. All are correct
(Warren 9th ed.)

232. Which of the following is not a characteristic of a corporation? A.


Corporations are organized as a separate legal taxable entity

B. Ownership is divided into shares of stock.


C. Corporations experience an ease in obtaining large amounts of resources by issuing stock.

D. A corporation’s resources are limited to their individual owners’ resources.

(Warren 9th ed.)

233. He refers to a partner who contributed not only money and property but also
industry to the newly formed partnership.

A. industrial partner
B. nominal partner
C. capitalist-industrial partner
D. capitalist partner
P a g e | 82

(CPAR handout, 2018)

234. It refers to a type of partnership wherein all partners are liable to the creditors
prorata up to the extent of personal or separate assets after the partnership’s
asset are exhausted.

A. General partnership
B. Partnership by estoppel
C. Limited partnership
D. Particular partnership
(CPAR handout, 2018) 235. Which of the following will decrease the capital balance of a
partner?

A. Share in partnership profit


B. Receipt of share in revaluation surplus from a partnership property, plant and equipment

C. Drawing made by partner


D. Advances made by a partner to the partnership
(CPAR handout, 2018)
A, B and C decided to form ABC Partnership. It was agreed that A will contribute an
equipment with assessed value of P100,000 with historical cost of P800,000 and
accumulated depreciation of P600,000. A day after the partnership formation, the
equipment was sold for P300,000.

B will contribute a land and building with carrying amount of P1,200,000 and fair value
of P1,500,000. The land and building are subject to a mortgage payable amounting to
P300,000 to be assumed by the partnership. The partners agreed that B will have 60%
capital interest in the partnership. The partners also agreed that C will contribute sufficient
cash to the partnership.

236. What is the total agreed capitalization of ABC Partnership?


A. 1,500,000
B. 2,000,000
C. 2,500,000
D. 3,000,000

237. What is the cash to be contributed by C in the ABC Partnership?


A. 500,000
B. 600,000
C. 700,000
D. 800,000
P a g e | 83

(CPAR handout, 2018)

238. When property other than cash is invested in a partnership, at what amount should the
noncash property be credited to the contributing partner’s capital account?

A. Contributing partner’s tax basis


B. Contributing partner’s original cost
C. Fair value at the date of contribution
D. Assessed valuation for property tax purposes
(Wiley 2014)
239. In a limited partnership, a general partner
A. Is excluded from management
B. Is not entitled to a bonus at the end of the year
C. Has limited liability for partnership debt
D. Has unlimited liability for partnership debt
(BCA & L 10e)
240. Partnership drawings are
A. Usually maintained in a separate account from the partner’s capital account
B. Equal to partner’s salaries
C. Similar to advances made to partners and are included as assets on the balance sheet
D. Not discussed in the specific contract provisions of the partnership
(FT & C 11e) 241. Which of the following is an advantage of a partnership?
A. Mutual agency
B. Limited life
C. Unlimited liability
D. None of these
(J & C 3e)
242. The profit and loss sharing ratio should be
A. In the same ratio as the percentage interest owned by each partner
B. Based on relative effort contributed to the firm by the partners
C. A weighted average of capital and effort contributions
D. Based on any formula that the partners choose
(J & C 3e)
243. Maxwell is trying to decide whether to accept a salary of $60,000 or a
salary of $25,000 plus a bonus 0f 20% of net income after the bonus as a
means of allocating profit among the partners. What amount of income
would be necessary so that Maxwell would consider the choices to be
equal?
A. $35,000
B. $85,000
C. $140,000 D. $210,000
(FT & C 11e)
244. Joan a senior partner in a fashion designing firm has a share of 30% in
earnings. In 19x8, she transferred to the firm, property with current fair
value of P25,000 but made capital withdrawal of P130,000. If her closing
capital balance was P60,000 less than her beginning capital balance, how
much was the partnership’s net income loss in 19x8?

A. P
(
4
5
,
0
P a g e | 84

0
0
)
B. P
1
3
5
,
0
0
0
C
.
P
1
5
0
,
0
0
0
D
.
P
1
8
0
,
0
0
0
(RRCPA 0598)
245. JJ and KK are partners who share profits and losses in the ratio of 60% and
40% respectively. JJ’s salary is P60,000 and P30,000 for KK, the partners
are also paid interest on their average capital balances. In 2012, JJ received
P30,000 of interest and KK, P12,000. The profit and loss allocation is
determined after deductions for the salary and interest payments. If KK’s
share in the residual income (income after deducting salaries and interest)
was P60,000 in 2012, what was the total partnership income?

A. P192,000 B.
P345,000 C.
P282,000
D. P387,000
(Dayag 11)

ADVANCED FINANCIAL ACCOUNTING AND REPORTING


Problem Portion

Numbers 1 and 2 (Partnership Formation)


P a g e | 85

A, B and C decided to form ABC Partnership. It was agreed that A will contribute an equipment with
assessed value of P100,000 with historical cost of P800,000 and accumulated depreciation of P600,000.
A day after the partnership formation, the equipment was sold for P 300,000.

B will contribute a land and building with carrying amount of P1,200,000 and fair value of P1,500,000.
The land and building are subject to a mortgage payable amounting to P300,000 to be assumed by the
partnership. The partners agreed that B will have 60% capital interest in the partnership. The partners
also agreed that C will contribute sufficient cash to the partnership.

1. What is the total agreed capitalization of the ABC Partnership?


A. 1,500,000
B. 2,000,000
C. 2,500,000
D. 3,000,000

2. What is the cash to be contributed by C in the ABC Partnership?


A. 500,000
B. 600,000
C. 700,000
D. 800,000

Numbers 3 and 4 ( Partnership Operation – Capital Account Transactions )

On January 1, 2018, A, B and C formed ABC Partnership with total agreed capitalization of P1,000,000.
The capital interest ratio of the ABC Partnership is 5:1:4 while the profit or loss ratio is 3:2:5,
respectively for A, B and C.

During 2018, A and B made additional investments of P200,000 and P500,000, respectively. At the end
of 2018, B and C made drawings of P300,000 and P100,000, respectively. On December 31, 2018, the
capital balance of B is reported at P200,000.

3. What is the net income or net loss of ABC Partnership for the year ended December 31, 2018?
A. 500,000 loss
B. 1 ,000,000 loss
C. 800,000 income
D. 1 ,200,000 income

4. What is the capital balance of C on December 31, 2018?


A. 150,000
B. 50,000
C. 200,000
D. 250,000
Numbers 5, 6, and 7 (Partnership Operation – Distribution of profit or loss)

On January 1, 2018, A, B and C formed ABC Partnership with original capital contribution of P300,000,
P500,000 and P200,000. A is appointed as managing partner.
P a g e | 86

During 2018, A, B and C made additional investments of P500,000, P200,000 and P300,000,
respectively. At the end of 2018, A, B and C made drawings of P200,000, P100,000 and P400,000,
respectively. At the end of 2018, the capital balance of C is reported at P320,000. The profit or loss
agreement of the partners is as follows:

• 10 % interest on original capital contribution of the partners.


• Quarterly salary of P40,000 and P10,000 for A and B, respectively.
• Bonus to A equivalent to 20% of Net Income after interest and salary to all partners Remainder
is to be distributed equally among the partners.

5. What is the partnership profit for the year ended December 31, 2018?
A. 900,000
B. 1,020,000
C. 1,050,000
D. 960,000

6. What is A’s share in partnership profit for 2018?


A. 190,000
B. 340,000
C. 540,000
D. 200,000

7. What is B’s share in partnership profit for 2018?


A. 200,000
B. 290,000
C. 50,000
D. 90,000
Number 8 (Admission of partner by purchase)

On December 31, 2018, the Statement of Financial Position of ABC Partnership provided the following
data with profit or loss ratio of 1:6:3:
Current Assets 1,000,000 Total Liabilities 600,000
Noncurrent Assets 2,000,000 A, Capital 900,000
B, Capital 800,000
C, Capital 700,000

On January 1, 2019, D is admitted to the partnership by purchasing 40% of the capital interest of B at a
price of P500,000.

What is the capital balance of B after the admission of D on January 1, 2019?


A. 540,000
B. 480,000
C. 420,000
D. 300,000

Number 9 (Retirement of partner)

On December 31, 2018, ABC Partnership’s Statement of Financial Positions shows that A, B and C have
capital balances of P500,000, P300,000 and P200,000 with profit or loss ratio of 1:3:6. On January 1,
P a g e | 87

2019, C retired from the partnership and received P350,000. At the time of C’s retirement, an asset of
the partnership is undervalued.

What is the capital balance of A after the retirement of C?


A. 462,500
B. 537,500
C. 562,500
D. 525,000

Number 10 (Retirement of partners)

On December 31, 2018, ABC Partnership’s Statement of Financial Position shows that A, B and C have
capital balances of P400,000, P300,000 and P100,000 with profit or loss ratio of 1:4:5. On January 1,
2019, C retired from the partnership and received P80,000. At the time of C’s retirement, the assets and
liabilities of the partnership are properly valued.

What is the capital balance of B after the retirement of C?


A. 284,000
B. 308,000
C. 316,000
D. 320,000
Number 11 (Partnership Dissolution – Admission of New Partner by Investment)

On December 31, 2018, the Statement of Financial Position of ABC Partnership provided the following
data with profit or loss ratio of 1:6:3:
Current Assets 1,300,000 Total Liabilities 300,000
Noncurrent Assets 2,000,000 A, Capital 1,400,000
B, Capital 700,000
C, Capital 900,000
On January 1, 2019, D is admitted to the partnership by investing P1,000,000 to the partnership for 20
% capital interest.
If the all the assets of the existing partnership are properly valued, what is the capital balance of C after
the admission of D?
A. 960,000
B. 900,000
C. 840,000
D. 1,200,000

Numbers 12 and 13 (Admission of new partner by investment)


On December 31, 2018, the Statement of Financial Position of ABC Partnership provided the following
data with profit or loss ratio of 5:1:4:
Current Assets 1,500,000 Total Liabilities 500,000
Noncurrent Assets 2,000,000 A, Capital 1,100,000
B, Capital 1,200,000
C, Capital 700,000
On January 1, 2019, D is admitted to the partnership by investing P500,000 to the partnership for 10%
capital interest. The total agreed capitalization of the new partnership is P3,000,000.

12. What is the capital balance of D after his admission to the partnership?
A. 500,000
P a g e | 88

B. 300,000
C. 350,000
D. 400,000

13. What is the capital balance of C after the admission of D to the partnership?
A. 580,000
B. 820,000
C. 500,000
D. 780,000
Numbers 14 and 15 (Partnership Liquidation – Lump Sum Liquidation)

On December 31, 2018, the Statement of Financial Position of ABC Partnership with profit or loss ratio
of 6:1:3 of partners A, B and C respectively, revealed the following data:

Cash 1,000,000 Other Liabilities 2,000,000


Receivable from A 500,000 Payable to B 1,000,000
Other noncash assets 2,000,000 Payable to C 100,000
A, Capital 700,000
B, Capital (650,000)
C, Capital 350,000
On January 1, 2019, the partners decided to liquidate the partnership. All partners are legally declared to
be personally insolvent. The other noncash assets were sold for P1,500,000. Liquidation expenses
amounting to P100,000 were incurred.

14. How much cash was received by B at the end of partnership liquidation?
A. 250,000
B. 150,000
C. 290,000
D. 270,000

15. How much cash was received by C at the end of partnership liquidation?
A. 270,000
B. 150,000
C. 350,000
D. 220,000
P a g e | 89

Page 6
P a g e | 90

Numbers 16, 17 and 18 (Partnership Liquidation – Installment Liquidation)

On December 31, 2018, the Statement of Financial Position of ABC Partnership with profit or loss ratio
of 5:3:2 of respective partners A, B and C. showed the following information:

Cash 1,600,000 Total Liabilities 2,000,000


Noncash assets 1,400,000 A, Capital 100,000
B, Capital 500,000
C, Capital 400,000
On January 1, 2019, the partners decided to liquidate the partnership in installment. All partners are
legally declared to be personally insolvent.

As of January 31, 2019, the following transactions occurred:


• Noncash assets with a carrying amount P1,000,000 were sold at a gain of P100,000.
• Liquidation expenses for the month of January amounting to P50,000 were paid.
• It is estimated that liquidation expenses amounting to P150,000 will be incurred for the month
of February, 2019.
• 20 % of the liabilities to third persons were settled. Available cash was distributed to the
partners.

As of February 28, 2019, the following transactions occurred:


• Remaining noncash assets were sold at a loss of P100,000.
• The final liquidation expenses for the month of February amounted to P100,000.
• The remaining liabilities to third persons were settled at a compromise amount of P1,500,000.
• Remaining cash was finally distributed to the partners.

16. What is the amount of cash received by partner C on January 31, 2019?
A. 260,000
B. 240,000
C. 300,000
D. 350,000

17. What is the share of B in the maximum possible loss on January 31, 2019?
A. 275,000
B. 110,000
C. 120,000
D. 165,000

18. What is the amount of total cash withheld on January 31, 2019?
A. 550,000
B. 1,600,000
C. 1,750,000
D. 1,700,000
P a g e | 91

Page 7
Numbers 19, 20 and 21 (Corporate Liquidation)

Cagayan Company is experiencing financial problems which resulted to ultimate bankruptcy. The
statement of financial position of the entity before liquidation is presented below:
Cash 100,000 Income tax payable 200,000
Inventory 300,000 Salaries payable 300,000
Land 200,000 Note payable 800,000
Mortgage payable 100,000
Accounts payable 400,000
Contributed capital 500,000
Deficit (1,700,000)
• The note payable is secured by the inventory with net realizable value of P250,000.
• The mortgage payable is secured by the land with fair value of P120,000.

19. What is the amount received by the holder of the note payable at the end of corporate liquidation?
A. 320,000
B. 300,000
C. 250,000
D. 260,000

20. What is the amount received by the holder of the mortgage payable at the end of corporate
liquidation?
A. 120,000
B. 200,000
C. 150,000
D. 100,000

21. What is the amount received by the employees at the end of corporate liquidation concerning their
salaries?
A. 100,000
B. 120,000
C. 72,000
D. 300,000

Page 8
P a g e | 92

Numbers 22 and 23 (Corporate Liquidation)

Surigao Company is bankrupt and has undergone corporate liquidation. Presented below is its statement
of financial position before the start of liquidation:

Cash 300,000 Accounts Payable 100,000


Machinery 500,000 Salaries Payable 200,000
Building 1,200,000 Income tax Payable 300,000
Loan Payable 400,000
Mortgage payable 500,000
Contributed capital 800,000
Deficit (300,000)
• Liquidation expenses amounting to P600,000 were paid.
• The loan payable is secured by the machinery with fair value of P300,000.
• The mortgage payable is secured by the building.
• At the end of liquidation, the holder of loan payable received P340,000.

22. What is the amount received by the holder of accounts payable at the end of liquidation?
A. 85,000
B. 15,000
C. 40,000
D. 60,000

23. What is the amount of net free assets available at the end of liquidation?
A. 80,000
B. 40,000
C. 120,000
D. 200,000
P a g e | 93

Page 9
Numbers 24, 25 and 26 (Joint Arrangement classified as Joint Operation)

Entity A and Entity B incorporated Entity C to manufacture a microchip to be used by the incorporating
entities as component for their final products of cellular phones and tablets.

The contractual agreement of the incorporating entities provided that the decisions on relevant activities
of Entity C will require the unanimous consent of both entities.

Entity A and Entity B have rights to the assets, and obligations for the liabilities, relating to the
arrangement. The ordinary shares of Entity C will be owned by Entity A and Entity B in the ratio of
60:40 . At the end of first operation of Entity C, the financial statements provided the following data :

Inventory 1,000,000 Accounts payable 2,000,000


Land 3,000,000 Note payable 1,000,000
Building 5,000,000 Loan payable 4,000,000
Share capital 1,000,000
Retained earnings 1,000,000
Sales revenue 5,000,000
The contractual agreement of Entity A and Entity B also provided for the following concerning the assets
and liabilities of Entity C:
• Entity A owns the land and incurs the loan payable of Entity C.
• Entity B owns the building and incurs the note payable of Entity C.
• The other assets and liabilities are owned or owed by Entity A and Entity B on the basis of their
capital interest in Entity C.
• The sales revenue of Entity C includes sales to Entity A and Entity B in the amount of P1,000,000
and P2,000,000, respectively. As of the end of the first year, Entity A and Entity B were able to resell
30% and 60% of the inventory coming from Entity C to third persons.

24. What is the amount of total assets to be reported by Entity A concerning its interest in Entity C?
A. 5,400,000
B. 3,000,000
C. 3,600,000
D. 5,000,000

25. What is the amount of total liabilities to be reported by Entity B concerning its interest in Entity C?
A. 1,800,000
B. 2,200,000
C. 2,800,000
D. 2,400,000

26. What is the amount of sales revenue to be reported by Entity A concerning its interest in Entity C?
A. 2,300,000
B. 2,100,000
C. 3,000,000
D. 2,500,000
P a g e | 94

Page 10
Numbers 27 and 28 ( Joint Arrangement classified as Joint Venture Equity Method )

On January 1, 2018, Entity A, a public entity, and Entity B, a public entity, incorporated Entity C which
has its fiscal and operational autonomy. The contractual agreement of the incorporating entities provided
that the decisions on relevant activities of Entity C will require the unanimous consent of both entities.
Entity A and Entity B will have rights to the net assets of Entity C.

Entity A and Entity B invested P1,000,000 and P1,500,000, respectively, equivalent to 40:60 capital
interest of Entity C. The financial statements of Entity C provided the following data for its two-year
operation:
Net income (loss) Dividends declared
2018 200,000 100,000
2019 (2,000,000) -

27. What is the balance of Investment in Entity C to be reported by Entity A in its Statement of Financial
Position on December 31, 2019?
A. 1,080,000
B. 1,040,000
C. 240,000
D. 200,000

28. What is the balance of Investment in Entity C to be reported by Entity B in its Statement of Financial
Position on December 31, 2019?
A. 1,500,000
B. 1,620,000
C. 360,000 D. 900,000
P a g e | 95

Page 11
Numbers 29 and 30 ( Joint venture - Intercompany Transaction )

On January 1, 2018, Entity A, a public entity, and Entity B, a public entity, incorporated Entity C by
investing P3,000,000 and P2,000,000 for capital interest ratio of 60:40. The contractual agreement of
the incorporating entities provided that the decisions on relevant activities of Entity C will require the
unanimous consent of both entities. Entity A and Entity B will have rights to the net assets of Entity C.

The financial statements of Entity C provided the following data for 2018:

• Entity C reported net income of P1,000,000 for 2018 and paid cash dividends of P400,000 on
December 31, 2018.

• During 2018, Entity C sold inventory to Entity A with gross profit of P50,000. Eighty percent of
those inventories were resold by Entity A to third persons during 2018 and the remainder was resold
to third persons during 2019.

• On July 1, 2018, Entity C sold a machinery to Entity B at a loss of P20,000. At the time of sale, the
machinery has remaining useful life of 2 years.

29. What is the investment income to be reported by Entity A for the year ended December 31, 2018?
A. 603,000
B. 606,000
C. 594,000 D. 597,000

30. What is the balance of Investment in Entity C to be reported by Entity B on December 31, 2018?
A. 2,242,000
B. 2,241,000
C. 2,238,000
D. 2,248,000
P a g e | 96

Page 12
Numbers 31 and 32 Joint Venture – IFRS for SMEs (Fair Value Model or Equity Method)

On January 1, 2018, Entity A and Entity B, both SMEs, incorporated Entity C, a jointly controlled entity
by investing P500,000 each in exchange for 10,000 ordinary shares each of Entity C. Entity A and Entity
B each incurred P20,000 transaction costs.
The contractual agreement of the incorporating entities provided that the decisions on relevant activities
of Entity C will require the unanimous consent of both entities. Entity A and Entity B will have rights to
the net assets of Entity C.
For the year ended December 31, 2018, Entity C reported net income of P100,000 and declared dividends
in the amount of P30,000.
On December 31, 2018, the ordinary shares of Entity C are quoted at P56.

31. If Entity A elected fair value model to account its investment in Entity C, what is the net effect on
Entity A’s profit or loss for the year ended December 31, 2018?
A. 55 ,000 net profit
B. 60 ,000 net profit
C. 15 ,000 net profit
D. 40 ,000 net profit

32. If Entity B elected equity method to account its investment in Entity C, what is the carrying amount
of Entity B’s Investment in Entity C on December 31, 2018?
A. 520,000
B. 540,000
C. 535,000 D. 555,000

Numbers 33 and 34 Joint Venture – IFRS for SMEs (Cost Method or Equity Method)

On January 1, 2018, Entity A and Entity B, both SMEs, incorporated Entity C, a jointly controlled entity
by investing P200,000 each in exchange for 20,000 ordinary shares each of Entity C. Entity A and Entity
B each incurred P10,000 transaction costs.
The contractual agreement of the incorporating entities provided that the decisions on relevant activities
of Entity C will require the unanimous consent of both entities. Entity A and Entity B will have rights to
the net assets of Entity C.
For the year ended December 31, 2018, Entity C reported net income of P50,000 and declared dividends
in the amount of P10,000.
On December 31, 2018, the investment in Entity C has value in use of P215,000.

33. If Entity A elected cost method to account its Investment in Entity C, what is the carrying amount of
Entity A’s Investment in Entity C on December 31, 2018?
A. 210,000
B. 215,000
C. 230,000
D. 200,000

34. If Entity B elected equity method to account its Investment in Entity C, what is the net effect in
Entity B’s profit or loss for the year ended December 31, 2018?
A. 25 ,000 net profit
B. 5 ,000 net profit
C. 10 ,000 net profit D. 15 ,000 net profit
P a g e | 97

Page 13
Number 35 (Installment sales)

Nikko Company, which began operations on January 5, 2018, appropriately uses the installment method
of revenue recognition. The following information pertains to the operations for 2018 and 2019:

2018 2019
Sales 300,000 450,000
Collections from :
2018 sales 100,000 50,000
2019 sales - 150,000
Accounts written off from
2018 sales 25,000 75,000
2019 sales - 150,000
Gross profit rates 30% 40 %
What amount should be reported as deferred gross profit on December 31, 2019?
A. 75,000
B. 80,000
C. 112,000
D. 125,000

Numbers 36 and 37 ( Installment sales)

Appliance Company reports gross profit on the installment basis. The following data are available:

2018 2019 2020


Installment sales 240,000 250,000 300,000
Cost of goods – installment sales 180,000 181,250 216,000
Gross profit 60,000 68,750 84,000

Collections
2018 installment contracts 45,000 75,000 72,500
2019 installment contracts 47,500 80,000
2020 installment contracts 62,500

Defaults
Unpaid balance of 2018 installment contracts 12,500 15,000
Value assigned to repossessed merchandise 6,500 6,000
Unpaid balance of 2019 installment contracts 16,000
Value assigned to repossessed merchandise 9,000
36. What is the realized gross profit before loss on repossession for 2020?
A. 49,775
B. 57,625
C. 48,975
D. 56,625

1. What is the loss on repossession for 2020?


A. 5,250
B. 2,600
C. 7,850
D. 9,000
P a g e | 98

Page 14

Number 38 (Installment sales)

Davao Company uses the installment method of income recognition. The entity provided the following
pertinent data:

2018 2019 2020


Installment sales 300,000 375,000 360,000
Cost of goods sold 225,000 285,000 252,000

Balance of Deferred Gross Profit at Year end


2018 52,500 15,000 -
2019 54,000 9,000
2020 72,000
What is the total balance of the Installment Accounts Receivable on December 31, 2020?
A. 270,000
B. 277,500
C. 279,000
D. 300,000

Numbers 39 and 40 (Installment Sales)

On January 1, 2018, an entity sold a car to a customer at a price of P400,000 with a production cost of
P300,000. It is the entity’s policy to employ installment method to recognize gross profit from
installment sales.

At the time of sale, the entity received cash amounting to 25% of the selling price and old car with trade-
in allowance of P50,000. The said old car has fair value of P150,000. The customer issued a 5- year
note for the balance to be payable in equal annual installments every December 31 starting 2018 . The
note payable is interest bearing with 10% rate due on the remaining balance of the note.

The customer was able to pay the first annual installment and corresponding interest due. However, after
the payment of the second interest due, the customer defaulted on the second annual installment which
resulted to the repossession of the car sold with appraised value of P110,000. On December 31, 2019 ,
the repossessed car was resold for P140,000 after reconditioning cost of P 10,000.

39. What is the entity’s realized gross profit for the year ended December 31, 2018?
A. 50,000
B. 120,000
C. 108,000
D. 128,000

40. What is the loss on repossession for the year ended December 31, 2019?
A. 30,000
B. 20,000
C. 10,000
D. 40,000
Page 15

Numbers 41, 42 and 43 (Revenue Recognition – Franchise Fees)

On January 1, 2018, an entity granted a franchise to a franchisee. The franchise agreement required the
franchisee to pay a nonrefundable upfront fee in the amount of P400,000 and on-going payment of
P a g e | 99

royalties equivalent to 5% of the sales of the franchisee. The franchisee paid the nonrefundable upfront
fee on January 1, 2018.

In relation to the nonrefundable upfront fee, the franchise agreement required the entity to render the
following performance obligations:

• To construct the franchisee’s stall with stand-alone selling price of P200,000.


• To deliver 10,000 units of raw materials to the franchisee with stand-alone selling price of P250,000.
• To allow the franchisee to use the entity tradename for a period of 10 years starting January 1, 2018
with stand-alone selling price of P50,000.

On June 30, 2018, the entity completed the construction of the franchisee’s stall. On December 31, 2018
, the entity was able to deliver 3,000 units of raw materials to the franchisee. For the year ended
December 31, 2018, the franchisee reported sales revenue amounting to P100,000.

The entity had determined that the performance obligations are separate and distinct from one another.

41. What is the amount of nonrefundable upfront fee to be allocated to the construction of the
franchisee’s stall?
A. 200,000
B. 160,000
C. 250,000
D. 120,000

42. What is the amount of revenue to be recognized in relation to the use of delivery of raw materials
for the year ended December 31, 2018?
A. 100,000
B. 200,000
C. 60,000
D. 75,000

43. What is the amount of revenue to be recognized in relation to the use of entity’s tradename for the
year ended December 31, 2018?
A. 5,000
B. 4,000
C. 50,000
D. 10,000
Page 16

Numbers 44 and 45 (Revenue Recognition – Net Income of Franchisor)

On January 1, 2018, an entity granted a franchise agreement to a franchisee. The contract provided that
the franchisee shall pay an initial franchise fee of P500,000 and on-going payment of royalties equivalent
to 8% of the sales of the franchisee.

On January 1, 2018, the franchisee paid downpayment of P200,000 and issued a 3-year noninterest
bearing note for the balance payable in three equal annual installments starting December 31, 2018. The
note has present value of P240,183 with effective interest rate of 12%.
P a g e | 100

On June 30, 2018, the entity completed the performance obligation of the franchise at a cost of P352,146.
Aside from that, the entity incurred indirect cost of P22,009.

The franchisee started operation on July 1, 2018 and reported sales revenue amounting to P50,000 for
the year ended December 31, 2018. The franchisee paid the first installment on its due date.

44. If the collection of the note receivable is reasonably assured, what is the gross profit to be recognized
by the entity for the year ended December 31, 2018 in relation to the initial franchise fee?
A. 66,028
B. 44,014
C. 22,009
D. 88,037

45. If the collection of the note receivable is reasonably assured, what is the net income to be reported
by the entity for the year ended December 31, 2018?
A. 98,850
B. 94,850
C. 70,028
D. 92,037
Page 17

Numbers 46, 47 and 48 (Construction contract - Percentage of Completion Method)

On January 1, 2018, Solid Company accepted a long-term construction project for an initial contract
price of P1,000,000 to be completed on June 30, 2020. On January 1, 2019, the contract price was
increased to P1,500,000 by reason of change in the design of the project. The outcome of the construction
contract can be estimated reliably. The project was completed on December 31, 2020 which resulted to
penalty amounting to P200,000. The entity provided the following data concerning the direct costs
related to the said project for 2018 and 2019:

2018 2019
Costs during the year 440,000 680,000
Remaining estimated costs to complete at year-end 660,000 280,000

46. What is the construction revenue for the year ended December 31, 2018?
A. 340,000
B. 400,000
C. 440,000 D. 360,000

47. What is the realized gross profit for the year ended December 31, 2019?
A. 200,000
B. 80,000
C. 180,000
D. 100,000

48. What is the balance of construction in progress on December 31, 2019?


A. 1,200,000
B. 1,020,000
P a g e | 101

C. 1,120,000
D. 900,000
Page 18

Numbers 49, 50 and 51 (Construction contract - Cost Recovery Method)

On January 1, 2018, Hardrock Company started the construction of a building at a fixed contract price
of P1,000,000. On the same date, the customer paid a mobilization fee equal to 5% of contract price that
will be deductible from the first billing. The outcome of construction contract cannot be estimated
reliably

During 2018, the entity billed the customer equivalent to 30% of the contract price. During 2019, the
entity billed again the customer amounting to 20% of the contract price. During 2020, the entity billed
again the customer amounting to 40% of the contract price. The remaining billing was made at the year
of completion of the project.

The entity made collection from the customer at the end of 2018, 2019 and 2020, in the amount of
P120,000, P450,000 and P180,000, respectively. The entity provided the following data concerning the
direct costs related to the said project:

2018 2019 2020


Cumulative costs incurred at year-end 360,000 800,000 870,000
Remaining estimated costs to complete at year-end 840,000 250,000 50,000
49. What is the realized gross profit for the year ended December 31, 2019?
A. 50,000
B. 200,000
C. 150,000
D. 0

50. What is the excess of construction in progress over progress billings or excess of progress billings
over construction in progress on December 31, 2020?
A. 30 ,000 excess billings
B. 80 ,000 excess billings
C. 20 ,000 excess construction in progress
D. 50 ,000 excess construction in progress

51. What is the balance of accounts receivable on December 31, 2020?


A. 150,000
B. 100,000
C. 120,000
D. 50,000
Page 19

Numbers 52, 53 and 54 ( Home Office, Branch and Agency Transactions )

Siargao Company set up a branch in a province. The entity and its branch provided the following data
for the second year of branch operation:

Home Office Branch


Sales revenue to outside customer 1,000,000 500,000
Beginning inventory 50,000 30,000
Purchases from outside supplier 400,000 100,000
Shipment to branch 200,000
P a g e | 102

Shipment from home office 250,000


Ending inventory 80,000 50,000
Operating expenses 150,000 40,000
• The home office to branch markup based on cost is 25% this year and last year.
• 20 % of the beginning inventory of the branch came from outside supplier.
• 24 % of the ending inventory of the branch came from the last year’s shipment from the home office
while 50% of the ending inventory of the branch came from current year’s shipment from the home
office.

52. What is the net income reported by the branch in its separate income statement for the current year?
A. 130,000
B. 124,000
C. 114,000
D. 95,000

53. What is the ending inventory to be reported by the entity in its combined statement of financial
position?
A. 128,000
B. 115,000
C. 130,000
D. 122,600

54. What is the overstatement in the cost of goods sold reported by the branch in its separate income
statement for the current year?
A. 54,000
B. 50,000
C. 52,000
D. 47,400
Page 20

Number 55 (Home office and branch)

The home office in Quezon City ships and bills merchandise to its provincial branch at cost. The branch
carries its own accounts receivable and makes its own collections. The branch also pays its expenses.
The branch transactions for 2018 are reflected in the following information:

Cash 20,000
Accounts receivable 80,000
Home Office 180,000
Shipments from Home Office 250,000
Sales 225,500
Expenses 55,500 December 31, 2018 inventory 65,000

What is the balance of the Investment in Branch account in the home office book?
A. 180,000
B. 195,000
C. 165,000
D. 175,000

Numbers 56 and 57 (Home office and branch)


P a g e | 103

Coffee Company decided to open a branch in Manila. Shipments of merchandise to the branch totaled
P54,000 which included a 20% markup on cost. All accounting records are kept at the home office. The
branch submitted the following report summarizing the operations for the year ended December 31,
2018:

Sales on account 74,000


Sales on cash basis 22,000
Collections of accounts receivable 60,000
Expenses paid 38,000
Expenses unpaid 12,000
Purchase of merchandise for cash 26,000
Inventory on hand, December 31; 80% from home office 30,000
Remittance to home office 55,000
56. What is the branch inventory on December 31, 2018 at cost?
A. 25,000
B. 20,000
C. 26,000
D. 10,000

57. What is the branch net income for the current year?
A. 1,000
B. 4,000
C. 800
D. 500
Page 21

Numbers 58, 59, 60 and 61 (Business Combination - Acquisition of Net Assets)

Entity A acquired the net assets of Entity B by issuing 10,000 ordinary shares with par value of P10 and
bonds payable with face amount of P500,000. The bonds are classified as financial liability at amortized
cost.

At the time of acquisition, the ordinary shares are publicly quoted at P20 per share. On the other hand,
the bonds payable are trading at 110.

Entity A paid P10,000 share issuance costs and P20,000 bond issue costs. Entity A also paid P40,000
acquisition related costs and P30,000 indirect costs of business combination.

Before the date of acquisition, Entity A and Entity B reported the following data:

Entity A Entity B
Current assets 1,000,000 500,000
Noncurrent assets 2,000,000 1,000,000
Current liabilities 200,000 400,000
Noncurrent liabilities 300,000 500,000
Ordinary shares 500,000 200,000
Share premium 1,200,000 300,000
Retained earnings 800,000 100,000
At the time of acquisition, the current assets of Entity A have fair value of P1,200,000 while the
noncurrent assets of Entity B have fair value of P1,300,000. On the same date, the current liabilities of
Entity B have fair value of P600,000 while the noncurrent liabilities of Entity A have fair value of
P500,000.

58. What is the goodwill or gain on bargain purchase arising from business combination?
A. 50 ,000 goodwill
B. 150,000 gain on bargain purchase
P a g e | 104

C. 120,000 goodwill
D. 70 ,000 gain on bargain purchase

59. What total amount should be expensed as incurred at the time of business combination?
A. 20,000
B. 70,000
C. 30,000
D. 50,000

60. What is Entity A’s amount of total assets after the business combination?
A. 4,520,000
B. 4,810,000
C. 4,750,000
D. 4,440,000

61. What is Entity A’s amount of total liabilities after the business combination?
A. 2,240,000
B. 2,510,000
C. 2,320,000
D. 2,130,000
P a g e | 105

Numbers 62 and 63 (Business Combination – Acquisition of majority shares)

Entity A acquired 80,000 out of 100,000 outstanding ordinary shares of Entity B which enabled the
former to obtain control of the latter at an acquisition price of P1,000,000. Entity A paid P100,000
acquisition related costs and P50,000 indirect costs of business combination.

At the date of acquisition, the net assets of Entity B are reported at P1,600,000. An asset of Entity B is
overvalued by P60,000 while one liability is undervalued by P40,000.

62. What is the initial measurement of noncontrolling interest in net assets in the consolidated statement
of financial position?
A. 320,000
B. 300,000
C. 250,000
D. 316,000

63. What is the goodwill or gain on bargain purchase arising from business combination?
A. 250,000 gain on bargain purchase
B. 150,000 gain on bargain purchase
C. 50 ,000 goodwill
D. 200,000 gain on bargain purchase

Numbers 64 and 65 (Step Acquisition)

On January 1, 2018, Entity A acquired 30,000 out of 100,000 outstanding ordinary shares of Entity B
for P90,000 or 30% interest. For the six months ended June 30, 2018, Entity B reported net income of
P40,000.

On July 1, 2018, Entity A acquired additional 60,000 ordinary shares of Entity B or 60% interest at a
price of P4 per share or total cost of P240,000. Entity A paid P20,000 acquisition related costs and
P10,000 indirect costs of business combination.

The acquisition price per share of the additional shares clearly reflected the fair value of the existing
interest of Entity A in Entity B. It is the policy of Entity A to initially measure the noncontrolling interest
in net assets of the acquiree at fair value. The fair value of the noncontrolling interest in net assets of the
acquiree is reliably measured at P50,000.

At the acquisition date, the net assets of Entity B were reported at P400,000. An asset of Entity B was
overvalued by P50,000 while one liability wass overvalued by P30,000.

64. What is the gain on remeasurement of the existing Investment in Entity B as a result of step
acquisition?
A. 18,000
B. 30,000
C. 24,000
D. 12,000

65. What is the goodwill or gain on bargain purchase as a result of the business combination?
A. 18 ,000 goodwill
B. 20 ,000 gain on bargain purchase
C. 24 ,000 goodwill
D. 30 ,000 goodwill
Page 23
P a g e | 106

Numbers 66, 67 and 68 (Consolidated Financial Statements)

On January 1, 2018, Entity A acquired 70% of outstanding ordinary shares of Entity B at a price of
P210,000. On the same date, the net assets of Entity B were reported at P260,000. On January 1, 2018
Entity A reported retained earnings of P2,000,000 while Entity B reported retained earnings of P200,000.

All the assets and liabilities of Entity B are fairly valued except machinery which is undervalued by
P80,000 and inventory which is overvalued by P10,000. The said machinery has remaining useful life
of four years while 40% of the said inventory remained unsold at the end of 2018.

For the year ended December 31, 2018, Entity A reported net income of P1,000,000 and declared
dividends of P200,000 in the separate financial statements while Entity B reported net income of
P150,000 and declared dividends of P20,000 in the separate financial statements.

Entity A accounted the investment in Entity B using cost method in the separate financial statements.

66. What is the noncontrolling interest in net assets on December 31, 2018?
A. 124,800
B. 130,200
C. 126,000
D. 133,800

67. What is the consolidated net income attributable to parent shareholders for the year ended December
31, 2018?
A. 1,102,200
B. 1,162,200
C. 1,141,200
D. 1,095,200

68. What is the amount of consolidated retained earnings on December 31, 2018?
A. 3,012,200
B. 2,991,200
C. 2,952,200
D. 2,945,200
Numbers 69, 70, 71 and 72 (Consolidated Financial Statements - Intercompany sales)

On January 1, 2019, Entity A acquired 60% of outstanding ordinary shares of Entity B at a gain on
bargain purchase of P40,000. For the year ended December 31, 2020, Entity A and Entity B reported
sales revenue of P2,000,000 and P1,000,000 in their respective separate income statements. At the same
year, Entity A and Entity B reported cost of goods sold of P1,200,000 and P700,000 in their respective
separate income statements.

During 2019, Entity A sold inventory to Entity B at a selling price of P280,000 with gross profit rate of
40 % based on cost. On the other hand, Entity B sold inventory to Entity A at a selling price of P400,000
with gross profit rate of 30% based on sales during 2020.
P a g e | 107

On December 31, 2019, 25% of the goods coming from Entity A remained in Entity B’s inventory but
all were eventually sold to third persons during 2020. As of December 31, 2020, 40% of the goods
coming from Entity B were eventually sold to third persons.

For the year ended December 31, 2020, Entity A reported net income of P500,000 while Entity B
reported net income of P200,000 and distributed dividends of P50,000. Entity A accounted for its
inventory in Entity B using cost method in its separate financial statements.

69. What is the consolidated sales revenue for the year ended December 31, 2020?
A. 2,600,000
B. 2,320,000
C. 3,000,000
D. 2,720,000

70. What is the consolidated gross profit for the year ended December 31, 2020?
A. 1,120,000
B. 1,048,000
C. 1,028,000
D. 1,152,000

71. What is the noncontrolling interest in net income for the year ended December 31, 2020?
A. 100,800
B. 59,200
C. 51,200
D. 88,000

72. What is the consolidated net income attributable to parent’s shareholders for the year ended
December 31, 2020?
A. 766,800
B. 596,800
C. 606,800
D. 626,800
P a g e | 108

Numbers 73, 74, 75 and 76 (Consolidated Statements-Intercompany gain or loss on disposal)

On January 1, 2019, Entity A acquired 80% of outstanding ordinary shares of Entity B at a gain on
bargain purchase of P180,000. The following intercompany transactions occurred for between the two
entities:

• On January 1, 2019, Entity B sold a land to Entity A with a cost of P1,000,000 at a selling price of
P1,100,000. The land was eventually sold by Entity A to third persons during 2020.

• On January 1, 2019, Entity A sold a white machinery to Entity B with a cost of P200,000 and
accumulated depreciation of P40,000 at a selling price of P180,000. The machinery is already 4 years
old at the date of sale. The residual value of white machinery is immaterial.

• On July 1, 2020, Entity B sold a black machinery to Entity A at with a cost of P270,000 and
accumulated depreciation of P180,000 at a selling price of P60,000. The machinery is already 6 years
old at the date of sale. The residual value of black machinery is immaterial.

For the year ended December 31, 2020, Entity A reported net income of P800,000 while Entity B
reported net income of P500,000 and distributed dividends of P150,000. Entity A accounted for its
inventory in Entity B using cost method in its separate financial statements.

73. What is the consolidated depreciation expense of machinery for 2020?


A. 40,000
B. 55,000
C. 61,667
D. 42,333

74. What is the consolidated carrying amount of machinery on December 31, 2020?
A. 225,000
B. 215,000
C. 200,000
D. 210,000

75. What is the noncontrolling interest in net income for 2020?


A. 124,000
B. 105,000
C. 125,000
D. 104,000

76. What is the consolidated net income attributable to parent shareholders for 2020?
A. 1,538,750
B. 1,518,750
C. 1,398,750
P a g e | 109

D. 1,418,750
Separate Financial Statements - Cost Method and Fair Value Model or Equity Method

Numbers 77, 78, 79 and 80

On January 1, 2020, Entity A acquired 90% of outstanding ordinary shares of Entity B at a price of
P900,000. Entity A paid P20,000 costs related to acquisition of shares.

At the acquisition date, the net assets of Entity B were reported at P950,000. All the assets of Entity B
are properly valued except for a machinery which is undervalued by P150,000. The machinery has a
remaining useful life of 5 years.

For the year ended December 31, 2020, Entity B reported net income of P200,000 and declared dividends
in the amount of P30,000.

The fair value of Investment in Entity B on December 31, 2020 is P1,000,000 while the cost of disposal
is 5%.

Entity A voluntarily prepared its separate financial statements.

77. If Entity A elects cost method to account its Investment in Entity B in its separate financial
statements, what is the carrying amount of the Investment in Entity B on December 31, 2020?
A. 900,000
B. 920,000
C. 1,000,000
D. 950,000

78. What is the investment income for 2020 if Entity A elects cost method to account its Investment
in Entity B in its separate financial statements?
A. 7,000
B. 27,000
C. 180,000
D. 107,000

79. If Entity A elects fair value model to account its Investment in Entity B in its separate financial
statements, what is the carrying amount of the Investment in Entity B on December 31, 2020?
A. 900,000
B. 920,000
C. 1,000,000
D. 950,000
80. What is the net effect in profit or loss for 2020 if Entity A elects fair value model to account its
Investment in Entity B in its separate financial statements?
A. 7,000
B. 27,000
C. 180,000
D. 107,000
Numbers 81, 82 and 83 (Nonprofit Organization – Statement of Financial Position)
P a g e | 110

In the first year of operations of a nonprofit organization, the following transactions occurred:

• The nonprofit organization received P1,000,000 fund from a donor who stipulated that it shall be
invested indefinitely and the dividend from such investment shall be used for research project of the
organization. Dividend amounting to P150,000 was received during the year but only P50,000 was
spent for the research project.

• The nonprofit organization received P300,000 fund from a donor who stipulated that it shall be used
for the acquisition of service car. The nonprofit organization used P100,000 of the fund for the
acquisition of a service car with useful life of 5 years. The car was acquired at the middle of the year.

• The nonprofit organization received P500,000 fund who stipulated that it shall be used based on the
discretion of the Board of Trustees of the nonprofit organization. The nonprofit organization used
P100,000 for the acquisition of souvenir items which were sold by the nonprofit organization for
P150,000. The remaining P400,000 was designated by the Board of Trustees for future fundraising
projects.

81. What is the amount of permanently restricted net assets at the end of the first year?
A. 1,100,000
B. 1,300,000
C. 1,200,000
D. 1,000,000

82. What is the amount of temporarily restricted net assets at the end of the year?
A. 100,000
B. 300,000
C. 200,000
D. 700,000

83. What is the amount of unrestricted net assets at the end of the year?
A. 640,000
B. 540,000
C. 590,000
D. 630,000
Nonprofit Organization – Statement of Activities and Statement of Cash Flows

Numbers 84, 85, 86 and 87

On January 1, 2020, a nonprofit organization received P1,000,000 cash donation from a donor who
stipulated that the amount should be invested indefinitely in revenue producing investment. The deed of
donation also provided that the dividend income shall be used for the acquisition of computers of the
nonprofit organization.
P a g e | 111

On December 31, 2020, the nonprofit organization received P100,000 cash as dividend income from the
investment of the fund.

On January 1, 2021, the nonprofit organization acquired a computer at a cost of P20,000 with a useful
life of 5 years without residual value.

84. In the statement of activities of the NPO for the year ended December 31, 2020, which of the
following is the proper effect of the transactions?
A. Increase in temporarily restricted net assets by P100,000.
B. Increase in unrestricted net assets by P1,000,000.
C. Increase in unrestricted net assets by P16,000.
D. Decrease in temporarily restricted net assets by P20,000.

85. In the statement of activities of the NPO for the year ended December 31, 2021, which of the
following is the proper effect of the transactions?
A. Increase in temporarily restricted net assets by P100,000.
B. Increase in unrestricted net assets by P1,000,000.
C. Increase in unrestricted net assets by P16,000.
D. Decrease in temporarily restricted net assets by P100,000.

86. How should the cash flows be reported in NPO’s Statement of Cash Flows for the year ended
December 31, 2020?
A. Cash receipts from operating activities by P100,000.
B. Cash receipts from financing activities by P1,100,000.
C. Cash disbursements for investing activities by P50,000.
D. Cash disbursements for financing activities by P1,000,000

87. How should the cash flows be reported in NPO’s Statement of Cash Flows for the year ended
December 31, 2021?
A. Cash receipts from operating activities by P100,000.
B. Cash receipts from financing activities by P1,100,000.
C. Cash disbursements for investing activities by P20,000.
D. Cash disbursements for investing activities by P100,000.
P a g e | 112

Page 29
Number 88
Government Accounting Manual
On December 31, 2018, the Department of Finance billed its lessee on one of its buildings in the amount
of P10,000. On January 31, 2019, the Department of Finance collected all of the accounts receivable. On
February 28, 2019, the Department of Finance remitted the entire collected amount to the Bureau of
Treasury. What is the journal entry to record the remittance by the Department of Finance to the Bureau
of Treasury?
A. Debit – Accounts Receivable P10,000 and Credit – Rent Income P10,000
B. Debit – Accounts Receivable P10,000 and Credit – Retained Earnings P10,000
C. Debit – Cash Collecting Officers P10,000 and Credit – Accounts Receivable P10,000
D. Debit – Cash – Treasury/Agency Deposit, Regular – P10,000 and Credit Cash – Collecting
Officer – P10,000

Numbers 89 and 90
On January 1, 2018, the Department of Public Works and Highways (DPWH) received a P10,000,000
appropriation from the national government for the acquisition of machinery. On February 1, 2018,
DPWH received the allotment from the Department of Budget and Management. On March 1, 2018,
DPWH entered into a contract with CAT Inc. for the acquisition of the machinery with a price of
P8,000,000. On April 1, 2018, DPWH received the Notice of Cash Allocation from Department of
Budget and Management net of 1% withholding tax for income tax of supplier and 5% withholding of
Final Tax on VAT of supplier. On May 1, 2018, CAT Inc. delivered the machinery to DPWH. On June
1 , 2018, DPWH paid the obligation to CAT Inc. On July 1, 2018, DPWH remitted the withheld income
tax and final VAT to BIR.

89. What is the journal entry on March 1, 2018? A. No


entry but just posting to appropriate RAPAL
B. No entry but just posting to appropriate RAPAL and to RAOD
C. No entry but just posting of ORS (Obligation Request and Status) to appropriate RAOD
D. Debit Machinery P8,000,000 and credit Accounts Payable P8,000,000

90. What is the journal entry on April 1, 2018?


A. Debit Cash-MDS, Regular P7,520,000 and Credit Subsidy Income from National
Government P7,520,000.
B. Debit Machinery P8,000,000 and Credit Accounts Payable P8,000,000
C. Debit Accounts Payable P8,000,000 and Credit Due to BIR P480,000 and Cash-MDS, Regular
P7,520,000.
D. Debit Due to BIR P480,000 and Credit Subsidy Income from National Government P480,000.

Number 91
Department of Health (DOH) received Notice of Cash Allocation in the amount of P100,000 from
Department of Budget and Management. DOH made a total cash disbursements in the amount of P95,000.
What is the journal entry to recognize reversion of unused Notice of Cash Allocation by DOH in its
books?
A. Debit Subsidy Income from National Government P5,000 and credit Cash-MDS, Regular
P5,000.
B. Debit Retained Earnings of DFA P5,000 and credit Cash-MDS, Regular P5,000.
C. Debit Expenses of DFA P5,000 and credit Cash-MDS, Regular P5,000.
D. Debit Investment of DFA P5,000 and credit Cash-MDS, Regular P5,000.

Number 92
The Bureau of Treasury received P20,000 cash remittance from Department of Agrarian Reform ( DAR)
from its miscellaneous income. What is the journal entry of the Bureau of Treasury in its accounting
books to record the receipt of cash remittance from the income of a national government agency?
A. Debit Cash in Bank, Local Bank P20,000 and Credit Cash-Treasury/Agency Deposit, Regular
P20,000.
B. Debit Cash in Bank, Local Bank P20,000 and Credit Miscellaneous Income of DA P20,000.
C. Debit Cash in Bank, Local Bank P20,000 and Credit Savings of DA, Regular P20,000.
P a g e | 113

D. Debit Cash in Bank, Local Bank P20,000 and Credit Cash-Collecting Officer, DA P20,000.

Number 93 (Foreign currency transaction)

On September 1, 2018, Bain Company received an order for equipment from a foreign customer for
300,000 local currency units (LCU) when the US dollar equivalent was $96,000. Bain shipped the
equipment on October 15, 2018, and billed the customer for 300,000 LCU when the US dollar equivalent
was $100,000. Bain received the customer remittance in full on November 16, 2018, and sold the 300,000
LCU for $105,000. In the income statement for the year ended December 31, 2018, what amount should
Bain report as part of net income a foreign exchange transaction gain?
A. $ 0
B. $4,000
C. $5,000
D. $9,000

Number 94 (Foreign currency transaction)

On September 1, 2018, Cano Company, a US corporation, sold merchandise to a foreign firm for 250,000
Botswana pula. Terms of the sale require payment in pula on February 1, 2019.. On September 1 , 2018,
the spot exchange rate was $.20 per pula. At December 31, 2018, Cano’s year-end, the spot rate was $.19,
but the rate increased to $.22 by February 1, 2019, when payment was received. How much should Cano
report as foreign exchange transaction gain or loss as part of 2019 income?
A. $ 0
B. $2,500 loss
C. $5,000 gain
D. $7,500 gain

Number 95 (Foreign currency transaction)

Hunt Company purchased merchandise for £300,000 from a vendor in London on November 30, 2018.
Payment in British pounds was due on January 30, 2019. The exchange rates to purchase one pound were
as follows:

November 30, 2018 December 31, 2018


Spot-rate $1.65 $1.62
30-day rate 1.64 1.59
60-day rate 1.63 1.56
In the income statement, what amount should Hunt report as foreign exchange transaction gain as part of
net income?
A. $12,000
B. $ 9,000
C. $ 6,000
D. $ 0
Number 96 (Foreign currency transaction)

Ball Company had the following foreign currency transactions during 2018:

• Merchandise was purchased from a foreign supplier on January 20, 2018, for the US dollar
equivalent of $90,000. The invoice was paid on March20, 2018, at the US dollar equivalent of
$96,000.

• On July 1, 2018, Ball borrowed the US dollar equivalent of $500,000 evidenced by a note payable
in the lender’s local currency on July 1, 2020. On December 31, 2018, the US dollar equivalents of
the principal amount and accrued interest were $520,000 and $26,000, respectively. Interest on the
note is 10% per annum.
P a g e | 114

In Ball’s 2018 income statement, what amount should be included as foreign exchange transaction loss
as part of net income?
A. $ 0
B. $ 6,000
C. $21,000
D. $27,000

Number 97 (Foreign currency transaction)

On November 30, 2018, Tyrola Publishing Company, located in Colorado, executed a contract with
Ernest Blyton, an author from Canada, providing for payment of 10% royalties on Canadian sales of
Blyton’s book. Payment is to be made in Canadian dollars each January 10 for the previous year’s sales.
Canadian sales of the book for the year ended December 31, 2019, totaled $50,000 Canadian. Tyrola paid
Blyton his 2019 royalties on January 10, 2020. Tyrola’s 2019 financial statements were issued on
February 1, 2020. Spot rates for Canadian dollars were as follows:

November 30, 2018 $.87


January 1, 2019 $.88
December 31, 2019 $.89
January 10, 2020 $.90
How much should Tyrola accrue for royalties payable at December 31, 2019?
A. $4,350
B. $4.425
C. $4,450
D. $4,500
Numbers 98, 99, 100 and 101 (Foreign Currency Transaction)

On November 1, 2020, an entity acquired on account goods from a foreign supplier at a cost of $1,000.
The accounts payable are paid on January 30, 2021.

On December 1, 2020, an entity sold on account the said goods to a foreign customer at a selling price of
$1,500. The accounts receivable are collected on February 28, 2021.

The entity is operating in Philippine economy wherein the functional currency is the Philippine Peso.

The following direct exchange rates are provided:

Buying spot rate Selling spot rate


November 1, 2020 P40 P42
December 1, 2020 39 40
December 31, 2020 45 47

98. What is the sales revenue for 2020?


A. 58,500
B. 60,000
C. 67,500
D. 72,000

99. What is the carrying amount of accounts receivable on December 31, 2020?
A. 58,500
B. 60,000
C. 67,500
D. 72,000
P a g e | 115

100. What is the carrying amount of accounts payable on December 31, 2020?
A. 40,000
B. 42,000
C. 45,000
D. 47,000

101. What is the net foreign currency gain for 2020?


A. 4,000
B. 5,000
C. 3,000
D. 6,000
Translation of Financial Statements in Functional Currency to Presentation Currency

Numbers 102, 103, 104 and 105

Entity A owns majority of the outstanding ordinary shares of Entity B which is operating in United
States of America wherein the functional currency is the USA $. However, the presentation currency of
Entity B is the Philippine Peso because that is the presentation currency of Entity A. For the year ended
December 31, 2020, Entity B presented its Statement of Financial Position in its functional currency of
USA $:
Current assets $10,000 Current liabilities $10,000
Noncurrent assets 40,000 Noncurrent liabilities 20,000
Ordinary share capital 5,000
Preference share capital 8,000
Retained earnings 7,000
Total Assets $50,000 Total Liabilities and shareholders $50,000
• The ordinary shares are issued on January 1, 2019 while the preference shares are issued on July 1,
2019.
• B reported $1,000 net income during 2020 and declared dividends in the amount of $200 on
December 1, 2020.
• The translated amount of retained earnings on December 31, 2019 is P300,000.

The following direct exchange rates are provided:

January 1, 2019 P40


July 1, 2019 42
December 31, 2019 43
December 1, 2020 41
December 31, 2020 45
Average rate 2020 44
102. What is the amount of net assets in US dollars on December 31, 2019?
A. 19,200
B. 20,000
C. 19,000
D. 20,200

103. What amount of translation gain as component of other comprehensive income should be presented
in the of statement of comprehensive income for the year ended December 31, 2020?
A. 38,600
B. 39,200
C. 40,400
D. 41,800
P a g e | 116

104. What is the translated retained earnings balance on December 31, 2020?
A. 300,000
B. 335,800
C. 344,000
D. 281,800

105. What is the cumulative translation credit that should to be presented in the statement of financial
position on December 31, 2020?
A. 25,400
B. 28,200
C. 26,800
D. 24,600

Numbers 106 and 107 (Standard Costing – Direct material variance)

Negros Company recently set-up its standard costs for its direct materials. The entity sets the benchmark
at 3 units of direct materials per product at a standard price of P5 per unit of direct material.

During the year, the entity acquired 400 units of direct materials at a total cost of P2,400 or P6 per unit.
The entity also manufactured 100 products using 250 units of direct materials.

106. What is the direct material price variance?


A. 250 unfavorable
B. 300 favorable
C. 350 favorable
D. 400 unfavorable

107. What is the direct material usage variance?


A. 150 unfavorable
B. 300 unfavorable
C. 250 favorable
D. 350 favorable

Numbers 108 and 109 (Standard costing - Direct labor variance)

Bacolod Company recently set-up its standard costs for its direct labor. The entity sets the benchmark at
2 direct labor hours per product at a standard rate of P100 per direct labor hour.

During the year, the entity manufactured 10 products using 30 direct labor hours at total direct labor costs
of P2,400 or P80 per direct labor hour.

108. What is the direct labor rate variance?


A. 600 favorable
B. 400 unfavorable
C. 200 favorable
D. 800 unfavorable
P a g e | 117

109. What is the direct labor efficiency variance?


A. 400 favorable
B. 1 ,000 unfavorable
C. 600 unfavorable
D. 200 favorable
P a g e | 118

Page 35
Number 110 (Job Order Costing)
Simple Company employs actual costing for its production. The entity provided the following data
concerning its production during the year:
Decrease in direct materials during the year 500,000
Labor cost during the year 400,000
Actual factory overhead during the year 300,000
Increase in work in process during the year 200,000
Decrease in finished goods during the year 100,000
What is the cost of goods manufactured during the year?
A. 1,200,000
B. 1,000,000
C. 1,400,000
D. 1,100,000

Numbers 111, 112 and 113 (Job order costing)

Marawi Company employs normal costing for its production. The following data are provided during
the current year:

Net purchases of raw materials during the year 500,000


Total labor costs during the year 800,000
Depreciation of factory assets during the year 100,000
Utilities on the factory during the year 300,000
Beginning Ending
Raw materials inventory 200,000 300,000
Work in process inventory 500,000 200,000
Finished goods inventory 600,000 300,000
• The entity uses a single account for its direct material and indirect materials. Indirect material used
is one-fourth of the total direct material used.
• The indirect labor cost is 1/8 of the total labor costs.
• The overhead application rate is 80% of direct labor costs.
• Any over or under application of overhead is considered material.

111. What is the total manufacturing cost during the current year?
A. 1,560,000
B. 1,500,000
C. 1,640,000
D. 1,740,000

112.What is the cost of goods manufactured during the current year?


A. 2,040,000
B. 1,860,000
C. 1,940,000
D. 1,800,000

113. What is the over or under application of overhead?


A. 60 ,000 over application
B. 140,000 under application
C. 40 ,000 under application
D. 160,000 over application
P a g e | 119

Numbers 114, 115 and 116 (Joint Product and By-Product Costing)

Silay Company is conducting a joint production at a total costs of P500,000. The joint production results
to the following inventories:

Alt Tab Del


Units produced 20,000 units 10,000 units 5 ,000
units
Selling price at split off P150 P200 P5
Alt and Tab are considered main products while Del is considered by-product. The entity considers its
by-product as material. The by-product requires additional processing cost per unit of P0.80 and its cost
of disposal is P0.20 per unit.

114. What is the value to be given to product Del?


A. 25,000
B. 21,000
C. 24,000
D. 20,000

115. What is the joint cost allocated to product Alt if the entity employs physical method?
A. 333,333
B. 316,667
C. 317,333
D. 320,000

116. What is the joint cost allocated to product Tab if the entity employs relative sales value method?
A. 300,000
B. 200,000
C. 192,000
D. 288,000
Number 117 (Just-in-Time Inventory and Backflush Costing)

Talisay Company is employing backflush costing in connection with just-in-time production process.
The entity provided the following production data for the year:

• The entity acquired direct materials during the year at a cost of P100,000 The entity reported direct
labor cost of P200,000.
• The actual factory overhead incurred during the year amounted to P170,000.
• The standard factory overhead application rate is 75% of direct labor cost.
• The ending finished goods inventory is reported at P120,000.

What is the cost of goods sold under backflush costing?


A. 470,000
B. 350,000
C. 330,000
D. 300,000
P a g e | 120

Numbers 118, 119 and 120 (Backflush costing)

Panay Company has a cycle of 3 days, uses a Raw and In Process Account (RIP) and charges all
conversion costs to cost of goods sold. At the end of each month, all inventories are counted, conversion
costs components are estimated and inventory account balances are adjusted. Raw material cost is
backflushed from Raw and in Process (RIP) Account to finished goods. The following information is
provided for the month of June:

Beginning Balance of RIP account, including P1,000 conversion cost 5,000


Beginning Balance of finished goods account including P6,000 conversion cost 10,000
Raw materials received on credit 400,000
Direct labor cost 300,000
Factory overhead applied 500,000
Ending RIP inventory per physical count, including P7,000 conversion cost 20,000
Ending finished goods inventory per physical count, including P4,000 conversion cost 6,000

118.What is the amount of conversion cost included cost of goods sold in June?
A. 802,000
B. 796,000
C. 794,000
D. 800,000

119.What is the amount of direct materials backflushed from RIP to finished goods?
A. 391,000
B. 404,000
C. 387,000
D. 395,000

120. What is the amount of direct materials backflushed from finished goods to cost of goods sold?
A. 395,000
B. 400,000
C. 393,000
D. 389,000
Numbers 121 and 122 (Activity Based Costing)

Romblon Company is choosing between traditional costing and activity-based costing. The following
data are provided:

Activity-Based Costing

Activity center Cost driver Amount of activity Center cost


Material handling Kilos handled 100,000 kg. 200,000
Painting Units painted 50,000 units 300,000
Assembly Machine hours 10,000 hours 500,000

Traditional Costing
Traditional Labor hours 100,000 hours 1,000,000
Job 1 contains 3,000 units. It weighs 10,000 kilos and uses 300 machine hours. The direct labor hours
on the job total 7,000 hours.
P a g e | 121

121. What is the applied overhead under traditional costing?


A. 70,000
B. 60,000
C. 80,000
D. 50,000

122. What is the applied overhead under Activity Based Costing?


A. 53,000
B. 56,000
C. 45,000
D. 43,000
Numbers 123, 124, 125 and 126 (Process Costing without Spoilage)

Tacloban Company is employing process costing regarding its production cycle.

Conversion costs are added uniformly during the production process while direct materials are added
10 % at the start of production process, 50% at the middle of the production process and the remainder
at the end of production process.

The production data of the entity during the year are:

Beginning Work in Process Inventory 10 ,000 units (30% incomplete as to conversion costs
)
Units started during the year 30,000 units
Ending Work in Process Inventory 5,000 units (75% incomplete as to conversion costs)
• There is no spoilage during the period.
• The costs of beginning inventory consist of P103,000 costs of direct materials and P107,500
conversion costs.
• The total manufacturing costs consist of P252,000 costs of direct materials and P146,250 conversion
costs.

123. What is the cost per unit of direct material under average process costing?
A. 10
B. 9
C. 8
D. 7

124. What is the cost per unit of conversion cost under average process costing?
A. 10
B. 9
C. 8
D. 7

125. What is the cost per unit of direct material under FIFO process costing?
A. 10
B. 9
C. 8
P a g e | 122

D. 7

126. What is the cost per unit of conversion cost under FIFO process costing?
A. 5
B. 9
C. 8
D. 7
P a g e | 123

Page 40

Numbers 127, 128, 129, 130 and 131 (Process Costing with Spoilage)

Samar Company is employing process costing regarding its production cycle.

Conversion costs are added uniformly during the production process while direct materials are added 20
% at the start of production process, 45% at the middle of the production process and the remainder at
the end of production process. Normal spoilage is 10% of units started during the year.

The entity is conducting inspection when the production process is at 45% of conversion cost. The entity
provided the following production data during the year:

Beginning Work in Process Inventory 10 ,000 units (40% incomplete as to conversion costs
)
Units started during the year 40,000 units
Ending Work in Process Inventory 5,000 units (80% complete as to conversion costs)
Units completed during the period 38 ,000 units
127. What is the abnormal spoilage in units during the year?
A. 7 ,000 units
B. 4 ,000 units
C. 3 ,000 units D. 2 ,000 units

128. What is the equivalent unit of production for direct material under average process costing?
A. 42 ,650 units
B. 41 ,150 units
C. 38 ,250 units
D. 43 ,750 units

129. What is the equivalent unit of production for conversion cost under average process costing?
A. 44 ,650 units
B. 45 ,150 units
C. 43 ,250 units
D. 46 ,150 units

130. What is the equivalent unit of production for direct material under FIFO costing?
A. 35 ,150 units
B. 37 ,250 units
C. 36 ,150 units
D. 38 ,450 units

131. What is the equivalent unit of production for conversion cost under FIFO costing?
A. 39 ,150 units
B. 41 ,250 units
C. 37 ,450 units
D. 38 ,650 units
P a g e | 124

Numbers 132, and 133 (Foreign currency hedge)

On December 1, 2020, Entity A imported goods at a price of $1,000 payable on March 1, 2021. In order
to hedge this foreign currency denominated importation, Entity A entered into a forward contract with a
bank to purchase $1,000. Entity A is operating in Philippine economy where the functional currency is
Philippine peso. The following direct exchange rates are given:

December 1, 2020 December 31, 2020 March 1, 2021


Buying spot P43 P40 P41
Selling spot 45 44 49
132. What is the foreign currency gain or loss on the hedged item for 2020?
A. 2 ,000 loss
B. 1 ,000 gain
C. 3 ,000 gain
D. 4 ,000 gain
133. What is the foreign currency gain or loss on the hedging instrument for 2021?
A. 4 ,000 gain
B. 2 ,000 loss
C. 1 ,000 loss
D. 3 ,000 gain
P a g e | 125

Numbers 134 and 135 (Hedging)

Kline Company purchased inventory on November 30, 2018 for $10,000 payable March 1, 2019. On
December 1, 2018, the entity entered into a forward contract to purchase $10,000in 90 days to hedge
the purchase of inventory on November 30, 2018. The relevant exchange rates are:

Spot rate Forward rate


November 30, 2018 P45 P47
December 1, 2018 46 48
December 31, 2018 50 51
134. What amount of foreign currency transaction gain from the forward contract should be included in
net income for 2018?
A. 50,000
B. 40,000
C. 30,000 D. 0

135. What amount of foreign currency transaction loss should be included in income from the
revaluation of accounts payable for 2018?
A. 40,000
B. 50,000
C. 10,000
D. 0

Number 136 (Hedging)

On December 1, 2018 Winston Company entered into a forward contract to purchase $10,000 in 90 days
to hedge a commitment to purchase equipment being manufactured to the entity’s specifications. The
expected delivery date is March 1, 2019, at which time settlement is due to the manufacturer. The hedge
qualifies as a fair value hedge. The relevant exchange rates are:

Spot rate Forward rate


December 1, 2018 P48 P49
December 31, 2018 52 51
What amount of foreign currency transaction gain from the forward contract should be included in net
income for 2018?
A. 20,000
B. 40,000
C. 10,000
D. 0
Numbers 137 and 138 (Fair value hedge)

On November 1, 2020, Entity A entered into a firm commitment for the exportation of goods at a price
of $2,000. Delivery will happen on January 31, 2020. In order to hedge this foreign currency denominated
firm commitment, Entity A entered into a forward contract with a bank to sell $2,000. Entity A is
operating in Philippine economy where the functional currency is Philippine peso. Entity A elects to use
fair value hedge to account this hedge of firm commitment. The following direct exchange rates are
given:
P a g e | 126

November 1, 2020 December 31, 2020 January 31, 2021

Spot rate P43 P40 P44


90-day forward rate 41 43 44
60-day forward rate 45 42 41
30-day forward rate 47 46 42
137. What is the carrying amount of firm commitment asset or liability on December 31, 2020?
A. 4 ,000 liability
B. 10 ,000 liability
C. 2 ,000 liability
D. 6 ,000 liability

138. What is the foreign currency gain or loss on hedging instrument for 2021?
A. 4 ,000 gain
B. 2 ,000 loss
C. 6 ,000 loss
D. 8 ,000 gain
Page 44

Numbers 139, 140, 141 and 142 (Cash flow hedge)

On November 1, 2020, Entity A anticipated the purchase of equipment on January 31, 2021 at a price of
$1,200. In order to hedge this highly probable forecasted importation, Entity A entered into a forward
contract with a bank to purchase $1,200. Entity A is operating in Philippine economy where the functional
currency is Philippine peso. The following direct exchange rates are made available:

November 1, 2020 December 31, 2020 January 31, 2021


Spot rate P45 P44 P43
90-day forward rate 42 41 43
60-day forward rate 46 45 40
30-day forward rate 48 44 40

139. What is the unrealized holding gain or loss to be recognized as component of other comprehensive
income in the statement of comprehensive income for the year ended December 31 , 2020?
A. 2 ,400 gain
B. 1 ,200 gain
C. 3 ,600 loss
D. 4 ,800 gain

140. What is the unrealized holding gain or loss to be recognized as component of other comprehensive
income in the statement of comprehensive income for the year ended December 31 , 2021?
A. 4 ,800 loss
B. 1 ,200 loss
C. 3 ,600 gain
D. 2 ,400 gain

141. What is the cumulative unrealized gain or loss before reclassification to be reported as component
of other comprehensive income in the Statement of Changes in equity on December 31 , 2021?
A. 1 ,200 gain
B. 1 ,800 loss
C. 2 ,400 gain D. 0

142. What is the cost of equipment in Philippine peso on January 31, 2021?
A. 48,000
B. 50,400
C. 49,200
D. 51,600
Page 45
Numbers 143, 144 and 145 (Cash Flow Hedge using option contract)

On November 1, 2020, Entity A anticipated the purchase of inventory on January 31, 2021 at a price of
$1,000. In order to hedge this highly probable forecasted importation, Entity A acquired a call option
from a bank giving it the right to purchase $1,000 at an option price of P40 by paying an option premium
of P300. Entity A is operating in Philippine economy where the functional currency is Philippine peso.
The following data are provided:

November 1, 2020 December 31, 2020 January 31, 2021


Spot rate P40 P 44 P43 Fair value of call option ? 4,500 ?

Entity A imported the goods on the date anticipated. Afterwards, Entity A was able to resell 30% of the
goods imported during 2021.

143. What is the unrealized holding gain or to be recognized as component of other comprehensive
income in the of statement of comprehensive income for the year ended December 31, 2020?
A. 4,000
B. 4,500
C. 4,300
D. 4,200

144. What is the unrealized holding gain to be recognized in the profit or loss in the statement
comprehensive income for the year ended December 31, 2020?
A. 300
B. 200
C. 500
D. 100

145. What is the unrealized holding loss to be recognized as component of other comprehensive income
in the statement of comprehensive income for the year ended December 31, 2021?
A. 3,000
B. 2,000
C. 1,000
D. 4,000

END
1.1 Partnership Formation
Problem 1 (ReSA)

On July 1, 2019, XX and YY decided to form a partnership. The firm is to take over business
assets and assume liabilities, and capitals are to be based on net assets transferred after the
following adjustments:

a) XX and YY’s inventory is to be valid at P31,000 and P22,000, respectively.


b) Accounts receivable of P2,000 in XX’s book and P1,000 in YY’s books are uncollectible.
c) Accrued salaries of P4,000 for XX and P5,000 for YY are still to be recognized in the
books.
d) Unused office supplies of XX amounted to P5,000, while that of YY amounted to P1,500.
e) Unrecorded patent of P7,000 and prepaid rent of P4,500 are to be recognized in the
books of XX and YY, respectively.
f) XX is to invest or withdrew cash necessary to have a 40% interest in the firm.

Balance sheets for XX and YY on July 1 before adjustments are given below:
XX YY
Cash Php 31,000 Php 50,000
Accounts Receivable 26,000 20,000
Inventory 32,000 24,000
Office Supplies 5,000
Equipment 20,000 24,000
Accumulated Depreciation - Equipment (9,000) (3,000)
Total Assets Php 100,000 Php 120,000

Accounts Payable Php 28,000.00 Php 20,000.00


Capitals 72,000 100,000
Total Liabilities and Capital Php 100,000 Php 120,000

Determine:
1. The net adjustments – capital in the books of XX and YY:
a. XX, P7,000 net debit; YY, P2,000 net credit
b. XX, P5,000 net debit; YY, P7,000 net credit
c. XX, P7,000 net credit; YY, P2,000 net debit
d. XX, P5,000 net credit; YY, P7,000 net debit

2. The adjusted capital of XX and YY in their respective books.


a. XX – P65,000; YY – P102,000 c. XX – P77,000; YY – P98,000
b. XX – P63,000; YY – P107,000 d. XX – P77,000; YY – P93,000
3. The additional investment (withdrawal) made by XX:
a. P(15,000.00) c. P3,000.00
b. P( 6,666.50) d. P8,377.50

4. The total assets of the partnership after formation:


a. P235,333.50 c. P220,333.50
b. P230,000.00 d. P212,000.00

5. The total liabilities of the partnership after formation:


a. P57,000.00 c. P54,000.00
b. P48,000.00 d. P51,000.00

6. The total capital of the partnership after formation:


a. P180,000.00 c. P163,333.50
b. P178,333.50 d. P155,000.00

7. The capital balances of XX and YY in the combined balance sheet:


a. XX, P81,250; YY, P72,000 c. XX, P100,000; YY, P75,000
b. XX, P81,250; YY, P75,000 d. XX, P 62,000; YY, P93,000

Solution
XX Capital YY Capital
1. D
a (1,000.00) a (2,000.00)

b (2,000.00) b (1,000.00)
c (4,000.00) c (5,000.00)

d 5,000.00 d (3,500.00)
e 7,000.00 e 4,500.00

Net Credit 5,000.00 Net Debit (7,000.00)

2. D

XX Capital- Unadjusted 72,000.00 YY Capital- Unadjusted 100,000.00

Net adjustment 5,000.00 Net adjustment (7,000.00)


Adjusted Capital 77,000.00 Adjusted Capital 93,000.00

3. A
Addtl Investment (
CC AC Withdrawal )
XX 77,000.00 62,000.00 (15,000.00)
YY 93,000.00 93,000.00
150,000.00 155,000.00 (15,000.00)

4. D From the accounting equation Asset = Liability + Capital


5. A
Total Asset Liability Capital
6. D
28,000.00 77,000.00
20,000.00 93,000.00
4,000.00 (15,000.00)
5,000.00
212,000.00 57,000.00 155,000.00
7. D
XX (155,000x40%) 62,000
YY (155,000x60%) 93,000

Problem 2 (ReSA)

On December 1, 2019, AA and BB formed a partnership with contributing the following assets at fair
market values:

AA BB

Cash ……………………………………… P 9,000 P 18,000 Machinery and equipment …..


13,500 - Land ……………………………………… - 90,000
Building ………………………………… - 27,000
Office Furniture ……………………. 13,500 -

The land and building are subject to a mortgage loan of P54,000 that the partnership will
assume. The partnership agreement provides that AA and BB share profits and losses, 40%
and 60%, respectively and partners agreed to bring their capital balances in proportion to the
profit and loss ratio and using the capital balance of BB as the basis. The additional cash
investment made by AA should be:
a. P18,000.00 c. P134,000.00
b. P85,500.00 d. P166,250.00

Solutions:

AA BB

Cash 9,000.00 18,000.00

Machinery & Eqmpt 13,500.00

Land 90,000.00

Building 27,000.00

Office Furniture 13,500.00

Mortgage Loan (54,000.00)

Capital 36,000.00 81,000.00

Addtl
Investment
CC AC ( Withdrawal )
AA 40% 36,000.00 54,000.00 18,000.00

BB 60% 81,000.00 81,000.00 -

117,000.00 135,000.00 18,000.00

Problem 3 (ReSA)

CC and DD are joining their separate business to form a partnership. Cash and non-cash assets
are to be contributed for a total capital of P150,000. The non-cash assets to be contributed
and liabilities to be assumed are:

CC DD
Book Value Fair Value Book Value Fair
Value
Accounts Receivable … P11,250.00 P11,250.00
Inventories ……………….. 11,250.00 16,875.00 P30,000.00
P33,750.00
Equipment ………………… 18,750.00 15,000.00 33,750.00
35,625.00
Accounts Payable …..... 5,637.50 5,625.00 3,750.00
3,750.00

The partner’s capital accounts are to be equal after all contributions of assets and assumptions of
liabilities.

Determine:

1. The total assets of the partnership.


a. P159,375.00 c. P140,625.00
b. P150,000.00 d. P112,500.00
2. The amount of cash that each partner must contribute:
a. CC – P37,500; DD – P9,375 c. CC – P80,625; DD – P78,750
b. CC – P37,500; DD – P5,625 d. CC – P63,750; DD – P5,625

Solution

1. A Asset Liability Capital


5,625.00 150,000.00
3,750.00 -
159,375.00 9,375.00 150,000.00

2. A
CC DD

Cash 37,500.00 9,375.00


Problem 4 (ReSA)
Accounts Receivable 11,250.00 0.00
On December 1, Inventories 16,875.00 33,750.00 2018, EE and FF
formed a partnership
agreeing to share Equipment 15,000.00 35,625.00 for profits and
losses in the ration of 2:3 respectively.
Total Assets 80,625.00 78,750.00
EE invested a parcel of land that
cost him 25,000. FF invested 30,000
cash. The land was sold for 50,000 on
the same date, Accounts Payable 5,625.00 3,750.00 three hours after
formation of the partnership. How
Capital 75,000.00 75,000.00
much should be the capital balance of
EE right after Total Liabilities and Capital 80,625.00 78,750.00 formation?

a. 25,000 c. 60,000
b. 30,000 d. 50,000

Solution: The contribution of noncash assets to a partnership should be recorded based on


their fair value. In this case, the fair value of the land would be measured by its sales price on
the date of sale, P50,000

Problem 5 (ReSA)

On March 1, 2018, Coco and Martin formed a partnership with each contributing the following assets:

Coco Martin
Cash 300,000 700,000
Machinery and Equipment 250,000 750,000
Building - 2,250,000
Furniture and Fixtures 100,000 -

The building is subject to mortgage loan of 800,000 which is to be assumed by the partnership
agreement provides that Coco and Martin share profits and losses 30% and 70% respectively.
On March 1, 2018 the balance in Martin’s capital account should be:

a. 3,700,000 c. 3,050,000
b. 3,140,000 d. 2,900,000

Solution:
Cash 700,000.00
Machinery and Equipment 750,000.00
Building Total assets invested 3,700,000.00 2,250,000.00
Mortgage assumed (800,000.00)

Capital Balance of Martin 2,900,000.00

Problem 6 (PRTC)

Baser and Michelle have just formed a partnership. Baser contributed cash of P920,000 and
office equipment that costs P422,000. The equipment had been used in his sole proprietorship
and had been 70% depreciated. The current value of the equipment is P295,000. Baser also
contributed a note payable of P87,000 to be assumed by the partnership. The partners agreed
on a profit and loss ratio of 50% each. Baser is to have a 70% interest in the partnership.
Michelle contributed only a merchandise inventory from her sole proprietorship carried at
P550,000 on a first-in- first-out basis. The current fair value of the merchandise is P525,000.

To consummate the formation of the partnership Baser should make additional investment or
(withdrawal) of:

A. P224,000 C. P97,000
B. P(30,000) D. P(80,000)

Solution:

Michelle’s total contribution P 525,000


Interest Ratio 30%
Total Capital P 1,750,000
Baser’s Ratio 70%
Required capital of Baser P 1,225,000
Total contribution of Baser
(920,000+295,000-87,000) (1,128,000)
97,000

Problem 7 (PRTC)

In 2018, Norma and Celso agreed to form a new partnership under the following general
agreements:

Partners’ contributions will be on a %:4 ratio; (2) Profit and loss, 5:5, and (3) Capital credits
57:43 ratio, respectively to Norma and Celso. Their respective contributions will come from old
proprietorships they owned.
Norma contributed the following items and amounts:
Cash P 748,800
512,000 Equipment (at book value per her proprietorship records)

Celso contributed the following items at their carrying amounts in the proprietorship records:

Accounts receivable 96,000


Inventory 268,800
Furniture and fixtures 514,560
220,800
Intangibles

All the non-cash contributions are not properly valued. The two partners have agreed that (a)
P7,680 of the accounts receivable are uncollectible; (b) the inventories are overstated by
P19,200; (c) the furniture and fixtures are understated by P11,520; and the intangibles include
a patent with a carrying value of P13,440, which must now be derecognized upon a court order.
The rest of the intangible items are fairly valued.

1. How much is the total depreciable fixed asset recorded by the partnership?
a. P1,060,080 c. P1,116,480
b. P403,200 d. P1,041,480

2. What is the capital balance of Celso after the formation of the partnership?

a. 1,036,541 c. 1,325,808
b. 1,339, 225 d. 1,071,360
Solution:

1. D

Celso’s Contribution @ BV P 1,100,160


Net decrease to FV (28,800)
Celso’s Contribution @ FV P 1,071,360
Contribution Ratio 5/4
FV of Norma’s Contribution P 1,339,200
Cash of Norma (748,800)
FV equipment investment 590,400
FV of Furniture and Fixture 526,080
Total Fixed Assets P 1,116,480

2. A
Partner CC CNA Difference N

P 1,374,019 P 1,339,200 P 34,819

C 1,036,541 1,071,360 (34,819)

Total P 2,410,560 P 2,410,560 -0-

Problem 8 (PRTC)

A, B and C formed the ABC Partnership on July 1, 2018, with the following assets, measured at book
values in their respective records, contributed by each partner:

A B C

Cash P 200,000 P 150,000 P 150,000

Accounts receivable 38,500 68,900


Inventory 135,000 118,000 67,000

Plant, Property and Equipment (PPE) 950,000 460,000 380,000

A part of A’s contribution, P25,000, comes from his personal borrowings. Also, the PPE of A
and B are mortgaged with the bank for P160,000 and P16,500, respectively. The partnership
is to assume responsibility for these PPE mortgages. The fair value of the accounts receivable
contributed by C is P43,000 and her PPE at this date has a fair value P365,000. All the other
assets contributed are fairly valued. The partners have agreed to share profits and losses on a
5:3:2 ratio, to A, B and C, respectively.

How much is the contribution of each partner? Calculate their contribution ratio.

Solution:

A B C Total
Cash 200,000 150,000 150,000 500,000
Accounts Receivable 38,500 43,000 81,500
Inventory 135,000 118,000 67,000 320,000
PPE 950,000 460,000 365,000 1,775,000
Total Assets 1,285,000 766,500 625,000 2,676,500
Liabilities -160,000 -16,500 -176,500
Net Asset 1,125,000 750,000 625,000 2,500,000
Contribution
Net Assets
Ratio
A 1,125,000 45%
B 750,000 30%
C 625,000 25%

Total 2,500,000 100 %

What is the capital balance for each partner at July 1, instead, if the interest ratio is agreed at 4:3:3
to A, B and C, respectively?

Answer:

A 1,000,000 (2,500,000 x 40%)


B 750,000 (2,500,000 x 30%)
C 750,000 (2,500,000 x 30%)
Total 2,500,000

Problem 9 (PRTC)

Roberts and Smith drafted a partnership agreement that lists the following assets contributed at the
partnership’s formation:
Contributed by

Roberts Smith
Cash 20,000 30,000
Inventory 15,000
Building 40,000
Furniture & Equipment 15,000

The building is subject to a mortgage of P 10,000, which the partnership has assumed. The
partnership agreement also specifies that profits and losses are to be distributed evenly.
1. What amounts should be recorded as capital for Roberts and Smith at the formation of the
partnership?

Roberts Smith
A. 35,000 85,000
B. 35,000 75,000
C. 55,000 55,000
D. 60,000 60,000
Solution:
Roberts: 20,000 + 15,000 = P35, 000
Smith: 30,000 + 15,000 + 40,000 – 10,000 = P75,000

Problem 10 (PRTC)

The Grey and Redd Partnership was formed on January 2, 2010. Under the partnership
agreement, each partner has an equal initial capital balance. Partnership net income or loss is
allocated 60% to Grey and 40% to Redd. To form the partnership, Grey originally contributed
assets costing P30,000 with a fair value of P60,000 on January 2, 2010, and Redd contributed
P20,000 cash. Drawings by the partners during 2010 totaled P3, 000 by Grey an P9,000 by
Redd. The partnership net income in 2010 was P25,000

1. Under the goodwill method, what is Redd’s initial capital balance in the partnership?
A. 20,000 C. 40,000
B. 25,000 D. 60,000

Solution:
Contributed Capital Agreed Capital Increase (Decrease)

Grey 60,000 60,000

Redd 20,000 60,000 40,000

Total 80,000 120,000 40,000

Problem 11 (CRC-ACE)

On May 1, 2018, the business assets and liabilities of Nathan and Janice were as follows:

Nathan Janice
Cash 8,000.00 62,000.00
Receivables 200,000.00 600,000.00
Inventories 120,000.00 200,000.00
Land, Building and Equipment 650,000.00 535,000.00
Other Assets 2,000.00 3,000.00
Accounts Payable (180,000.00) (250,000.00)
Nathan and Janice agreed to from a partnership by contributing their net assets, subject to the
following adjustments:

• Receivables of P20,000 in Nathan’s books and P40,000 in Janice’s books are


uncollectible.
• Inventories of P6,000 and P7,000 in the respective books of Nathan and Janice are
worthless
• Other assets in both books are written off

Upon the partnerships formation:

Cash 8,000.00 62,000.00 1. The


Receivables 200,000.00 600,000.00 respective
capital of
Inventories 120,000.00 200,000.00
partners
Land Building and Equipment 650,000.00 535,000.00 Nathan and
Other Assets 2,000.00 3,000.00 Janice
Accounts Payable (180,000.00) (250,000.00) would
be_____________;
2. The total assets of the partnership would be_____________________.

Solution:
Nathan Janice

800,000.00 1,150,000.00
Uncollectible (20,000.00) (40,000.00)
Inventories (6,000.00) (7,000.00)
Written off (2,000.00) (3,000.00)
Total Capital 772,000.00 1,100,000.00

Problem 12 (CRC-ACE)

James admits Dani as a partner in business. Accounts in the ledger of James on June 1, 2018, just
before the admission of Dani, show the following balances:

Cash P26,000 Accounts Payable P264,000


Accounts Receivable 120,000 James, Capital 62,000 Merchandise Inventory
180,000
It is agreed that for purposes of establishing James’s interest, the following adjustments should be
made:

• An allowance for doubtful accounts of 2% of accounts receivable is to be established


• The merchandise inventory is to be valued at P202,000.
• Prepaid expenses of P6,500 and accrued expenses of P4,000 are to be established

Dani is to invest sufficient funds in order to receive a 1/3 interest in the partnership.

1. How much is the adjusted capital of James?


2. How much cash should Dani invest?
3. How much is the total assets of the partnership.

Solution:
Cash 26,000.00
A/R 120,000.00
Merchandise Inventory 180,000.00
A/P (264,000.00)
62,000.00
2% Allow. For doubtful acc. (2,400.00)
Merch. Inventory 22,000.00
Prepaid Exp. 6,500.00
Accrued Exp. (4,000.00)

James adjusted cap. 2/3 84,100.00


Dani 1/3 42,050.00
126 ,150.00
James Capital 84,100.00
Dani Capital 42,050.00
Accounts Payable 264,000.00
Accrued Expense 4,000.00 Total Assets
394,150.00

Problem 13 (CRC-ACE)

The balance sheet as of July 31, 2018, for the business owned by Ethan, shows the following
assets and liabilities:

Cash P100,000 Fixtures P328,000


Accounts Receivable 268,000 Accounts Payable 57,600
Merchandise 440,000

It is estimated that 5% of the receivable will prove uncollectible. The cash balance includes
1,000 share certificates of PNB at its cost, P8,000; the stock last sold on the market at P70.00
per share. Merchandise includes obsolete items costing P36,000 that will probably realize only
P8,000. Depreciation has never been recorded; the fixtures are 2 years old, have an estimated
life of 10 years, and would cost P480,000 if purchased new currently. Sundry prepaid items
amount to P10,000. ava is to be admitted as a partner upon investing P400,000 cash and
P200,000 merchandise.

1. What will be that total capital after the formation of the partnership?

Solution:

Cash 100,000.00
A/R 268,000.00
Merchandise 440,000.00
Fixtures 328,000.00
Accts. Payable (57,600.00)

Unadjusted Cap. 1,078,400.00


Uncollectible (13,000.00)
Share 62,000.00
Obsolete Merchandise (28,000.00)
Prepaid Items 10,000.00
Depreciation 56,000.00
Investment 600,000.00
Adjusted Capital 1,765,400.00

Problem 14 (CRC-ACE)

Harold and Cherry are partners sharing profits 60:40. A balance sheet prepared for the partnership
on April 1, 2018 shows the following:

Cash 48,000.00 Accounts Payable 89,000


Accounts Receivable 92,000.00 Harold, capital 133,000
Inventory 165,000.00 Cherry, capital 108,000
Equipment 70,000.00
Accumulated Depreciation (45,000.00)
Total Assets 330,000.00 330,000.00

On this date, the partners afree to admit lucas as a partner. The terms of the agreement is that
assets and liabilities are to be restated as follows:

• An allowance for possible uncollectible of P4,500 is to be established.


• Inventories are to be restated at their present replacement values of P170,000
Equipment are to be restated at a value of P35,000 Accrued expenses
of P4,000 are to be recognized.

Harold, Cherry, and Lucas will divide profits in the ratio of 5:3:2. Capital balances for the new
partners are to be in this ratio with Harold and Cherry making cash settlement outside of the
partnership for the required capital adjustment between themselves and Lucas investing cash
in the partnership for his interest.

Questions:

1. How much cash Lucas should contribute?

Solution:
H 136,900 17,787.5 154,687.5
C 110,600 (17,787.5) 92,812.5
L 61,875 - 61,875
309,375 0 309,375

Problem 15 (CRC-ACE)

Ferdinand and Daniel establish a partnership to operate a used-furniture business under the
name of F and D Furniture. Ferdinand contributes furniture that cost P60,000 and has a fair
value of P90,000. Daniel contributes P30,000 cash and delivery equipment that cost P40,000
and has a fair value of P30,000. the partners agree to share profits and losses 60% to Ferdinand
and 40% to Daniel.

1. Calculate the peso amount of inequity that will result if the initial noncash contributions of the
partners are recorded at cost rather than fair market value.

Solution:
F D T
Should be 90,000 60,000 150,000
Recovered 60,000 70,000 130,000
U&D 30,000 10,000 20,000
Oshi (12,000) (8,000) (20,000)
18,000 18,000 -0-

2019 1.2 Partnership Operations

Problem 1 (ReSA)

Left and Right are partners. Their capital accounts during 2019 were as follows:

Left, Capital Right, Capital

8/23 P 3,000 1/1 P15,000 3/5 P4,500 1/1 P 25,000


4/3 4,000 7/6 3,500
10/31 3,000 10/7 2,500

Partnership net income is P25,000 for the year. The partnership agreement provides for the
division of net income as follows:

• Each partner is credited 10 percent interest on his or her average capital (rounded to the
nearest month)
• Because of prior work experience of, Left is entitled to an annual salary of P6,000 and
Right is credited with P4,000
• Any remainder income or loss is to be allocated based on the beginning capital

How much of the partnership net income for 2019 should be assigned to Left and Right?

a. Left, P11,833.33; Right, P13,166.50 c. Left, P13,194; Right, P11,806


b. Left, P9,375; Right, P15,625 d. Left, P12,500; Right, P12,500

Solution:

Left Right Total

Interest 1,750.00 2,362.50 4,112.50


11,833.00 Salaries 6,000.00 4,000.00 10,000.00 13,166.50
Balance 4,083.00 6,804.00 10,887.50 25,000.00

Left Right

15,000 x 3 45,000.00 25,000 x 2 50,000.00


19,000 x 5 95,000.00 20,500 x 4 82,000.00
16,000 x 2 32,000.00 24,000 x 3 72,000.00
19,000 x 2 38,000.00 26,500 x 3 79,500.00

23 ,625.00

17 ,500.00
210,000.00 283,500.00
Divide: 12 12
Average Average
Capital Capital

Problem 2 (ReSA)

Hunt, Rob, Turman and Kelly own a publishing company that they operate as a partnership. The
partnership agreement includes the following:

• Hunt receives a salary of P10,000 and a bonus of 3% of income after all bonuses.
• Rob receives a salary of P5,000 and a bonus of 2% of income after all bonuses.
All partners are to receive 10% interest on their average capital balances.

The average capital balances are Hunt, P25,000; Rob, P22,500; Turman, P10,000 and Kelly,
P23,500. Any remaining profit and losses are to be allocated among the partners.

a. Hunt, P20,725; Rob, P14,975; Turman, P7,725; Kelly, P9,075


b. Hunt, P14,000; Rob, P8,250; Turman, P1,000; Kelly, P2,350
c. Hunt, P19,850; Rob, P14,600; Turman, P8,350; Kelly, P9,700
d. Cannot be determined.

Solution:

Hunt Rob Turman Kelly Total

Salary 10,000.00 5,000.00 - - 15,000.00


10% Interest 2,500.00 2,250.00 1,000.00 2,350.00 8,100.00
Bonus 1,500.00 1,000.00 - - 2,500.00
Balance :
Equally 6,725.00 6,725.00 6,725.00 6,725.00 26,900.00
20,725.00 14,975.00 7,725.00 9,075.00 52,500.00

Problem 3 (ReSA)

PP and QQ are partners operating a chain of retail stores. The partnership agreement provides for
the following:

PP QQ

Salaries ………………………………………………… P5,000 P2,500


Interest on capital balances ………………… 10% 10%
Bonus …………………………………………………… 20% of net income
before interest but
after bonus & salaries
Remainder ……………………………………………. 30% 70%
The income summary account for year 2019 shows a credit balance of P25,000 before any
deductions. Average capital balances for PP and QQ are P25,000 and P37,500, respectively.
The share of PP and QQ in the P25,500 net income would be:

a. PP, P12,031.25; QQ, P13,468.75 c. PP, P11,750; QQ, P13,750


b. PP, P13,275.75; QQ, P12,229.25 d. PP, P13,125; QQ, P12,375
Solution:
PP QQ Total

Salaries 5,000.00 2,500.00 7,500.00


13,125.00 10% Interest 2,500.00 3,750.00 6,250.00 12,375.00
Bonus 3,000.00 - 3,000.00 25,500.00
Balance 30%,
70% 2,625.00 6,125.00 8,750.00

Problem 4 (ReSA)

XX and YY formed a partnership on January 2, 2019 and agreed to share profits and loss in
the ratio of 90% and 10%, respectively. XX contributed capital of P6,250. YY contributed no
capital but has a specialized expertise and manages the firm full time. There were no
withdrawals during the year. The partnership agreement provides for the following:

• Capital accounts are to be credited annually with interest at 5% of the beginning capital
• YY is to be paid a salary of P250 a month
• YY is to receive a bonus of 20% of net income calculated before deducting his salary and
interest on both capital accounts
• Bonus, interest, and YY’s salary are to be considered as partnership expenses

The partnership’s income statement for 2019 follows:

Revenues …………………………………………………………………………… P24,112.50


Less: Expenses (including salary, interest, and bonus)…… 12,425.00
Net Income ………………………………………………………………………… 11,687.50
1. What is YY’s 2019 bonus?
a. P2,922.00 c. P3,750.00
b. P3,000.00 d. P3,934.50
2. How much is the total share of YY on the 2019 partnership net income?
a. P7,084.50 c. P7,918.75
b. P7,162.50 d. P8,097.00
Solution:
1. C

Net Income after salaries interest and


bonus 11,687.50
Salaries 3,000.00
Interest 312.50

Net income after bonus 15,000.00


Divide 80%
Net income before salaries, interest and bonus 18,750.00
20%
Bonus 3,750.00

2. C

XX YY Total
5% Interest 312.50 312.50
Salaries 3,000.00 3,000.00
Bonus
3,750.00 3,750.00
Balance 9:1
10,518.25 1,168.75 11,687.00
Share in Net
Income 10,830.75 7,918.75 18,749.50

Problem 5 (ReSA)

The Trading Company, a partnership, was formed on January 1, 2019, with four partners, DD,
EE, FF, and GG. Capital contributions were as follows: DD, P25,000; EE, P12,500; FF, P12,500;
GG, P10,000. The partnership agreement provides that partners shall receive 5% interest in
the amounts of their capital contributions. In addition, DD is to receive a salary of P2,500 and
EE a salary of P1,500. The agreement further provides that FF shall receive a minimum of
P1,250 per annum from the partnership and GG a minimum of P3,000 per annum, both
including amounts allowed as interest on capital and their respective shares of profits. The
balance of the profits is to be shared in the following proportions: DD, 30%; EE, 30%; FF,
20%; and GG, 20%. Calculate the amount that must be earned by the partnership during
2019, before any charges for interest on capital or partners’ salaries, in order that DD may
receive an aggregate of P6,250 including interest, salaries and share of profits.

a. P 8,333.33 c. P15,333.33
Solution:
b. P 15,000.00 DD EE FF GG Total
2,500.00 1,500.00 0.00 0.00 4,000.00
1,250.00 625.00 625.00 500.00 3,000.00
2,500.00 2,500.00 1,666.00 1,667.00 8,333.00
d. 6,250.00 4,625.00 2,291.00 2,167.00 15,333.00
833.00 833.00
P16,166.67
Salaries
Interest
Balance 3:3:2:2

3,000.00 16,166.00

Problem 6 (CRC-ACE)

David and Ruby organized the DR Partnership on January 1, 2018. the following entries were made
in their capital accounts during 2018:

Debit Credit

David, capital:
January 1 180,000.00
April 1 50,000.00
October 1 10,000.00

Ruby, capital
January 1 60,000.00
March 1 10,000.00
September 20,000.00
November 1 10,000.00

Required:
If the partnership net income, computed before salaries, interest and bonus is P56,000 for
2018, indicate its division between the partners under each of the following independent profit-
sharing agreements:
Solution:
a. Interest at 4% is allowed on average capital investments, and the balance is divided
equally.
b. A salary of P24,000 is to be credited to Ruby, 4% interest is allowed on each partner
on their ending capital balance, and the balance in the ratio of beginning capital
balances.
c. Salaries allowed to David and Ruby in the amounts of P34,000 and P38,000.
respectively, and remaining profits ad losses are divided in the ratio of average capital balances.
d. A bonus of 10% of partnership net income is credited to David, a salary of P16,000
is allowed to Ruby, and remaining profits and losses are shared equally. (The bonus
is regarded as an expense for purposes of calculating the bonus amount).
DAVID RUBY
43,101 43,101
180,000 *12/12 180,000 60,000 *12/12 60,000
43,191 43,160
(50,000) *9/12 (37,500) (10,000) *10/12 8,333
43,374 43,344
(10,000) *3/12 (2,500) 20,000 *4/12 6,667
43,405
120,000 140,000 10,000 *2/12 1,666
AVE.
80,000 CAP 60,000
A. DAVID RUBY TOTAL C. DAVID RUBY TOTAL

INTEREST 5,600 2,400 8,000 SALARIES 34,000 38,000 72,000

BALANCE 24,000 24,000 48,000 BALANCE (11,200) (4,800) (16,000)

29,600 26,400 56,000 22,800 33,200 56,000


D. DAVID RUBY TOTAL

B. DAVID RUBY TOTAL SALARY 16,000 16,000

SALARIES 24,000 24,000 BONUS 5,091 5,091

INTEREST 4,800 3,200 8,000 BALANCE 17,455 17,455 34,909

BALANCE 18,000 6,000 24,000 22,546 33,455 56,000

56,000

Problem 7 (CRC-ACE)

X,Y and Z, doctors, agree to form a partnership and to share profits in the ratio 5:3:2. they also
agreed that Z is to be allowed a salary of P140,000 and that Y is to be guaranteed P105,000
higher as his share of the profits. During the first year of operations, income from fees are
P900,000, while expenses total P480,000.
How much of the profits should be credited to X?, to Y? to Z?

Solution:

X Y Z TOTAL

SALARY 140,000 140,000

BALANCE 140,000 84,000 56,000 280,000

140,000 84,000 196,000 420,000

(15,000) 21,000 (6,000)

PROFIT 125,000 105,000 190,000 420,000

Problem 8 (CRC-ACE)

Partners L and M share profits 3:1 after annual salary allowances of P400,000 and P60,000,
respectively; however, if profits are not adequate to meet the salary allowances, the entire profit
is to be divided in the salary ratio. Profits of P90,000 were reported for the year 2018. in 2019
it is ascertained that in calculating net income for the year ended December 31, 2018,
depreciation was overstated by P36,000 and ending inventory was understated by P80,000.
What adjustments should be made on the capital of L and M?
Adjusting entry needed to correct the partner’s capital balances.

Solution:

L M TOTAL

40,000 60,000 100,000

25,500 8,500 34,000

SHOULD BE 65,500 68,500 134,000

MADE (36,000) (54,000) (90,000)

ADJUSTMENTS
29,500 14,500 44,000

ENTRIES: MDSE 8,000


AD 36,000

L 29,500

M 14,500
Problem 9 (CRC-ACE)

NEGOSYO TO Company a partnership was formed on January 1, 2018, with four partners, C, P,
A and S. Capital contributions were as follows: C- P1,000,000; PP500,000; A- P500,000; and S-
P400,000. the partnership agreement provides that each partner shall receive 5%interest on the
amount of his capital contribution. In addition, C is to receive a salary of P100,000 and P a salary
of P60,000 which are to be charged as expenses of the business. The agreement provides that
A shall receive a minimum of P50,000 per annum from the partnership and S a minimum of
P120,000 per annum, both including amounts allowed as interest on capital and their respective
shares of profits. The balance of the profits to be shared in the following proportions: C- 30%;
P- 30% A- 20% and S-20%.

Calculate the amount that must be earned by the partnership during 2018, before any charge
for interest on capital or partners ‘ salaries, in order that C may receive an aggregate of
P250,000, including interest, salary and share of profits.

Solution:

C P A S TOTAL

INTEREST 50,000 25,000 25,000 20,000 120,000

SALARIES 100,000 60,000 160,000

BALANCE 100,000 100,000 66,667 66,666 333,333

250,000 185,000 91,667 86,666 613,333

33,334 33,334

250,000 185,000 91,667 120,000 646,667

50,000 25,000 25,000 20,000 120,000

Problem 10 (CRC-ACE)

The following account balances appear in the ledger for the firm of X and Y at the end of 2018 before
the profit for the year has been transferred to the partner’s accounts:
X, drawing 72,000.00
Y, drawing 125,000.00

X, loan 175,000.00

X, capital 500,000.00
Y, capital 500,000.00

Profit and loss 302,250.00

The following information is to be considered in closing the profit and loss account and the drawing
accounts:
• The cost of installing equipment at the beginning of 2018, P27,000, was charged to
expense. The installation relates to equipment with a 10-year life.
• The loan to the firm was made by X on March 1, 2018. No entry has been made for
interest on the loan, which is 6% and is to be paid to X at the time the loan is
prepaid.
The partnership agreement permits X and Y to withdraw weekly sums of P1,500 and P2,250,
respectively, these amounts to be regarded as salaries. Actual withdrawals by partners differed
from allowed amounts and are summarized in the drawing accounts.

Y, the managing partner, is entitled to special bonus of 25% of the net profit after deduction of
all special allowances to partners (including the bonus), and any remaining profit is to be
distributed equally.

1. How much should be the Dec. 31 ending capital balance of each partner?

Solution:

X Y TOTAL PROFIT AND LOSS 302,500

SALARIES 78,000 117,000 195,000 INSTALLATION 27,000


-
B 24,610 24,610 ACCUP. DEP. 2,700

BALANCE 49,220 49,220 98,440 326,800


-
127,220 198,830 318,050 INTEREST 8,750
- -
DRAWING 72,000 125,000 ADJUSTED PROFIT 318,050

55,220 65,830

CAPITAL 500,000 500,000


555,220 565,830

Problem 11 (PRTC)
Linda and Mario created a partnership to own and operate a health-food store. The partnership
agreement provided that Linda receives an annual salary of P10,000 and Mario a salary of P5,000
to recognize their relative time spent in operating the store. Remaining profits and losses were
divided 60:40 to Linda and Mario, respectively. Income of P13,000 for 2017, the first year of
operations, was allocated P8,800 to Linda and P4,200 to Mario. On January 1, 2018, the
partnership agreement was changed to reflect the fact that Mario could no longer devote any
time to the store’s operations. The new agreement allows Linda a salary of P18,000, and the
remaining profits and losses are divided equally. In 2018, an error was discovered such that
2017 reported income was understated by P4,000. The partnership income of P25,000 for 2018
included this P4,000 related to 2017.
1. In the reported new income of P25,000 for the year 2018, Linda would have
A. P21,900 B. P17,100
B. P0 D. P12,500
Solution:
2018 income to allocate (25,000-4000=21,000)

Linda Mario Total


Salary 18,000 18,000
Remainder to divide
1,500 1,500 3,000
income
2017
2,400 1,600 4,000
understatement
21,900 3,100

Problem 12 (PRTC)

Derha, a senior partner in a law firm, has a 30% participation in the firm’s profit and losses.
During 2018, Derha withdrew P130,000 against her capital but contributed property with a fair
value of P25,000. Derha’s capital increased by P15,000 during 2018.
2. The net income of the partnership for 2018 is
A. P150,000 C. P.350,000
B. P400,000 D. P550,000

Solution:
Increase in Capital P 15,000
Contributed Property (25,000)
Withdrawal 130,000
Share in Net Income 120,000
Ratio 30%
Net Income of Partnership 400,000

Problem 13 (PRTC)

Elmo, Fred and Greg invest P40,000, P30,000 and P25,000 respectively, in a partnership on June
30, 2017. They agree to divide net income or loss as follows:
A. Interest at 10% on beginning capital account balances
B. Salaries of P10,000, P8,000 and P6,000, respectively to Elmo, Fred and Greg, respectively.
C. Remaining net income or loss is divided equally
D. A minimum of P18,000 of income is guaranteed to Greg regardless of the result of operations.
3. If the net income for the year ended June 30, 2018 before interest and salaries allowances to
partners was P44,000, the amount of the net income credited to Elmo is:

A. P21,875 C. P18,334
B. P20,000 D. P14,500

Solution:

Elmo Fred Greg Total


Interest 4,000 3,000 2,500 9,500
Salaries 10,000 8,000 6,000 24,000
Unallocated 3,500 3,500 3,500 10,500
Unadjusted
17,500 14,500 12,000 44,000
share
Guarantee to
(3,000) (3,000) 6,000 -
Greg
Net Income 14,500 11,500 18,000 44,000

Problem 14 (PRTC)

X, Y and Z are partners with average capital balances during 2018 of P120,000, P60,000 and
P40,000, respectively. Partners receive 10% interest on their average capital balances. After
deducting salaries of P30,000 to X and P20,000 to Y, the residual profit or loss is divided equally.
In 2018 the partnership sustained a P33,000 loss before interest and salaries to partners.
4. By what amount should X’s capital account change?
A. P7,000 increase C. P11,000 decrease
B. P35,000 decrease D. P42,000 increase

Solution:
X Y Z Total
Interest 12,000 6,000 4,000 22,000
Salaries 30,000 20,000 - 50,000
Unallocated (35,000) (35,000) (35,000) (105,000)
Total 7,000 (9,000) (31,000) (33,000)

Problem 15 (PRTC)

Partners Joyce and Marie share profits 3:1 after annual salary allowances of P4,000 and P6,000
respectively; however, if profits are not adequate to meet the salary allowances, the entire profit
is to be divided in the salary ratio. Profits of P9,000 were reported for the year 2017. in 2018, it
is ascertained that in calculating net income for the year ended December 31, 2017, depreciation
was overstated by P3,600 and the ending inventory was understated by P800.
5. The amount of the net adjustments in the books of Joyce and Marie are:
Joyce Marie
A P(3,699) P(1,813)
B P2,950 P1,450
C P8,188 P8,563
D P2,300 P3,475

Solution:
2017 Net Income = 9,000
Joyce Marie Total
4:6 Ratio 3,600 5,400 9,000
2017 corrected Net Income = 9,000 + 3,600 + 800 = 13,400
Salaries 4,000 6,000 10,000
Unallocated (3:1) 2,550 850 3,400

Total 6,550 6,850 13,400


Ratio (3600.00) (5400.00)

Distribution 2,950 1,450

2020 1.3 Partnership Dissolution

Problem 1 (ReSA)

A partnership had the following condensed balance sheet:

Assets Liabilities and Capital

Cash 2,500.00 Liabilities 7,500.00


Noncash Assets 32,500.00 XX Capital (80%) 20,000.00
XX Loan 2,500.00 YY Capital (20%) 10,000.00
Total 37,500.00 37,500.00
The percentages in parentheses after the partner's capital balances represent their respective
interests in profits and losses. The partners agree admit ZZ as a member of the firm.

1. ZZ purchases a ¼ interest in the firm. One fourth of each partner's capital is to be transferred
to the new partner. ZZ pays the partners which is divided between them in proportion to the
equities given up. The capital balances of XX, YY, and ZZ after should be:

XX YY ZZ XX YY ZZ
a. 15,000 7,500 9,375 c. 15,000 7,500 7,500
b. 12,500 12,500 12,500 d. 10,000 10,000 10,000

Solution:
XX (20,000 x 3/4) 15,000.00
YY (10,000 x 3/4) 7,500.00
ZZ (30,000 x 1/4) 7,500.00
30,000.00

Problem 2 (ReSA)

WW desires to purchase a one-fourth capital and profit and loss interest in the partnership of EE,
GG, DD. The three partners agree to sell WW a one fourth of their respective capital and profit
and loss interest in exchange for a total payment of 40,000. The capital accounts and the
respective EE, capital (60%) 80,000.00 percentage
interest in profits GG, capital (30%) 40,000.00 and losses
immediately DD, capital (10%) 20,000.00 before the
sale to WW are:

Total 140,000.00

All other assets and liabilities are fairly valued and with no adjustments is to be recorded prior
to the acquisition by WW immediately after WW’s acquisition, what would be the capital balances
of EE, GG and DD respectively?

a. 60,000; 30,000; 15,000 c. 77,000; 38,500; 105,000.00


19,500
b. 69,000; 34,500; 16,500 d. 92,000; 46,000;
22,000

Solution: Problem 3
(ReSA)
EE (80,000 x 3/4) 60,000.00
GG (40,000 x 3/4) 30,000.00
The DD (20,000 x 3/4) 15,000.00 following
condensed
balance sheet is presented for the partnership of AA and BB who share profit and losses in the ratio
of 6:4 Cash 33,750.00 respectively:
Other Asset 468,750.00
525,000.00 BB, loan 22,500.00
Accounts Payable 90,000.00
AA, capital 261,000.00
BB, capital 174,000.00
525,000.00

The assets and liabilities are fairly valued on the balance sheet. AA and BB decide to admit CC
as a new partner with 20% interest. No bonus or goodwill is to be recorded. What amount should
CC contribute or invest in cash and other assets?

a. 82,500 c. 105,000
b. 87,000 d. 108,750

Solution:

AA, capital 261,000.00


BB, capital 174,000.00

Total 435,000.00
Divide: 80%
Total Agreed Capital 543,750.00
108,750.00 CC's interest 20%

Problem 4 (ReSA)

XX and YY are partners who have capital balances of 300,000 and 240,000 sharing profits in the
ratio of 3:2. ZZ is admitted as a partner upon investing 250,000 for a 25% interest in the firm,
profits are to be allocated equally. Given the choice between goodwill and bonus method, ZZ
will:

a. Prefer bonus method due to ZZ’s gain of 17,500


b. Prefer bonus method due to ZZ’s gain of 70,000
c. Prefer goodwill method due to ZZ’s gain of 70,000
d. Be indifferent for the goodwill and bonus methods are the same

Solution:

Bonus Method:
CC AC Bonus
XX 300,000.00 331,500.00 31,500.00 3/5
YY 240,000.00 261,000.00 21,000.00 2/5

540,000.00 592,500.00 52,500.00


ZZ 250,000.00 197,500.00 (52,500.00)

790,000.00 790,000.00 -

Goodwill Method:
CC AC Goodwill
XX 300,000.00 426,000.00 126,000.00
YY 240,000.00 324,000.00 84,000.00

540,000.00 750,000.00 210,000.00


ZZ 250,000.00 250,000.00 -

,000.00 1,000,000.00 210,000.00

For purposes of comparing bonus and goodwill, goodwill is assumed not realized and it should be written
off outright as a loss, therefore:

XX YY ZZ
Capital balance if Goodwill method is
used 426,000.00 324,000.00 250,000.00
Less: write off of Goodwill (equally) (70,000.00) (70,000.00) (70,000.00)
Capital balance after write off of goodwill
356,000.00 254,000.00 180,000.00
Capital balance of Bonus method is used
331,500.00 261,000.00 197,500.00
Gain (loss) if Bonus method is used
24,500.00 (7,000.00) 17,500.00

Problem 5 (ReSA)
ASSETS LIABILITES AND CAPITAL
Cash 60,000.00 Accounts Payable 4,000.00
Inventories 40,000.00 FF, loan 3,000.00
Furn. & Fixt (net) 50,000.00 DD, capital 70,000.00
Patent 15,000.00 EE, capital 60,000.00
FF, capital 30,000.00
FF, drawings (2,000.00)
DD, EE and FF are partners sharing profits and losses of 50%, 30% and 20% respectively. The
December 31, 2019 balance sheet of the partnership before any profit allocation was summarized
as follows:

Total Assets 165,000.00 Total Liabilities and Capital 165,000.00

The partnership net income for the year amounted to 30,000. On January 1, 2020, FF has decided
to retire from the partnership and by mutual agreement among partners; the following have
been arrived at:

a. Inventories amounting to 5,000 is considered obsolete and must be written off


b. Furniture and fixtures should be adjusted to their current value of 65,000
c. Patents are considered worthless and must be written off immediately before the
retirement of FF

It was agreed that the partners will pay FF for his interest in the partnership inclusive of loan balance

1. The interest of FF immediately before his retirement amounted to:


a. 37,000 c. 35,000
b. 36,000 d. 24,000

2. FF retires by receiving 36,000 cash payment at book value, the capital balances of
DD and EE after the retirement of FF:
a. DD, 82,500; EE, 67,500 c. DD, 67,500; EE, 58,500
b. DD, 85,000; EE, 69,000 d. DD, 57,500’ EE, 52,500

3. FF retires by receiving 38,000 cash (payment at more than book value), using
bonus method, the capital balances of DD and EE after the retirement of FF: a. DD,
81,250; EE, 66,750 c. DD, 81,875; EE, 67,125
b. DD, 83,750; EE, 68,250 d. DD, 82,500; EE, 67,500

4. FF retires by receiving 38,000 cash (payment at more than book value), using total
implied goodwill method, the capital balances of DD and EE after the retirement of
FF:
a. DD, 87,500; EE, 70,500 c. DD, 81,875; EE, 67,125
b. DD, 83,750; EE, 68,250 d. DD, 82,500; EE, 67,500

Solution:

1. B
2. A

Unadjusted Capital 70,000.00 60,000.00 30,000.00


Share in Net Income 15,000.00 9,000.00 6,000.00
DD EE FF

Inventories written off (2,500.00) (1,500.00) (1,000.00)


Furniture and Fixtures 7,500.00 4,500.00 3,000.00
Patent (7,500.00) (4,500.00) (3,000.00)

Total 85,000.00 69,000.00 36,000.00

Adjusted Capital 82,500.00 67,500.00 35,000.00

3. A

Bonus Method (38,000-36,000) 2,000


DD EE
Capital 82,500.00 67,500.00
(2,000 x 5/8)
(1,250.00)
(2,000 x 3/8)
(750.00)
Capital Balances
81,250.00 66,750.00

4. A

DD EE
Capital 82,500.00 67,500.00
(10,000 x 50%) 5,000.00
(10,000 x 30%)
3,000.00
Capital Balances 87,500.00 70,500.00

Problem 6 (PRTC)

The capital accounts of the Sarah and Opel partnership on January 1, 2018 were:

Sarah, Capital (75% profit percentage) P 140,000


Opel, Capital (25% profit percentge) 60,000

Total Capital P 200,000

On October 1, Tina was admitted for a 40% interest in the partnership when she purchased 40%
of each existing partner’s capital for P100,000, paid directly to Sarah and Opel. The partnership’s
net income for the year is P82,500 and 2/3 of it was earned in the last quarter of the year.

1. What are the capital balances of Sarah, Opel and Tina after Tina’s admission to the partnership?
A. P105,000; P45,000; P100,000
B. P135,875; P55,313; P127,500
C. P96,375; P40,125; P91,000
D. P112,500; P50,000; P87,500

Solution:

Sarah Opel Tina Total


Beginning 140,000 60,000 200,000
Balance
Net Income
20,625 6,875 27,500
(27,500)

Adjusted Net
160,625 66,875 227,500
Income
Purchase Interest (64,250) (26,750) 91,000

Ending Balance 96,375 40,125 91,000 227,500

2. How much will Sarah receive from the above transaction?


A. P71,000 C. P86,250
B. P92,500 D. P118,750
Solution:
Sarah 64,250 6,750 71,000
Opel 26,750 2,250 29,000

Total 91,000 9,000 100,000


A. P68,750; (P6,250) C. P89,063; P5,313
B. P79,063; (P13,125) D. P59,125;(P7,750)
Solution:

Interest Ratio
Partner TAC CAN Difference 3. Assume
Tina is
Sarah 137,375 160,625 (23,250)
admitted by
Opel 59,125 66,875 (7,750) investing the
40% Tina 131,000 100,000 31,000 P100,000
into the
partnership for a 40% interest, how much is the ending capital balance of Opel after admission
and the bonus (given)/received to/from Tina?

Total 3,275,000 327,500 0

Problem 7 (PRTC)

The balance sheet at December 31, 2018, for the Beth, Daisy and Maya partnership is summarized as
follows:
Daisy is retiring from the partnership. The partners agreed that the partnership assets,

Assets P 1,000,000 Liabilities P 250,000


Loan to Daisy 125,000 Beth Capital (50%) 375,000

Daisy Capital (40%) 375,000


Maya Capital (10%) 125,000

Total P 1,125,000 Total P 1,1125,000


excluding Daisy’s loan, should be adjusted to their fair market value of P1,250,000 and that Daisy
should receive P380,000 for her capital balance net of the P125,000 loan.

How much is the capital balance of Beth and Maya immediately after Daisy’s retirement.

A. P475,000; P145,000 C. P481,250; P146,250


B. P500,000; P150,000 D. P385,416; P127,084

Adjusted interest 500,000 Total 475,000 150,000 1,125,000


Cash paid to
Solution:

Beth Daisy Maya Total


Beg. Balance 375,000 375,000 125,000 875,000
Adjusment 125,000 100,000 25,000 250,000
Daisy -505,000 -505,000
Bonus -25,000 30,000 -5,000 0

End. Balance 475,000 0 145,000 620,000

Problem 8 (PRTC)

On January 2, 2018, Lexy and Ace dissolve their partnership and transfer all assets and liabilities
to a newly formed corporation. At the date of incorporation, the fair value of the net assets was
P22,500 more than the carrying amount on the partnership’s books. Of which P12,500 was
assigned to tangible assets and P10,000 was assigned to patent. Lexy and Ace were each issued
5,000 shares of the corporations P12.50 par common stock.
5. Immediately following incorporation, additional paid-in capital in excess of par should be credited
for
A. P160,000 C. P25,000
B. P47,500 D. P137,500

Solution:
FV of Net Assets (150,000+22,500) 172,500
PV of shares issued (10,000x12.5) (125,000)

APIC 47,500

Problem 9 (PRTC)

On June 30, 2017, the balance sheet for the partnership of D, E and F, together with their respective
profit and loss ratios, is summarized as follows:

Assets, at cost P 375,000 D, Loan P 18,750

D, Capital (20%) 87,500

E, Capital (20%) 81,250

F, Capital (60%) 187,500


Total P 375,000

D has decided to retire from the partnership, and by mutual agreement the assets are to be
adjusted to their fair value of P450,000 at June 30, 2018. It is agreed that the partnership will
pay D P127,500 cash for his partnership interest exclusive of his loan, which is to be repaid in
full.

1. After D’s retirement, what are the capital account balances of partners E and F, respectively?
A. P81,250 and P187,500 C. P121,250 and P307,500
B. P90,000 and P213,750 D. P96,250 and P232,500

Solution:

D E F Total
Beg. Balance 87,500 81,250 187,500 356,250
Adjustment 15,000 15,000 45,000 75,000
Adjusted Balance 102,500 96,250 232,500 431,250
Cash Paid (127,500) (127,500)
Bonus 25,000 (6,250) (18,750)

90,000 213,750 303,750

Problem 10 (PRTC)

Partners Boba and Tess, who share profits and losses equally, have decided to incorporate the
partnership at December 31, 2018. The partnership net assets after the following adjustments
will be contributed in exchange for share of stocks from the corporation.

I. Provision of allowance for doubtful accounts, P6,250.


II. Adjustment of overstated equipment by P2,500
III. Adjustment of understated inventory by P13,750 and IV. Recognition of additional
depreciation of P5,000.

The corporation’s ordinary share is to have a par value of P250 each and the partners are to be
issued corresponding shares equivalent to 80% of their adjusted capital balances.
The partnership balance sheet at December 31, 2018 follows:

Cash P 112,500 Liabilities P 107,500


Accounts Receivable 62,500 Accounts Payable 5,000
Inventory 87,500 Boba, Capital 106,250
Equipment 50,000 Tess, Capital 93,750
Total P 312,500 Total P 312,500

1. Determine the total credit to APIC upon incorporation of partnership


A. P61,875 C. P40,000
B. P144,375 D. P140,000

Solution:
BV of Net Asset (106,250 + 93, 750) 200,000
Net Adjustment -
FV 200,000
PV of Shares (200,000 x 80%)

Total credit of APIC for the excess of credits

2. The number of ordinary shares issued to Partner Tess is


A. 210 C. 238
B. 300 D. 217

Solution:
Tess' FV contribution 93,750
80%

PV of share issued to Tess 75,000


Number of shares received by Tess /250
300

Problem 11 (CRC-ACE)

Capital balances and profit sharing percentages for the partnership of Aaron, Nimrod, and Elijah
on January 1,2018 are as follows:

Aaron (36%) P140,000


Nimrod (24%) 100,000 Elijah (40%) 160,000
On January 2,2018 the partners agree to admit Ruth in the partnership for a 25% interest in
capital and earnings for her investment in the partnership of P120,000. Partnership are not to be
revalued.

a. The capital balance of Aaron, Nimrod, Elijah and Ruth, immediately after the admission
of Ruth would be:
b. What will be new profit and loss ratio for Aaron, Nimrod, Elijah, and Ruth, if old partners
will share profits using the old ratio?

Solution:

TAC B

A 140,000 (36,000) 136,400 27%


N 100,000 (2,400) 97,600 18%

E 160,000 (4,000) 156,000 30%

R 120,000 (10,000) 130,000 25%

T 520,000 0 520,000 100%

Problem 12 (CRC-ACE)

The balance sheet of Dylan and Samuel Partnership at December 31, 218, appears below:

Assets: Liabilities:
Cash P15,000 Accounts Payable
P35,000
Accounts Receivable (net) 45,000 Notes Payable
25,000
Inventories 75,000 Accrued Liabilities
40,000
PPE (net) 225,000 Mortgage Payable
110,000
Dylan, Capital
60,000
Samuel. Capital
90,000
P360,000
P360,000
Determine the capital balances of partners immediately after the admission of Sebastian under the ff.
independent situations:
a. Sebastian acquired 25% interest in the partnership capital directly from Dylan
and Samuel for P50,000. Sebastian paid P18,750 directly from Dylan and
P31,250 directly to Samuel. Total Assets of the partnership after the admission
of Sebastian were P360,000. How much must be the capital balance of Dylan
immediately after the admission of Sebastian.
b. Assume the same facts as in a except that total assets of the partnership were
P410,000 after the admission of Sebastian. At January 1,2019, inventories had
a fair value of P85,000, while PPE (net) had a fair value of P265,000. Both Dylan
and Samuel decided to revalue the partnership’s assets before the admission of
Sebastian. Determine the capital balance of Samuel immediately after the
admission of Sebastian
c. Sebastian acquired a 25% interest in capital by investing P50,000 of cash into
the partnership. Total capital of the Dylan-Samuel-Sebastian Partnership on
January 1,2019, amounted to P200,000. Determine the capital balance of
Sebastian immediately after his admission
d. Sebastian acquired 25% interest in capital by investing P80,000 of cash into the
partnership. Total capital of the Dylan-Samuel-Sebastian Partnership after
Sebastian’s admission amounted to P320,000. The fair value of the inventories was P85,000
and the fair value of the PPE (net) was P305,000 on January 1,2019. Determine the capital
balance of Dylan, Samuel and Sebastian immediately after Sebastian’s admission.

Solution:

B
D 60,000 15,000 3,750
S 90,000 22,500 8,750 S 50,000
200,000

A. D S TOTAL
CAPITAL SOLD 13,750 23,750 37,500
GAIN/BONUS 5,000 7,500 12,500
SELLING PRICE 18,750 31,250 50,000

60,000+3,750= 46,250

B. D D SEB TOTAL

SEB.CAP 50,000 60,000 90,000 0 150,000


REVAL. TAC 200,000 20,000 30,000 0 50,000
TCB
VAL. 150,000 80,000 120,000 0 200,000
400,000 18,750 31,250 50,000 200,000
61,250 C.
TCC
D 60,000 60,000
S 90,000 90,000
S 50,000 50,000
200,000 200,000
BONUS METHOD

D.
D 60,000 36,000 96,000
S 90,000 54,000 144,000
S 80,000 80,000
230,000 90,000 320,000
Problem 13 (CRC-ACE)

1. A, B and C have capital balances of P112,000, P130,000 and P58,000, respectively and share
profits in the ratio 3:2:1. D invest cash in the partnership for a ¼ interest.
a. D receives a ¼ interest in the assets of the partnership, which includes credit
for 25,000 of goodwill that is recognized upon admission. How much cash D
invest?
b. D receives a ¼ interest in the assets of the partnership and B is credited with
P15,000 of the bonus from D, how much cash D invest?

Solution:

A. TCC
A 112,000
B 130,000
C 58,000 300,000
D 75,000 25,000 100,000
375,000 25,000 400,000

B.
A 112,000 22,500
B 130,000 15,000
C 58,000 7,500
-
D 160,000 45,000 115,000
460,000 0 460,000

Problem 14 (CRC-ACE)

L, M and M are partners sharing profits in the ratio of 3:2:1, respectively. Capital accounts are
P500,000. P300,000 and P200,000 on December 31,2018, when N decides to withdraw. It is
agreed to pay P300,000 for N’s interest. Profits after the withdrawal of N are to be shared
equally.

Questions:
a. Using the bonus approach, how much are the capital balances of L and M after
N’s withdrawal?
b. Using the goodwill approach, how much are the capital balances of L and M
after N’s withdrawal?
Solution:

A.
L M
CAP 440,000 260,000

N CAP. 200,000
L 60,000
M 40,000
CASH 300,000

B.
VALUATION L M
600,000 800,000 500,000

ASSET 600,000
CL 300,000
M 200,000
N 100,000

Problem 15 (CRC-ACE)
O, P and Q share profits in the ratio of 5:3:2, Q is permitted to withdraw from the firm on
December 31, 2018. Profits after withdrawal of Q are to be shared 3:2. The partnership balance
sheet on this date is as follows:

Receivable from Q P10,000 Liabilities P80,000


Goodwill 80,000 Payable to P 30,000
Other Assets 190,000 O, capital 70,000
P, capital 60,000
Q, capital 40,000
280,000 280,000
a. Assuming that Q is paid P44,000 in full settlement of the capital interest and
P10,000 claim balance, using the bonus method of recording the withdrawal of
Q, how much are the capital balances of O and P after Q’s withdrawal?
b. Using the data in question A, using the goodwill method of recording the
withdrawal of Q, how much are the capital balances of O and P after Q’s
withdrawal?

Solution:

A.
PAID 44,000 CAP. -30,000
BONUS 14,000

14,000*5/8 8,750-70,000= 61,250 O


5,250-60,000= 54,750 P

Q CAP 40,000 O CAP 8,750


P CAP 5,250
R'BLE OF Q 10,000
CASH 10,000

B. 44,000
-30,000
14,000 SHARE OF Q IN VALUATION

/2
70,000
2021 1.4 Partnership Liquidation

Problem 1 (ReSA)

On December 31, 2019, the accounting record of MM, NN, OO Partnership (a general partnership)
included the following ledger account balances:

(Dr.) Cr.

MM, drawing (15,000.00)


OO, drawing (5,625.00)
NN, loan 18,750.00
MM, capital 76,875.00
NN, capital 62,812.50
OO, capital 67,500.00

Total assets of the partnership amounted to P299,062.50 including P32,812.50 cash and
partnership liabilities totalled, P93,750. The partnership was liquidated on December 31, 2019
and OO received P52,031.25 cash pursuant to the liquidation. MM, NN and OO shared net income
and losses in a 5:3:2 ratio, respectively.

1. The loss on realization


a. 9,843.75 c. 49,218.75
b. 15,468.75 d. 77,343.50
2. The amount realized from sale of non-cash assets?
a. 160,781.25 c. 217,031.25
b. 188,906.25 d. 266,250.00
3. The cash balance after payment of liabilities?
a. 156,093.75 c. 221,718.75
b. 193,593.75 c. 249,843.75

Solution:

1. C Cash Proceeds 217,031.25


2. C Book Value of Asset (266,250.00)
3. A (49,218.75)

Cash Other Asset Liabilities MM NN OO

32,812.50 266,250.00 93,750.00 61,875.00 81,562.50 61,875.00


2) 217,031.25 266,250.00 9,843.75

,843.75 - 93,750.00 52,031.25


(93,750.00) (93,750.00)

3) 156,093.75 -

Problem 2 (ReSA)

Fleming, Durano and Mart are partners in a wholesale business. On January 1, 2019 the total capital
was P30,00 and drawings presented as follows:

Capitals Drawings

Fleming 6,250.00 3,750.00


Durano 5,000.00 2,500.00
Mart 18,750.00 1,250.00

Partners agree that profit and loss ratio are shared equally. Because of the failure of some
debtors to pay their outstanding accounts, the partnership loses heavily and is compelled to
liquidate. After exhausting the partnership assets, including those arising from an operating
profit of P4,500 in 2019, they still owe P5,250 to creditors on December 31, 2019. Fleming has
no personal but the others are well off.

1. The partnership liquidation loss:


a. None c. 27,750
b. 10,000 d. 32,250
2. The amount to be received by Mart as a result of the liquidation:
a. 818.75 c. 7,125
b. 4,875 d. 9,750

Solution:
Asset Liabilities
Capital 5,250.00 22,500.00
Profit 4,500.00

Liquidation
Loss 5,250.00 32,250.00 27,000.00

Fleming Durano Mart

Capital 6,250.00 5,000.00 18,750.00


Drawings (3,750.00) (2,500.00) (1,250.00)

2,500.00 2,500.00 17,500.00


Profit 1,500.00 1,500.00 1,500.00
Loss on Realization (10,750.00) (10,750.00) (10,750.00)

(6,750.00) (6,750.00) 8,250.00


6,750.00 (3,375.00) (3,375.00)

- (10,125.00) 4,875.00

Problem 3 (ReSA)

Following is the balance sheet of DD, EE and FF partnership (a general partnership) on June 4, 2019
immediately prior to its liquidation:

Assets Liabilities and Capital


Cash 6,000.00 Liabilities 20,000.00
Other Asset 94,000.00 EE, loan 4,000.00
DD, capital 27,000.00
EE, capital 39,000.00
FF, capital 10,000.00

100,000.00 Total 100,000.00

The partners shared net income and losses as follows: DD, 40%; EE, 40% and FF, 20%. On June
4, 2019, the other cash were realized at P30,700 and P20,500 had to be paid to liquidate the
liabilities because of an unrecorded trade accounts payable of P500. DD and EE were solvent,
but FF’s personal liabilities exceeded personal assets by P5,000. How much would each partner
receive?
a. DD, 1,680; EE, 17,680; FF, 0
b. DD, 1,480; EE, 17,480; FF, 0
c. DD, 100; EE, 12,100; FF, 0
d. DD, 100; EE, 16,100; FF, 0

Solution:

40% 40% 20%


DD EE FF Total
27,000.00 43,000.00 10,000.00 80,000.00
(25,520.00) (25,520.00) (12,760.00) 63,800.00
1,480.00 17,480.00 (2,760.00) 16,200.00 (1,380.00) (1,380.00) 2,760.00

100.00 16,100.00 -

Cash Beg. 6,000.00


Proceeds 30,700.00
Payment of Liabilities (20,500.00)
Payment to Partners 16,200.00

Problem 4 (ReSA)

When Ray and Conniff, general partners of the Ray Conniff partnership who shared net income
and losses in a 4:6 ratio were incapacitated in an accident, a liquidator was appointed to raise
up the partnership. The partnership’s balance sheet showed the following:

Assets Liabilities and Capital


Cash 17,500.00 Liabilities 10,000.00
Other Asset 50,000.00 Ray, capital 35,500.00
Goodwill 5,000.00 Conniff, capital 27,000.00

72 ,500.00 Total 72,500.00

Liquidation expenses paid P2,500 for advertising, rent, travel, etc. and in the process of
liquidating the partnership an overlooked bill for landscaping of P1,000 is discovered and in
addition, partners agree to keep a P1,500 contingent fun. Determine the amount of cash that
should be paid to each partner:
a. Ray, 11,500; Conniff, 0 c. Ray, 7500; Conniff, 0
b. Ray, 2,500; Conniff, 0 d. Ray, 5,000; Conniff, 0

Solution:

40% 60%
Ray Conniff Total
35,500.00 27,000.00 62,500.00

(24,000.00) (36,000.00) 60,000.00


11,500.00 (9,000.00) 2,500.00

(9,000.00) 9,000.00
2,500.00 - 2,500.00

Cash beg 17,500.00


Liquidation Expenses (2,500.00)
Payment of Liability (10,000.00)
Unrecorded (1,000.00)
Cash Withheld (1,500.00)
Payment to Partners 2,500.00

Problem 5 (ReSA)

The partnership of JJ, KK, LL and MM is preparing to liquidate. Profit and loss sharing ratios are shown
is the summarized balance sheet at December 31, 2019 as follows:

Total 465,000.00 Total 465,000.00


Assets Liabilities and Capital
Cash 100,000.00 Other Liabilities 50,000.00
Inventories 100,000.00 JJ, loan 50,000.00
Loan to KK 10,000.00 JJ, capital (40%) 100,000.00
Other Assets 255,000.00 KK, capital (20%) 160,000.00
LL, capital (20%) 50,000.00
MM, capital (20%) 55,000.00
During January 2020, the inventories are sold for P42,500, the others liabilities are paid and P25,000
is set-aside for contingencies

Compute the total cash payment to partners:

Payment to
Partners

a. 97,500.00
b. 102,500.00
c. 72,500.00
d. 67,500.00

Solution:

Cash beg. 100,000.00


Proceeds 42,500.00
Payment of Liability (50,000.00)
Cash Withheld (25,000.00)
Payment to Partners 67,500.00

Problem 6 (PRTC)

Partners Edong, Sally and Zarah decided to liquidate their partnership on November 30, 2017. Their
capital balances and profit and loss are as follows:
Capitals P&L ratio
Edong P 600,000 40%
Sally 784,000 40%
Zarah 240,000 20%
The net income from January 1, 2017 to November 30, 2017 is P656,000. On November 30,
2017, the cash balance is P520,000, and that of liabilities is P1,160,000. Edong is to receive
P706,560 in the settlement of his interest.

1. Calculate: (1) The loss on realization, and (2) the amount to be realized from the sale of non-
cash assets?

A. (1) P 389,600 (2) P2,530,400


B. (1) P 248,000 (2) P5,100,000
C. (1) P 620,000 (2) P3,860,000
D. (1) P 522,000 (2) P3,860,000
Solution:
Edong Sally Zarah Total
Beg. Balance 600,000 784,000 240,000 1,624,000
Net Income 262,400 262,400 131,200 656,000

Adjusted Balance 862,400 1,046,400 371,200 2,280,000


Cumulative Loss (155,840) (155,840) (77,920) (389,600)
Cash Payment 706,560 890,560 293,280 1,890,400

(1,160,000 + 2,280,000 -
Book Value of NCA 520,000) 2,920,000
Loss on Realization (389,600)
Proceeds 2,530,400

Problem 7 (PRTC)

The partnership of Mikee and Rosa is in the process of liquidation. On January 1, 2017, the ledger
shows account balances as follows:
Cash P 8,000 Accounts Payable P 12,000
Accounts Receivable 20,000 Mikee, Capital 32,000
Lumber Inventory 32,000 Rosa, Capital 16,000
On January 10, 2017, the lumber inventory is sold for P20,000, and during January, accounts
receivable of P16,800 is collected. No further collections on the receivables are expected and the
partners have incurred P3,200 of liquidation expenses. Profits are shared 60% for Mikee and
40% for Rosa.
2. How much cash will partner Mikee and Rosa receive upon liquidation?

A. P22,800; P9,920 C. P20,960; P8,640


B. P37,600; P18,400 D. P20,500; P20,500

Solution:

Mikee Rosa Total

Beg. Balance 32,000 16,000 48,000


Cumulative Loss (11,040) (7,360) (18,400)

Cash Payments 20,960 8,640 29,600

Problem 8 (PRTC)

The partnership ABC is currently liquidating and on February 15, 2017, their balances in capital and
their profit and loss ratios are shown below:
Apple, Capital (P&L 40%) P 22,000
Bryan, Capital (P&L 20%) 14,000
Cecile, Capital (P&L 40%) -12,000
Assume non-cash assets have been all disposed and Cecile has promised to pay his deficiency in a
week’s time.
3. Calculate the amount to be received by one of the partners if cash is paid immediately on
February 15, 2017.

A. Apple, P22,000 C. Bryan, P10,000


B. Bryan, P12,000 D. Apple, P12,000

Solution:

Apple Bryan Cecile


Balance 22,000 14,000 12,000
APL -8,000 -4,000 -12,000

Free Interest 14,000 10,000 0

Problem 9 (PRTC)

The balance sheet for Chester, Joana and John partnership, who share profits and losses in the
ratio of 50%, 25% and 25%, respectively, shows the following balances just before liquidation.

Cash P 24,000
Other Assets 119,000
Liabilities 40,000
Chester, Capital 44,000
Joana, Capital 31,000
John, Capital 28,000
On the first month of liquidation, certain assets are sold for P64,000. Liquidation expense of
P2,000 are paid, and additional liquidation expenses are anticipated. Liabilities are paid
amounting to P10,800 and sufficient cash is retained to insure the payment to creditors before
making payments to partners. On the first payment to the partners, Chester receives P12,500

4. Determine the amount of cash withheld for anticipated liquidation expenses.

A. P35,200 C. P33,200
B. P29,200 D. P6,000

Solution:

Chester Joana John Total

12,500 15,250 12,250 40,000


Beg.
44,000 31,000 28,000 103,000
Balance
Loss (31,500) (15,750) (15,750) (63,000)
Cash
Payment

24,000 + 64,000 - 40,000 - 2,000 -


6,000
40,000

Problem 10 (PRTC)

A condensed balance sheet with profit sharing percentages for the E, F and G partnership on January
1, 2017, shows the following:

Cash P 100,000 Liabilities P 80,000


Other Assets 500,000 E, Capital (40%) 100,000
F, Capital (40%) 250,000
G, Capital (20%) 170,000
Total P 600,000 Total P 600,000
On January 2, 2017, the partners decided to liquidate the business, and during January they sell
assets with a book value of P300,000 for P170,000.

5. How much cash will the partners receive if all available cash, except for a P10,000 contingency fund,
is distributed immediately after the sale

A. All partners will receive P60,000


B. Partners F and G will both receive P90,000
C. Partner F will receive P96,667 and Partner G will receive P93,333
D. Partner F will receive P190,000

Solution:

E F G

-36,000 114,000 102,000


36,000 (24,000) (12,000)
0 90,000 90,000

Beg. Balance 100,000 250,000 170,000


Cumulative Loss (136,000) (136,000) (68,000)
Balance
Absorption
Cash

Problem 11 (CRC-ACE)

A, B, and C are partners sharing profits in the ratio of 5:3:2, respectively. A balance sheet prepared
just prior to partnership liquidation shows the following:

A B C
Capital Balances P 122,000 P 72,000 P47,000
Loan Balances P 43,000 P 48,000 P 6,000
Assets are sold and cash is distributed to partners in monthly instalments during the course of
liquidation as follows:
January P 20,000
February 50,000
March 80,000
April (final distribution) 20,000

Required:

a. Prepare a program to show how cash is to be distributed during the entire course of
liquidation.
b. Using the program developed above, prepare a schedule summarizing the payments
to be made to partners at the end of each month.

Solution:

A. CCP/ACDP A B C
TOTAL EQUITY 165,000 120,000 153,000
CASH
/ P&L 1 0.2 0.3 DISTRIBUTION
330,000 400,000 265,000 A B C TOTAL
P1 B. -70,000 21,000 21,000
-65,000 -65,000 32,500 19,500 52,000
265,000 265,000 265,000 ANY CASH IN EXCESS OF 73,000
IS ALLOCATED AT P/L

T TOTAL A B C
JANUARY 20,000
-
P1 20,000 20,000
FEBRUARY 50,000
P1 -1,000 1,000
-
P2 49,000 30,625 18,375
50,000 30,625 19,375
MARCH 80,000
P2 -3,000 1,875 1,125
-
P/L 77,000 38,500 23,100 15,400
80,000 40,375 24,225 15,400
APRIL 20,000 10,000 6,000 4,000
Problem 12 (CRC-ACE)

Elizabeth, Diana, Anthony, and Scarlett were partners who decided to liquidate the affairs of the
partnership. Prior to dissolution, the condensed balance sheet together with the profit and loss
sharing ratio was derived as follows:

P P
Cash 100,000 Liabilities 750,000
Other Assets 1,800,000 Diana, Loan 60,000
Scarlett, Loan 50,000
Elizabeth,Capital (30%) 420,000
Diana, Capital (30%) 315,000
Anthony, Capital (20%) 205,000
Scarlett, Capital (20%) 100,000
P
1,900,000 P 1,900,000

The other assets were sold for P 1,200,000. Payments were made to creditors and final distributions of
cash were made to partners.

a. The partner who got paid the most was:


b. The cash received by Scarlett will be applied:

Solution:

E D A S TOTAL
420,000 375,000 205,000 150,000 1,150,000
- - - -
180,000 180,000 120,000 120,000 -600,000
240,000 195,000 85,000 30,000 550,000

A. E
B.
LOAN

Problem 13 (CRC-ACE)
D, E, and F are partners sharing profits in the ratio of 40:35:25, respectively. On December 31,
2018, they agree to liquidate. A balance sheet prepared on this date follows:

DEF Partnership
Balance Sheet As of December 31, 2018
Cash P 2,000 Liabilities P 6,000
Other Assets 46,000 E, Loan 5,000
F, Loan 2,500
D, Capital 14,450
E, Capital 12,550
F, Capital 7,500
P 48,000 P 48,000
The results of liquidation are summarized below:

Expenses Cash Withheld


Realizations Book Cash of at end of month Liability Value Realized Realization for estimated Paid
future expenses
January P 12,000 P 10,500 P 500 P 2,000 P 4,000
February 7,000 6,000 750 1,250 2,000
March 15,000 10,000 600 500 -
April 12,000 4,000 400 - -
All cash available, except the amount withheld for future expenses, is distributed at the end of
each month.

Required: Determine the share of each partner every month of distribution.

Solution:

JAN. FEB. MARCH APRIL


BEGINNING 2,000 4,000 1,250 500
PROCEEDS 10,500 6,000 10,000 4,000
EXPENSE -500 -750 -600 -400
LIABILITIES -6,000 -2,000
CASH WITHELD -2,000 -1,250 -500
CAFD 4,000 6,000 10,150 4,100

JANUARY
D E F TOTAL
TOTAL EQUITY 14,450 17,550 10,000 42,000
-15,200 -13,200 -9,500 -38,000
CAFD -750 4,250 500 4,000
750 -440 -312.5
0 3,812.50 187.5 4,000

FEBRUARY
14,450 13,737.50 9,812.50 38,000

(12,800) (11,200) (8,000)


(32,000)

1,650 2,540 1,813


6,000

4,060 3,553 2,540


MARCH 10,150

APRIL 1,640 1,435 1,025 4,100

Problem 14 (CRC-ACE)

The balance sheet of J, K, and L Partnership shows the following information as of December 31,
2018:
Cash P 2,000 Liabilities P 5,000
Other
Assets 28,000 J, Loan 2,500
J, Capital 12,500
K, Capital 7,000
P 30,000 L, Capital 3,000
P
30,000

Profit and loss ratio is 3:2:1, respectively, for J, K, and L. Other assets were realized as follows:

Date Cash Received Book Value


January, 2018 P 8,000 P 9,000
February, 2018 3,500 7,700
March, 2018 12,500 11,300

Cash is distributed as assets are realized.

a. How much is the total loss to J?


b. How much is the total cash received by K?
c. How much cash does L receive in January?

Solution:

J K L TOTAL

TOTAL EQUITY 15,000 7,000 3,000 25,000

(2,000) (1,333) (667) 4,000

13,000 5,667 2,333 21,000

JANUARY 15,000 7,000 3,000 25,000

(10,000) (6,667) (3,333) 20,000

5,000 333 (333) 5,000

(200) (133) 333

4,800 200 - 5,000

A 2,000

B 5,667
C -0-

Problem 15 (CRC-ACE)

Balance sheet data for the firm of W, X, and Y as of January 1, 2018, follow:

P
P P Assets 1,225,000 Liabilities P 675,000
1,225,000 W, Capital 200,000
X, Capital 200,000
Y, Capital 200,000
1,225,000

Partners share profits equally after allowance of a salary to Y, the managing partner, of P7,500
monthly. As a result of operation losses sustained at the beginning of 2018, W advanced P
150,000 to the firm on April 1; it was agreed that he would be allowed interest at 6%. With
continued losses, the members decided to liquidate. Y agreed to take over partnership equipment
in part of settlement of his interest, the transfer being made at an agreed value of P 40,000. On
November1, P 200,000 cash was available for distribution to partners after the sale of remaining
assets and payment of partnership obligations to outsiders. Y had withdrawn his salary for
January and February but had not received his salary for the period of March 1 to November 1;
no other cash payments had been made to partners. Available cash was distributed on November
1 and the firm was declared dissolved.

How much cash should W received in the distribution of P 200,000 cash available?

Solution:

W X Y TOTAL
200,000 200,000 200,000 600,000

150,000

5,250

(1,750) (1,750) (1,750) (5,250)

(40,000) (40,000)

60,000 60,000

353,500 198,250 218,250 770,000

(190,000) (190,000) (190,000) (570,000)

163,500 8,250 28,250 200,000

2.0 Corporate Liquidation

Problem 1 (ReSA)

The following data were taken from the statement of affairs for Liquo Company:

Asset pledged for fully secured liabilities


(fair value, P75,000) 90,000.00
Asset pledged for partially secured liabilities
(fair value, P52,000) 74,000.00
Free Assets (fair value, P40,000) 70,000.00
Unsecured Liabilities with priority 7,000.00
Fully secured liabilities 30,000.00
Partially secured liabilities 60,000.00
Unsecured liabilities without priority 112,000.00

1. Total estimated deficiency to unsecured creditors amounted to:


a. 27,000 c. 35,000
b. 34,000 d. 42,000
2. The expected recovery per peso of unsecured claims amounted to:
a. 0.35 c. 0.70
b. 0.65 d. 0.71

Solution:

Free assets on assets pledged to fully secured assets (75,000-30,000) 45,000


Free assets on assets pledged to fully secured assets (75,000-30,000) 40,000

Total Free assets 85,000


Less: Unsecured creditors w/ priority (7,000)

Net free assets 78,000


Unsecured Creditors:
Partially secured creditors (60,000-52,000) 8,000
Unsecured creditors without priority 112,000 120,000

Estimated deficiency to unsecured creditors 42,000

Expected recovery per peso of unsecured creditors


Net free assets / Total unsecured creditors
78,000/120,000 0.65

Problem 2 (ReSA)

Zero Na Corp. has been undergoing liquidation since January 1. As of March 31, its condensed statement
of realization and liquidation is presented below:

Assets:
Assets to be realized 95,000
Assets acquired 5,000
Assets realized 30,000
Assets not realized 42,000
Liabilities
Liabilities liquidated 35,000
Liabilities not liquidated 31,850
Liabilties to be liquidated 65,000
Liabilities assumed Revenue 1,500
and Expenses:
Sales on account 5,000
Purchases 1,500
Payment of expenses of trustee 7,500
Sales for cash 25,000
Interest on marketable securities 150

The net gain (loss) for the three-month period ending March 31 is:

a. 7,200 c. 49,500
b. (7,200) d. (17,500)

Solution:

Statement of Realization and Liquidation


95 ,000 30 ,000
5 ,000 42 ,000
35 ,000 65 ,000
31 ,850 1 ,500
1 ,500 5 ,000
7 ,500 25 ,000
150
175,850 168,650
7,200

Problem 3 (ReSA)

Orville Company recently petitioned for bankruptcy and is now in the process of preparing a
statement of affairs. The carrying values and estimated fair values of the assets or Orville
Company are as follows:

Carrying Value Fair Value

Cash 20,000 20,000


Accounts Receivable 45,000 30,000
Inventory 60,000 35,000
Land 75,000 70,000
Building (net) 180,000 100,000
Equipment (net) 170,000 80,000
Total 550,000 335,000
Total

Debts of Orville are as follows:

Accounts payable 60,000.00


Wages payable (all have priority) 10,000.00
Taxes payable 10,000.00
Notes payable (secured by receivable and inventory) 120,000.00
Interest on Notes Payable 6,000.00
Bonds Payable (secured by land and building) 150,000.00
Interest on Bonds payable 7,000.00
363,000.00

1. What is the total amounts of unsecured claims


a. 93,000 c. 121,000
b. 113,000 d. 126,000

2. What is the estimated amount will be available for general unsecured creditors upon
liquidation?
a. 28,000 c. 113,000
b. 93,000 d. 121,000

3. What is the estimated dividend percentage?


a. 23% c. 77%
b. 93% d. 68%

Solution:

1. 60,000 + [(120,000+)] – (30,000) + (35,000) = 121,000


2. 20,000+80,000+[170,000-(150,000+7,0000]=113,000–(10,000+10,000)=
93,000
3. 93,000/121,000= 77%

Problem 4 (ReSA)

Kareindeer Corporation filed a voluntary petition for bankruptcy on January 2016. On March 31,
2016, the trustee provided the following information about the corporation’s financial affairs:

Est. Realizable
Assets Book Value Value
Cash 40,000 40,000
Accounts receivable- net 200,000 150,000
Inventories 300,000 140,000
Plant assets - net 500,000 560,000
Total Assets 1 ,040,000

Liabilities
Liabilities for priority claims 160,000
Accounts payable - unsecured 300,000
Notes payable, secured by accounts
receivable 200,000
Mortgage payable, secured by all plant
assets 440,000

Total Liabilities 1 ,100,000

1. The amount expected to be available for unsecured claims without priority (net free
assets):
a. 300,000 c. 140,000
b. 580,000 d. 310,000

2. The expected recovery per peso of unsecured creditors:


a. .215 c. .415
b. .223 d. .400

3. The estimated payment to creditors:


a. 730,000 c. 770,000
b. 45,000 d. 890,000

Solution:

1.
Cash 40,000
Inventories 140,000
Plant Assets (560,000-440,000) 120,000
Liabilities w/ priority claims (160,000)

Net Free Assets 140,000

2.
Expected recovery per peso of unsecured creditors
Net free assets / Total unsecured creditors
140,000/350,000 0.40

3.

Secured Liability 440,000


Liability w/ priority 160,000
Liability w/out priority (300,000x40%) 120,000
Partially secured [150,000+(50,000x40%)] 170,000

Est. payment to creditors 890,000

Problem 5 (ReSA)

The unsecured creditors of Insolve Corporation filed a petition in July 1, 2016 to force Insolve
Corporation into bankruptcy. The court order for relief was granted on July 10 at which time an
interim trustees was appointed to supervise liquidation of the estate. A listing of assets and
liabilities of Insolve Corporation as of July 10, 2016, along with estimated realizable value is as
follows:

Assets Book Value Est. Realizable Value

Cash 61,400 61,400

Accounts Receivable 250,000


15% of the accounts receivable is
Allownce for D/A (20,000) estimated to be uncollectible
Estimated selling price , P340,000
which will require additional cost of
Inventories 420,000 P50,000

Prepaid Expenses 40,000 ?

Investments 180,000 110,000

Land 210,000
An offer of P500,000 has been
Buildings (net) 260,000 received for land and buildings

Machinery and Equipment 220,000 53,900

Goodwill 200,000 ?

Total Assets 1,821,400


Liabilities & Equity

Accounts Payable 670,000

Wages payable 3,400

Notes payable 160,000


Accrued Interest notes 5 ,000
Mortgage payable, secured
by land and building 400,000
Capital Stock 800,000
Addtl Paid in Capital 80 ,000
Deficit (297,000)
Total Liab.& Equity 1 ,821,400

Additional information:

a. Patents completely written off the books in past years but with a realizable value of
P10,000
b. The books do not show the following accruals (unrecorded expenses/additional liabilities):

Taxes 16,400
Interest on Mortgage 10,000

c. The investment have been pledged as security for holder of the notes payable
d. The trustee fees and other costs of liquidating the estate are estimated to be P60,000

Determine:

1. The total free assets should be:


a. 1,831,400 c. 717,800
b. 1,821,400 d. 638,000

2. The net free assets should be:


a. 717,800 c. 638,000
b. 698,000 d. 628,000

3. The estimated deficiency to unsecured creditors should be:


a. 87,000 c. 27,000
b. 47,800 d. 7,200
Free Assets:
Cash 61,400
Accounts Rec. (250,000x85%) 212,500
Inventories (340,000-50,000) 290,000
Prepaid Expenses -
Machinery & Equipment 53,900
Goodwill -
Additional Assets/unrecorded assets:
Patent 10,000
Solution:

1.

Assets pledged to fully secured creditors:


Land and Buildings 500,000
Less: Mortgage Payable 400,000
Interest Payable 10,000 410,000 90,000

Total Free Assets 717,800

2.

Total Free Assets 717,800


Unsecured creditors w/ priority 3,400
Taxes payable 16,400
Administrative expenses 60,000 79,800

Net Free Assets 638,000

Secured Creditors:
Investments 110,000
Less: Notes Payable 160,000
Interest payable 5,000 165,000 55,000

Unsecured Creditors w/out priority:


Accounts payable 670,000

Total Unsecured creditor w/out priority 725,000


Net Free Assets (638,000)
87,000

PROBLEM 6(PRTC)
The following data were taken from the statement of affairs of MIRIAM CORPORATION: Assets
pledged for
(current fair value, P93,750) P112,500 fully secured
Assets pledged for partially secured liabilities liabilities
(current fair value P65,000) 92,500
Free assets (current fair value, P50,000) 87,500
Unsecured liabilities with priority 18,750
Fully secured liabilities 37,500
Partially secured liabilities 75,000
Unsecured liabilities without priority 140,000

1. The amount that will be paid to creditors with priority is:


A. P8,750 C. P9,375
B. P7,500 D. P7,750
2. The amount to be paid fully secured creditors is:
A. P37,500 C. P25,000
B. P40,000 D. P43,750
3. The amount to be paid to partially secured creditors is:
A. P65,875 C. P70,250
B. P71,500 D. P71,250
3. The amount to be paid to unsecured creditors is:
A. P97,750 C. P90,000
B. P88,500 D. P91,000

Solution:
Cash available (93,750+65,50,000) 208,750
Prioritized Claims
Fully secured 37,500
Partially secured liabilities (secured) 65,000
W/ Priority 8,750 (111,250)
52,500
Net Cash 97,500 Unsecured
Unsecured Amount w/out priority
140,000
Partially secured liabilities (unsecured) 10,000 65%
W/out Priority 140,000 150,000
ERR = 97,500/150,000 65%

Partially secured 75,000


Secured Portion 65,000 100% 65,000
Unsecured Portion 10,000 65% 6,500

71,500

91,000 PROBLEM 7 (PRTC)


The Statement of Affairs for CANDY CORPORATION shows that approximately P0.78 on the peso
probably will be paid to unsecured creditors without priority. The corporation owes TOY COMPANY
P28,750 on a promissory note, plus accrued interest of P1,175.. Inventories with a current fair
value of P24,000 collateralize the note payable. Compute the amount that the TOY COMPANY
would receive from CANDY CORPORATION assuming that the actual payments to unsecured
creditors without priority consist of 78% of total claims. Round all amounts to the nearest peso.
A. P24,000 C. P42,483.75
B. P28,612.50 D. P65,250

Solution:
Partially secured (28,750 + 1,175) 29,925
Secured Portion 24,000 100% 24,000
Unsecured Portion 5,925 78% 4,621.50
28,621.50

PROBLEM 8 (PRTC)

The statement of affairs of ADAMSON CORPORATION shows the following:


Estimated gains on realization of assets P1,600,000
Estimated losses on realization of assets 2,805,000
Contingent assets 1,250,000
Contingent liabilities 375,000
Capital stock 2,500,000 Deficit 11,125,000
The pro-rata payment on the peso, to stockholders, is:
A. P0.78 C. P0.76
B. P0.43 D. P0.75

Solution:
Est. gross loss (2,805,000 + 375,000) 3,180,000
Gain Contingent asset (1,600,000+1,250,000) (2,850,000)
Est. loss (net) (330,000)
BV of SHE (2,500,000 - 1,125,000) 1,375,000

Est. amt. recoverable by stockholders 1,045,000


/ 1,375,000

Pro rata payment 76 %

PROBLEM 9 (PRTC)
When NATIONAL COMPANY filed for liquidation with Securities and Exchange Commission, it
prepared following balance sheet.
Current assets, net realizable value, P137,500 P100,000
Land and buildings, fair value, P225,000 250,000
Goodwill, fair value, PO 50,000
Total assets P400,000

Accounts payable P200,000


Mortgage payable, secured by land and building 250,000
Common stock 125,000
Retained earnings, deficit (175,000)
Total equities P320,000
1. How much would the holders of the mortgage payable likely to get?
A. 75,000 C. 240,000
B. 450,000 D. 301,250

2. What is the estimated deficiency to unsecured amounts?

Estimated deficiency (87,500)

ERR = 137,500/225,000 61.11%

MNP (Total BV) 250,000


Unsecured Portion 225,000 100% 225,000
Unsecured Portion Balance 25,000 61.11% 15,278
240,278
A. 87,500 C. 240,000
B. 112,500 D. 175,000

Solution:

Cash available (137,500+225,000) 362,500


Prioritized Claim - MNP (Secured) (225,000)
Net Cash 137,500
Unsecured Amount
MNP (unsecured) 25,000
A/P 200,000 225,000

PROBLEM 10 (PRTC)

BENILDE ENTERPRISES. has been forced into bankruptcy and liquidated. Unsecured claims will
be paid at the rate of P0.70 on the peso. LETRAN ENTERPRISES holds a noninterest bearing note
receivable from BENILDE in the amount of P75,000 collateralized by machinery with a liquidation
value of P12,500. The total amount to be realized by LETRAN on this note receivable is:
A. P56,250 C. P31,250
B. P37,500 D. P12,500

Solution:
Partially Secured Claims 75,000
56,250 Secured Portion 12,500 100% 12,500
Unsecured Portion 62,500 70.00% 43,750

PROBLEM 11 (CRC-ACE)

The following information is available on June 1, 2018 to Samsung Company, which is having difficulty
in paying its liabilities as they come due:
Carrying amount
cash 8,960
accounts receivable, net, fair value equal to carrying amount 103,040
inventories, current fair value, P40,320 pledged on P47,040 of 87,360
notes payable
machinery and equipment, net, current fair value of P150,976 pledged 239,680
on mortgage note payable
office supplies, current fair value of P5,600 4,480
wages payable 12,992
taxes payable 2,688
accounts payable 134,400
notes payable, P47,040 of which is secured by inventories 89,600
mortgage note payable 112,896
common stock, P5 par 224,000
retained earnings, deficit 133,056
Additional information:
(1) estimated liability to the trustee is P58,240
(2) a delivery van previously given to the supervisor was returned to the company, fair market
value, P56,000

REQUIRED:
a) compute the estimated recoverable amounts to the different types of creditors in the event
of liquidation.
b) prepare statement of deficiency to unsecured creditors

ANSWER:
Asset @ FV Secured Unsecured Free Asset
Cash 8,960
A/R 103,040
Inventory 40,320 6,720
PPE 112,896 38,080
Supplies 5,600
Delivery van 56,000

Total Free asset 211,680


Total free asset unsecured 211,860
liabilities w/ priority
wages payable 12,992 taxes payable
2,688
trustee 58,240 73,920
net free asset 137,760

w/o priority
Accounts payable 134,400
Notes payable 42,560
Partially secured liab. 6,720 183680
Estimated deficiency 45,920

PROBLEM 12 (CRC-ACE)
SMDC Corp. a closely-held corporation was undergoing liquidation. The total cash value of
SMDC’s bankruptcy estate after the sale of all assets and payment of administrative expenses is
P100,000.

SMDC has the following creditors:

• BDO bank is owed P75,000 on a mortgage loan secured by SMDC’s real property.
The property was valued at and sold, in bankruptcy, for P70,000.
• The BIR has a P12,000 recorded judgement for unpaid corporate income tax.
• National Office Supplies has an unsecured claim of P3,000 that was timely filed.
ACE Electric Company has an unsecured claim of P10,000 that was timely filed.
• REH Publications has a claim of P16,000, which is secured by SMD’s inventory that
was valued and sold, in bankruptcy, for P2,000. The claim was timely filed.

REQUIRED:
a) Calculate the total amount recoverable by partially-secured creditors.

ANSWER:
Partially secured liability 72,000
9,500
81,500

PROBLEM 13 (CRC-ACE)

A Company that was to be liquidated had the following liabilities:


Income Taxes 10,000
Notes Payable secured by land 100,000
Accounts Payable 251,050
Salaries Payable 12,950
Administrative expenses or liquidation 20,000
The Company had the following assets: Book Value Fair Value
Current assets 100,000 95,000
Land 50,000 75,000
Building 150,000 200,000

Determine the following:

1. Total free assets

Current Asset FV 95,000


Building FV 200,000
Total assets 295,000
2. Total liabilities with priority
Income taxes 10,000
Salaries payable 12,950
Admin. Expense 20,000
42,950

3. Net free assets


Total Assets 295,000
Fully secured liability 0
Unsecured liability w/ priority 42,950
Net free asset 252,050

PROBLEM 14 (CRC-ACE)

The following data were taken from the statement of realization and liquidation of CRASHED CO.

Assets to be realized 1,375,000 Assets acquired 750,000


Supplementary credits 2,800,000 Assets realized 1,200,000
Liabilities to be liquidated 2,250,000 Liabilities assumed 1,625,000
Supplementary charges 3,125,000 Assets not realized 1,375,000
Liabilities liquidated 1,875,000 Liabilities not liquidated 1,700,000

The ending balance of capital stock and retained earnings are P1,500,000 and P238,000, respectively.
A net loss of P738,000 was reported for the period.

1. What is the net gain/(loss) for the three-month period?


a. (325,000) c. 425,000
b. 250,000 d. 750,000

Solution:

Supplementary credits 2,800,000


Liabilities to be liquidated 2,250,000
Assets realized 1,200,000
Liabilities assumed 1,625,000
Assets not realized 1,375,000
Less: 9,250,000

Assets to be realized 1,375,000


Supplementary charges 3,125,000
Liabilities liquidated 1,875,000
Assets aquired 750,000
Liabilities not liquidated 1,700,000
8,825,000
425,000

PROBLEM 15 (CRC-ACE)

The following data were taken from the statement of realization and liquidation of Bagsak Corporation
for the three month period ended December 31, 2018:

Assets to be realized 55,000


Assets acquired 60,000
Assets realized 70,000
Assets not realized 25,000
Liabilities to be liquidated 90,000
Liabilities assumed 30,000
Liabilities liquidated 60,000
Liabilities not liquidated 75,000
Supplementary credits 85,000
Supplementary charges 78,000

What is the net income (loss) for the period?


ANSWER:

Assets to be realized 55,000


Assets aquired 60,000
Liabilities not liquidated 75,000
Liabilities liquidated 60,000
Suplementary charges 78,000 328,000

Assets realized 70,000


Assets not ealized 25,000
Liabilities assumed 30,000
Suplemerntary credits 85,000 300,000
Net loss 28,000

3.0 Joint Arrangements

Problem 1 (ReSA)

The joint operation accounts in the books of the operators, X, Y and Z, show the balances below, upon
termination of the joint arrangement and distribution of profits:

Accounts With X Dr. (Cr.) Y Dr. (Cr.) Z Dr. (Cr.)

X - 2,500 2,500
Y 4,000 - 4,500
Z (6,500) (6,500) -

Final settlement of joint operations will require payments as follows:

a. X pays 2,500 to Z and Y pays 4,000 to Z


b. Z pays 2,500 to X and 4,000 to Y
c. Y pays 6,500 to X and Z pays 2,500 to Y
d. None of these

Solution:
Joint Operations - X Joint Operation - Y
4 ,000 6 ,500 2 ,500 6 ,500
2,500 4,000

Joint Operation - Z
2,500

4,000
6,500

Problem 2 (ReSA)

Soriente, Santos and Salazar formed a joint operations, Soriente has been designated as
manager of the arrangements for which he is to receive a bonus of 15% of the profit after
deduction of the bonus as an expense. The net profit, after bonus has been agreed to be divided
as follows: Soriente, 25%; Santos 40% and Salazar 35%

After 5 months, the joint arrangement is terminated as of May 31, 2012. On this date, the trial
balance kept by Soriente contains the following balances:
Debit Credit

Investment in Joint
Arrangement 9,000
Santos 500
Salazar 2,000

The joint operations has still some undisposed merchandise, which Soriente agreed to purchase at its
costs of P2,500. The bonus of Soriente has not yet taken up.

1. The net profit of the joint arrangement, after bonuss of Soriente is:
a. 1,500 b. 9,000 c. 10,000 d. 11,500
2. The share of Santos in the joint arrangement is:
a. 3,500 b. 3,600 c. 4,000 d. 4,600

Solution:
Joint Operations
1.
9,000

2,500

11,500
2. (1,500)

10,000
Soriente Santos Salazar Total

Bonus 1,500 - - 1,500


Share in
Net Profit after
Profit
Bonus 2,500 4,000 3,500 10,000

4 ,000 4,000 3 ,500 11 ,500

Problem 3 (ReSA)

Ace Company purchase 40% of Basket Company on January 1 for P500,000 that carry voting
rights at a general meeting of shareholders of Basket Company. Ace Company and Blake
Company immediately agreed to share control (wherein unanimous consentis needed to all the
parties involved) over Basket Company. Basket report assets on that date of P1,400,000 with
liabilities of P500,000. One building with a seven-year life is undervalued on Basket’s books by
P140,000. Also Basket’s book value for its trademark (10 year life) is undervalued by P210,000.
During the year, Basket reports net income of P90,000, while paying dividends of P30,000.
1. What is the Investment in Basket Company balance (equity method) in Ace’s financial
records as of December 31?
a. 504,000 b. 507,600 c. 513,900 d. 516,000

2. The income form Investment in Basket Company in Ace’s financial records as of December
31?
a. 36,000 b. 19,600 c. 12,000 d. 7,600

Solution:

1.
Net Income (30,000x40%) Investment
(90,000x40%) Dividends 500,000
Amortization* Invesment Balance
36 ,000 12 ,000
16 ,400
536,000 28 ,400
507,600

*Amortizaton
Building (140,000x40%) 56,000
Useful life 7 8,000

Trademark (210,000x40%) 84,000


Useful life 10 8,400

16 ,400

2.

Income from Investment


Income 36,000

Amortization 16,400
Expense
19,600

Problem 4 (ReSA)

K and L form a joint arrangement for the sale of certain merchandise. The joint operators agree
to the following: K shall be allowed a commission of 10% on his net purchases, the joint operators
shall be allowed commissions of 25% on their respective sales, and K and L shall divide the profit
or loss 60% and 40% respectively. Joint arrangements transactions follow:

Dec. 1: K make cash purchase of P57,000


Dec 3: L pays joint arrangement expenses of P9,000
Dec 5: Sales are as follows: K, P48,000; L, P36,000. The operators keep their own cash receipts
Dec 7: K returns unsold merchandise and receives P15,000 cash Dec 15:
The operators make cash settlement.
1. In the distribution of the balance in net profit of the joint arrangement, the shares
of K and L:
a. K, 4,260; L, 3,230 c. K, 4,820; L, 3,430
b. K, 4,680; L, 3,120 d. K, 4,840; L, 4,230

2. In the final cash settlement, L would pay K the amount of:


a. 14,100 b. 14,880 c. 15,100 d. 15,890

Solution:

1.

Investment in Joint Operations


Sales 57,000 48,000
Sales 9,000 36,000
Commission Exp (42,000x10%) 4,200 15,000 Merchandise Returned
Commission Exp (48,000x25%) 12,000
Commission Exp (36,000x25%) 9,000
91,200 99,000
Income 7,800
K(7,800*60%) 4,680

L(7,800*40%) 3,120
2.

K
57,000

48,000 4,200
15,000 12,000 4,680

14,880

Problem 5 (ReSA)

Panner Inc. owns 30% of Watkins and applies the equity method. During the current year, Panner
buys inventory costing P54,000 and then sells its Watkins for P90,000. At the end of the year,
Watkins still holds only P20,000 of merchandise. What amount of unrealized gross profit must
Panner defer in reporting this investment using the equity method?

a. 2,400 c. 8,000
b. 4,800 d. 10,800
Solution:

Gross Profit Mark-up: 36,000/90,000 = 40%


Inventory Remaining at year end 20,000
x: Markup 40%

Unrealized profit in ending inventory 8,000


x: Ownership 30%

Intercompany Unrealized profit in ending inventory

2,400
PROBLEM 6 (PRTC)

On January 1, 2018, HHH, III, and JJJ (all are corporations) establish a joint undertaking to
manufacture a product they agree to share equally. Each will contribute P200,000 into the
operation; HHH and III are to contribute cash while JJJ is to contribute equipment with a cost of
P185,000. The equipment has a remaining life of 10 years when contributed.

1. Determine the amount JJJ will show the Equipment in JO account in its balance sheet
at January 1, 2018.
A. P61,667 C. P66,667
B. P50,000 D. P65,000

Solution:
FV/SP 200,000
BV 185,000
COS 15,000 * 1/3 5,000 - Deferred Gain

JJJ's Share in FV
(200,000 x 1/3) 66,667
Unamortized deffered gain
(2000,000 - 185,000) x 1/3 (5,000)

Equipment in JJJ's books, 12/31/18 61,667

2. Determine the net amount JJJ will show the Equipment in JO account in its balance
sheet at December 31, 2018.
A. P45,000 C. P60,000
B. P55,500 D. P58,500

Solution:
JJJ's Share in Current carrying value
(200,000 * 90%) x 1/3 60,000
Unamortized deffered gain
(5,000 * 90%) (4,500)
Equipment in JJJ's books, 12/31/18 55,500

3. Determine the net amount HHH (or III) will show the Equipment in JO account in its
balance sheet at December 31, 2018.
A. P45,000 C. P60,000
B. P55,500 D. P58,500

Solution:
200,000 x 90% x 1/3 = 60,000

PROBLEM 7 (PRTC)

HHH and III are venturers in a joint arrangement sharing control and profits equally. They
contributed P625,000 each to establish Joint Venture JJ) early in 2018. The Joint Venture paid
cash dividends of P45,000 and reported a net income of P180,000 during the year. On the other
hand, HHH paid cash dividends of P22,500 and reported a net income of P90,000 during the
year. Its Retained Earnings at the beginning of the year is P125,000.

1. At what amounts will HHH report in its December 31, 2018 balance sheet the Investment in Joint
Venture and Retained Earnings accounts, respectively?
A. P629,500 and P251,000 C. P692,500 and P282,500
B. P625,900 and P250,100 D. P652,900 and P201,500

Solution:

Investment in JV: Retained Earnings:


Initial Investment 625,000 Beginning 125,000
Share in Profit (180,000/2) 90,000 Income 180,000
Dividend Received (45,000/2) (22,500) Dividends Declared (22,500)
692,500 RE, Ending 282,500 PROBLEM 8 (PRTC)

TRINA and BELLA in a joint venture, contributed P30,000 each in order to purchase merchandise
which were sold in lots at a closing-out sale. They agreed to divide their profits equally and each
shall record her purchases, sales, and expenses in her own books. After almost all merchandise
had been sold, they wind up their venture.

The following are the venture transactions: TRINA BELLA


Purchases of merchandise P30,000 P30,000
Expenses paid from Jt Venture
Cash 3,000 3,900
Jt venture credit balances (24,000) (21,000)
Undisposed merchandise upon termination of JV 900 1,400
All transactions for the joint venture are in cash. The venturers are to take over the unsold merchandise
at cost.

1. Calculate the net profit of the joint venture undertaking


A. P 47,300 C. P 24,900
B. P 54,900 D. P 30,000

2. Determine the amount of cash BELLA would receive/ (pay) from/ to TRINA upon final cash
settlement by the venturers.
Sales (57,000+54,900) 111,900
COS: Purchases 60,000
End. Inventory (900 + 1,400) (2,300) 57,700
Gross Profit 54,200
OPEX (6,900)
Net Income 47,300

Cash Settlement Trina Bella


Cash Due
Share in NI (42,300/2) 23,650 23,650
Investments 30,000 30,000
Unsold Merchandise (900) (1400)
A. P(1,250) C. P(2,150)
B. P 2,150 D. P 1,250

Solution:
Joint Venture - Trina Joint Venture - Trina
Purchase 30,000 57 ,ooo Sales Purchase 30,000 54 ,9oo Sales
Expenses 3,000 Expenses 3,900
33,000 57,000 33,900 54,900
24,000 24,000

,750 52,250
Cash on hand (57,000 - 3,000) (54000)
(54,900 - 3,900) (51000)
Cash paid/received (1,250) 1,250

PROBLEM 9 (PRTC)

JRU CORPORATION, a joint venturer with a 50% equity in Joint Venture ABC INCORPORATED,
prepared the following draft of its combined financial statements at December 31, 2018 before
the year-end adjustments under the equity method.
Revenues P10,800,000
Expenses 9,280,000
Profit 1,520,000
Ordinary shares 3,000,000
Retained earnings 920,000
Liabilities 840,000
Totals P6,280,000

Current assets P1,830,000


Plant assets 3,900,000
Accumulated Deprn (700,000)
Investment in JV 1,250,000
Totals P6,280,000

Joint venture ABC reported a net profit of P115,000 for the year ended December 31, 2018.
1. Determine the total assets that will be shown in the balance sheet of JRU CORPORATION
at December 31, 2018.
A. P5,030,000 C. P6,280,000
B. P6,337,500 D. P5,280,000

Solution:
Current Assets 1,830,000
Plant Assets, net 3,200,000
Inv. In JV [1,250,000 + (115,000*50%) 1307500

Total Assets 6,337,500

2. Determine the total stockholders' equity that will be shown in the balance sheet of JRU
at December 31, 2018.
A. P4,190,000 C. P5,497,500
B. P5,440,000 D. P4,440,000
Solution:
Share Capital 3,000,000

RE, 1/1 920,000

Profit Share from own operations 152,000

Profit share from JV 57,500 2,497,500


Shareholder's Equity 5,497,500

PROBLEM 10 (PRTC)

• On January 1, 2018 SME JV acquired a 35% equity of Z CORPORATION for P37,000,


SME JV shares in the joint control over the strategic financial and operating
decisions of Z CORPORATION. Transactions costs of 5% of the purchase price of the
shares were incurred by SME JV.
• On December 31, 2018 Z CORPORATION declared and paid a dividend of P24,000.
Z CORPORATION recognized a profit of P18,000 for that year.
• Published price quotations do not exist for the shares of Z CORPORATION. Using
appropriate valuation techniques SME IV determined the fair value of its investment
in Z CORPORATION at December 31, 2018 as P49,000. Costs to sell are estimated
at 9% of the fair value of the investments. SME A does not prepare consolidated
financial statements because it does not have any subsidiary.

1. What is the profit (loss) of SME JV to be presented in the income statement for Z
CORPORATION using the fair value method?

A. PP20,400 C. P15,990
B. P18,550 D. P14,140

Solution:
Transaction cost (exp. 37,000 x 5%) (1850)
Dividend income (24,000 x 35%) 8,400
Gain on FVR (49,000 - 37,000) 12,000

Net gain 18,550

2. What is the profit (loss) of SME IV to be presented in the income statement for Z
CORPORATION using the cost model?
A. P(8,575) C. P 5,250
B. P 8,400 D. P (1,750)

Solution:

Dividend income (24,000 x 35%) 8,400

3. What is the investment balance of SME JV at the end of the year in Z CORPORATION
using the fair value model?
A. P 52,325 C. P49,000
B. P 57,575 D. P 47,075
4. What is the investment balance of SME JV at the end of the year in Z CORPORATION
using the equity model?
A. P38,850 C. P 34,125
B. P42,525 D. P 36,750

Solution:
Investment Cost 37,000
Transaction Cost 1,850
Dividend Income -8,400
Share of Profit (18,000 x 35%) 6,300

Carrying Value, 12/31 36,750

PROBLEM 11 (CRC-ACE)

Barnes and Carter join in a venture for a sale of football souvenirs at the Rose Bowl Games
Partners agree to the following: 1) Barnes must be allowed a commission o 10% on net
purchases, 2) members shall be allowed a commission of 25% on the respective sales, 3) any
remaining profit shall be shared equally, Venture transactions follows:

Dec-20 Barnes make cash purchase, P9,500


Jan-01 Carter pays venture expenses, P1,500
Jan-01 Sales are follows: Barnes, P8,000; Carter, P6,000
(members kept their own cash receipts)
Jan-06 Barnes returns unsold merchandise and receives cash of
P2,500 on the return
Jan-06 The partners makes cash settlement
REQUIRED: Separate books for the venture are not kept. What entries would be made
on the books of Barnes and Carter?

ANSWER:

Barnes Carter
Joint venture 9,500 Joint venture 9,500
Cash 9,500 Barnes 9,500

Joint venture 1,500 Joint venture 1,500


Carter 1,500 Cash 1,500

Sales 8,000 Cash 6,000


Carter 6,000 Barnes 8,000
Joint venture 14,000 Joint venture 14,000

Cash 2,500 Barnes 2,500


Joint venture 2,500 Joint venture 2,500

Joint venture 5,500 Joint venture 5,500


Inc. 3,350 Inc. 2,150
Carter 2,150 Barnes 3,350

PROBLEM 12 (CRC-ACE)

On January 1, 2018 entities A and B (the venturers) form a Joint venture (entity X). upon
incorporation of entity X, entities A and B each take up 50 per cent of the share capital of entity
X. In return or their interests in entity X entities A and B each contribute P1000,000 and a
carrying amount of P80,000. Entity B’s contribution is P100,000 cash.
The machine contributed by entity A has an estimated useful life of 10 years with no residual value.

Entity X’s profit for the year ended December 31, 2018 is P300,000 (after deducting depreciation
expense of P10,000 on the machine contributed by entity A). Entity A accounts for his investment
using the equity method.
What is the cost of investment of entity A on December 31, 2018

ANSWER:
Investment of Machine, January 1, 2018

Carrying amount 80,000


Realized gain (P100,000-P80,000)50% 10,000 90,000
Proit shares (50%xP30,000) 15,000
Realized gain on machine (P10,000/10yrs) 1,000
Investment account balance, December 31, 2018 106,000

PROBLEM 13 (CRC-ACE)

On March 1, 2018 entities A and B each acquired 30% of the ordinary shares that carry voting
rights at a general meeting of shareholders of entity Z for P300,000. Entities A and B immediately
agreed to share control over entity Z.

On December 31, 2018 entity Z declared a dividend of P100,000 for the year 2018.
Entity Z reported a profit of P60,000 for the year ended December 31, 2018. At
December 31, 2018 the recoverable amount of each venturer’s investment in entity Z is
P292,000 (fair value of P295,000 less cost to sell of P3,000). Entities A and B uses the equity
method to account for its investment in entity Z. However, there is no published price quotation
for entity Z.

On December 31, 2018, entities A and B must each report its investment in entity Z at:

ANSWER:
Cost of investment 300,000
Profit share (10/12xPP60,000)x30% 15,000
Dividend income (30%xP100,000) (30,000)
Investment in entity Z, December 31, 2018 285,000

PROBLEM 14 (CRC-ACE)

On January 1, 2018 entities M and N each acquired 30% of the ordinary shares that carry voting
rights at a general meeting of shareholders of entity Z for P300,000. Contingent consideration
probable to the paid by entity M is measured reliably at P50,000. Entities M and N immediately
greed to share control over entity Z.

For the year ended December 31, 2018 entity Z recognized a profit of P400,000. On December
30, 2018 entity Z declared and paid a dividend of P150,000 for the year 2018. At December 31,
2018 the fair value of each venturers’ investment in entity Z is P425,000. However, there is no
published price quotation for entity Z.

On December 31, 2018 entity M sells goods for P60,000 to entity Z. at December 31, 2018 this
goods were in the inventories of equity Z. entity M sells goods at a 50 per cent mark-up on cost.
entities M and N account for its investment in entity Z using the equity method.
At December 31,2 108 entity M would report its investment in entity Z at?

ANSWER:

Cost of investment, Jan. 1, 2018(P300,000+P50,000) 350,000


Profit share (30%xP400,000) 120,000
Unrealized profit (50/150 x P60,000) 30% (6,000)
Dividend income(30%xP150,000) (45,000)
Investment in entity Z, Dec. 31, 2018 419,000

PROBLEM 15 (CRC-ACE)

On January 1, 2018 entities A and B each acquired 30% of ordinary shares that carry voting
rights at a general meeting of shareholders of entity M for P100,000. The purchase price is equal
to the fair value of 30% of entity M’s identifiable assts less 30% of its identifiable liabilities.

Entities A and B immediately agreed to share control over entity M.

For the year ended December 31, 2018 entity M recognized a loss of P600,000. Entities A and B
have no constructive or legal obligation with respect of their jointly controlled entity’s loss and
have made no payments on its behalf.

Entity M recognized profit for the year ended December 31, 2018 of P800,000. There is no
published price quotation for entity M. investments are accounted for using the equity method.

At December 31, 2018 how much investment in entity M should be reported by each venture?

ANSWER:
Cost of investment 100,000
Loss share (100,000)
Investment in entity M, Dec. 31, 2018 0
4.1 Revenue from Contracts with Customers

Problem 1 (ReSA)

Hold Industries received a P2,000 prepayment from the Ramirez Company for the sale of new
office furniture. Holt will bill Ramirez an additional P3,000 upon delivery of the furniture to
Ramirez. Upon receipt of the P2,000 prepayment, how much should Holt recognize for a contract
asset, a contract liability and accounts receivable?

a. Contract asset: 0; contract liability, P2,000; accounts receivable, 0


b. Contract asset: 0; contract liability, 0; accounts receivable, 0
c. Contract asset: P2,000; contract liability, 0; accounts receivable, 0
d. Contract asset: 0; contract liability, 0; accounts receivable, 2,000

Answer: (A) – Holt has a contract liability, deferred revenue of P2,000. It never has a contract
asset because it hasn’t satisfied a performance obligation for which payment depends on
something other than passage of time. It does not have an accounts receivable for the P3,000
until it delivers the furniture to Ramirez.

Problem 2 (ReSA)

On January 15, 2015, Bella Vista Company entered into a contract to build custom equipment
for ABC Carpet Company. The contract specified a delivery date of March 1. The equipment was
not delivered until March 31. The contract required full payment of P75,000 30 days after
delivery. This contract should be:

a. Recorded on January 15, 2015 c. Recorded on March 31, 2015


b. Recorded on March 1, 2015 d. Recorded on April 30, 2015

Problem 3 (ReSA)

Pampanga Communications contracted to set up a call center for the City of San Fernando. Under
the terms of the contract, Pampanga Communications will design and set up a call center with
the following costs:

Computers, servers, telephone equipment 10,000


Computers, servers, telephone equipment 275,000
Software 85,000
Installation and testing equipment 15,000
Selling commission 25,000
Annual service contract 50,000

In addition, Pampanga Communications will maintain and service the equipment and software to ensure
smooth operations of the call center for an annual fee of P90,000.
Ownership of equipment installed remains with the City of San Fernando. The contract costs that
should be capitalized is

a. 460,000 c. 360,000
b. 410,000 d. 370,000

Solution:

Computers, servers, telephone equipment 10,000


Computers, servers, telephone equipment 275,000
Software 85,000
Installation and testing equipment 15,000
Selling commission 25,000

410,000

Problem 4 (ReSA)

On October 1, 2016, Acme Fuel Co. sold 100,000 gallons of healing oil to Karn Co. at P3 per
gallon. Fifty thousand gallons were delivered on December 15, 2016 and the remaining 50,000
gallons were delivered on January 15, 2017. Payment terms were 50% due on October 1, 2016,
25% due on first delivery, and the remaining 25% due on second delivery. What amount of
revenue should Acme recognize form this sale during 2017?

a. 75,000 c. 225,000
b. 150,000 d. 300,000

Solution:

50,000 gallons x P3 = 150,000

Problem 5 (ReSA)

On June 1, 2015, Johnson & Sons sold equipment to James Landscaping Services. In exchange
for a zero-interest bearing note with a face value of P55,000 with payment due in 12 months.
The fair value of the equipment on the date of sale was P50,000. The amount of revenue to be
recognized on this transaction in 2015 is

a. 55,000 c. 50,000
b. 5,000 d. 50,000 sales revenue and 2,917
interest revenue

Solution:

(55,000-50,000) x 7/12 = 2,917


PROBLEM 6 (PRTC)

1. Which of the following is typically true for a bill-and-hold arrangement? A.


Revenue is recognized when goods are manufactured.
B. Revenue is recognized when the arrangement is made.
C. Revenue is recognized when the delivery of goods is made.
D. Revenue is recognized at the point in time at which payment from the
customer is received.

Answer: Bill-and-hold arrangements normally do not quality for revenue recognition until
delivery is made to the customer. Prior to that point, control of goods usually is not viewed as
having passed to the customer.

PROBLEM 7 (PRTC)

1. On June 1st, Joseph & Company received a P500 deposit for 80 cases of wine. On June 10 th
the customer identified specific vintages that are included in Joseph's inventory, and asked that
Joseph not ship the wine until June 20 so the customer could ready space to store the wine so,
Joseph set those wines aside for the customer, boxed and ready for shipment to the customer.
On June 20th the wine was shipped and delivered to the customer. Joseph likely would recognize
revenue on:
A. June 20th C. June 1st
th
B. June 10 D. Upon consumption of the wine by the customer

Answer: Bill-and-hold arrangements normally do not qualify for revenue recognition until
delivery is made to the customer. Prior to that point, control of goods is not viewed as having
passed to the customer. However, sellers can recognize revenue prior to delivery if it is concluded
that the customer controls the product (the customer specifically identified the goods), there is
good reason for the bill-and-hold arrangement (the customer needed time to make space for the
wine), and the product is specifically identified as belonging to the customer and is ready for
shipment (Joseph has a good faith deposit, the customer selected the goods, the goods were
prepared for shipment and set aside from regular goods for sale).

PROBLEM 8 (PRTC)
1. Horowitz Paint Shop sold P3,000 of paint to a local construction company for cash on June 25,
20x6. Because of a flood in the area, the customer requested that Horowitz not ship the items
from its warehouse until July 3, 20x6, so Horowitz set aside the paint on June 25, packaged and
ready to ship on July 3.For the second quarter ending on June 30, how Horowitz recognize for
the sale to the local construction company?
A. No contract exists C. P1,500
B. Zero D. P3,000

Answer: P3,000. In a bill-and-hold arrangement, the key issue normally is that the customer
does not have physical possession of the asset until the seller has delivered it. However, since
the customer requested that Horowitz hold the goods, has been paid for the goods, and the
goods are separated from Horowitz's inventory and ready for shipment, Horowitz likely would be
viewed as shifting control to the customer in June.

PROBLEM 9 (PRTC)

Ralf Laurentii’s Perfume, Inc., sold 3,210 boxes of white musk soap during January of 20x6 at
the price of P90 per box. The company offers a full refund for any product returned within 30
days from the date of purchase. Based on historical experience, Ralf Laurentii’s Perfume expects
that 3% of sales will be returned.
1. How many performance obligations are there in each sale of a box of soap
A. No contract exist C. 2
B. 1 D. 3

Answer: Number of performance obligations in the contract: 1. A right of return is not a


performance obligation. Instead, the right of return represents a potential failure to satisfy the
original performance obligation to deliver goods to the customer. Because the total amount of
cash received from the customer depends on the amount of returns, a right of return is a type
of variable consideration.
PROBLEM 10 (PRTC)

Taster Choice sell natural supplements to customers with an unconditional right of return if they
are not satisfied. The right of returns extends 60 days. On February 10, 20x4, a customer
purchases P3,000 of products (cost P1,500). Assuming that based on prior experience, estimated
returns are 20%.

1. The journal entry to record the sale and cost of goods sold includes a A.
debit to Cash and a credit to Sales Revenue of P3,000.
B. credit to Refund Liability of P600 and a credit to Sales Revenue of
P2,400.
C. debit to Cost of Goods Sold and credit to Inventory for P1,500.
D. credit to Estimated Inventory Returns of P300

Answer: P3,000 x 2 = P600; P3,000-P600 = P2,400.

2. The journal entry to record the return of P200 of merchandise includes a


A. credit to Refund Liability for P200.
B. credit to Returned Inventory for P100.
C. credit to Estimated Inventory Returns for P100.
D. debit to Estimated Inventory Returns for P100.

Answer: P1,500/ P3,000 = 5: P200 x .5 = P100.

PROBLEM 11 (CRC-ACE)

Assume that a customer enrolls in AAA's Premier Membership, which provides 12 months 01
roadside assistance for P120. On August I. 20x6, a customer purchases a contract that runs from
that date through July 31, 20x7. Given that roadside assistance requests occur equally
throughout the contract period, AAA uses "proportion of time" as its measure of progress toward
completion. The amount of sales revenue on August 1?

a. Zero c. P120
b. P10 d. P1,140

Answer:

August 1

Cash 120

Deferred revenue 120

PROBLEM 12 (CRC-ACE)

Lux Hotels, Inc. has signed a service outsourcing contract with Deluxe Rooms,
Inc. for P3 million, which was received in cash at contract inception. Under the agreement.
Deluxe Rooms is obligated to clean and prepare over 5.000 hotels rooms managed by Lux Hotel
on a daily basis from August 1, 20x6 to July 31. 20x7.

When should Lux Hotels recognize revenue?

a. No transaction c. Point in time


b. No revenue d. Over time

Answer:

This service contract qualifies for revenue recognition over time, because the customer consumes the
benefit of the seller's work as it is performed.

PROBLEM 13 (CRC-ACE)

On February 1st H&B Bank originated a loan for P50,000 at an interest rate of 7.2%. On March
15th. an interest payment of P300 was received. Which of the following best describes when
interest revenue should be recognized?

a. At a point in time feb 1st c. At a point in time (march 31st)


b. At a point in time march 15th d. Over time

ANSWER

This announcement qualities for revenue recognition over time because the customer consumes the
benefit of

the setters service as the seller provides it

PROBLEM 14 (CRC-ACE)

On January 1, the Cost Driver Company, a consulting firm, entered into a three-month contract
with Coco Seafood Restaurant to analyze its cost structure in order to find a way to reduce
operating costs and increase profits. Cost Driver promises to share findings with the restaurant
every two weeks and to provide the restaurant with a final analytical report at the end of the
contract. This service is customized to Coco. and Cost Driver would need to start from scratch it
provided a similar service to another client. Coco promises to pay P5.000 per month. If Coco
chooses to terminate the contract, it is entitled to receive a report detailing analyses to that
stage. When should revenue be recognized?

a. No transaction c. Point in time


b. No revenue d. Over time

Answer:

The team of the contract and on the related facts and circumstances Indicate that Coco has the
ability to direct the use or, and receive the benefit born. the consulting services as they are
performed. The restaurant has on unconditional obligation to pay throughout the contract as
evidenced by the nonrefundable progress payments, and the right to a report regardless of
contract terminated Also the report has no alternate use to Cost Driver. Therefore, the Cost
Driver Company's performance obligation is to provide the restaurant with services continuously
during the three Months of the contract, and Cost Driver should recognize revenue over the life
of the contract.

PROBLEM 15 (CRC-ACE)
Squeaky Shine provides car washing services in Sampaloc, Manila. A threemonth pass for
automatic car wash sells for P60. which entitles the customer for an unlimited number of car
washes during the contract period. Squeaky Shine estimates that pass holders wash their cars
equally throughout the three-month period. On December 1st. customers purchased P1,260 of
the three-month passes. with purchases of the possess occurring evenly throughout December.
The amount of soles revenue on December 1:

a. Zero c. 210
b. P60 d. P1,260

Answer:

December 1 entry

Cash 1,260

Deferred revenue 1,260

4.2 Long Term Construction Contracts

Problem 1 (ReSA)
DJ Builders Construction enters into a contract with a customer to build a warehouse for
P850,000 on March 30, 2015 with a performance bonus of P50,000 if the building is completed
by July 31, 2015. The bonus is reduced by P10,000 each week that completion is delayed. DJ
Builders commonly includes these completion bonuses in its contracts and based on prior
experience, estimates the following completion outcome:

Completed by Probability
July 31, 2015 65%
August 7, 2015 25%
August 14, 2015 5%
August 21, 2015 5%

The transaction price for this transaction is:

a. 895,000 c. 585,000
b. 850,000 d. 552,500

Solution:

900,000x65% 585,000
890,000x25% 222,500
880,000x5% 44,000
870,000x5% 43,500
895,000

Problem 2 (ReSA)

Jarflloydee Construction Company enters into a contract with a customer to build a 50 kilometers
road for P100 million with a performance bonus of P60 million that will be paid based on the
timing of completion. The amount of the performance bonus decreases by 10% per week for
every week beyond the agreed upon completion date. The contract requirements are similar to
contracts that Jarflloydee has performed previously and management believes that such
experience is predictive for this contract. Management estimates that there is a 60% probability
that the contract will be completed by the agree upon completion date, a 30% probability that it
will be completed one week late and only 10% probability that it will be completed two weeks
late. Determine the probability-weighted amount for the management to determine transaction
price?
a. 96 million c. 142.2 million
b. 111 million d. 157 million

Solution:

160m x 60% 96 million


154m x 30% 46.2 million
148m x 10% 14.8 million
157 million

Problem 3 (ReSA)

AJD Company recognizes construction revenue and expenses using the percentage of completion
method. During 2014, a single long term project was begun which continued through 2005.
Information on the project were as follows:

2014 2015

Accounts Receivable from


construction contract 200,000 600,000
Construction expenses 210,000 384,000
Construction in progress 244,000 728,000
Partial billings on contract 200,000 840,000

The profit recognize form the long-term construction contract should amount to:

a. 2014, P44,000; 2015, P456,000


b. 2014, P44,000; 2015, P200,000
c. 2014, P34,000; 2015, P256,000
d. 2014, P34,000; 2015, P100,000

Solution:

Construction in Progress

CI in 2014 210,000

RGP in 2014 34,000


244,000
CI in 2015 384,000

RGP in 2015 100,000

End of 2015 728,000

Problem 4 (ReSA)
Chicane Builders, Inc. employs the cost to cost method in determining the percentage of completion
for revenue recognition. The company’s record show the following information on a recently
completed project for a contract price of P5,000,0000

2014 2015 2016


Cost incurred to date 900,000 2,550,000 ?

Gross Profit (loss) 100,000 350,000 (50,000)

1. The estimated cost to complete project at December 31, 2015:


a. 850,000 c. 2,300,000
b. 1,700,000 d. 2,550,000

2. The actual cost incurred during the year 2016


a. 2,550,000 c. 2,200,000
b. 2,300,000 d. 2,050,000

Solution:

2014 2015 2016


Contract Price 5,000,000 5,000,000 5,000,000
Cost incurred each year 2,050,000
Add: Cost incurred in prior year 900,000 2,550,000
Cost incurred to date 900,000 2,550,000 4,600,000
Add: Estimated cost to complete 3,600,000
1,700,000 - 4,500,000 4,250,000 4,600,000
Total Estimated Cost
500,000 750,000 400,000
Estimated Gross Profit
20% 60% 100%
Multiply by: percentage of completion
Recognized gross profit to date 100,000 450,000 400,000
Less: Recognized gross profit in prior years
0 (100,000) (450,000)
Recognized gross profit each year
100,000 350,000 (50,000)

Problem 5 (ReSA)

Seasons Construction is constructing an office building under contract for Cannon Café. The
contract calls for progress billing and payments of P620,000 each quarter. The total contract
price is P7,440,000 and Seasons estimates total costs of P7,100,000. Seasons estimates that
the building will take 3 years to complete and commences construction on January 2, 2014.

At December 31, 2014, Seasons estimates that it is 30% complete with the construction, based
on cost incurred. What is the total amount of Revenue from Long term contracts recognized for
2014 and what is the balance of accounts receivable account assuming Cannon Café has not yet
made its last quarterly payment?
Accounts
Revenue Receivable

a. 2,480,000 2,480,000
b. 2,130,000 620,000
c. 2,232,000 620,000
d. 620,000 2,480,000

Solution:

Revenue: P7,440,000x30 = 2,232,000 A/R = 620,000

PROBLEM 6 (PRTC)

STRONG CONSTRUCTIONS uses the percentage of completion method in recogr. 9 income. In


2016, STRONG was engaged by SM on a fixed-price-contract to build a 3storey shopping mall.
On January 1, 2018, a fire damaged the accounting records of STRONG CONSTRUCTIONS The
president of the company has contracted you to reconstruct the contract information
The following data were taken from the salvaged files:
31-Dec
2016 2017
Architect's estimated total cost of completion P7,500,000 P8,000,000
Costs incurred 3,000,000
Percentage of completion 60%
Income recognized to date 500,000 1,200,000

Compute for the percentage completed in 2016 on the SM shopping mall.


2017 2016 A. 40%
C.
Contract Price 10,000,000 10,000,000
20%
Total Cost (8,000,000) (7,500,000)
B. 25%
Gross Profit 2,000,000 2,500,000 D.
30%

Solution:

GP x % of Completion = RGP
GP = RGP/% of Completion
= 1,200,000/60%
= 2,000,000
GP x % of Completion = RGP
% of Completion = RGP/GP
= 500,000/2500,000
= 20%

PROBLEM 7 (PRTC)

On January 2, 2018, QUICKBUILD ERECTORS entered into contract to construct two projects. The
following data relate to the construction activities.
Proiect A Project B
Contract price P945,000 P675,000
Cost incurred during 2016 540,000 630,000
Estimated costs to complete 270,000 157,500
Billings to customer 337,500 607,500

1. What amount of gross profit should QUICKBUILD ERECTORS report in its 2018 income statement
under the following methods?
Percentage of Zero Profit
Completion Method Method
A. P (0) P (90,000)
B. P (112,500) P (22,500)
C. P ( 22,500) P (0)
D. P ( 22,500) P(112,500)

Solution:
Project A Project B
Contract Price 945,000 675,000
Actual Cost 540,000 630,000
Estimated Cost to Complete 270,000 157,500
Estimated Total Cost 810,000 787,500
GP/GL 135,000 787,500
% 66.67% 100%
% of Completion 90,000 + (112,500) = (22,500)
Zero Profit 0 + (112,500) = (112,500) PROBLEM 8 (PRTC)

BEST - EVER CONSTRUCTION, INC recognizes construction revenue and costs using the
percentage of completion method. During 2017, a single long-term project was begun which
continued through 2018. Information on the project follows:
2017 2018
Accounts receivable P350,000 P1,050,000
Incurred costs during year 367,500 672,000
Construction in progress 427,000 1,274,000
Billings on contract 350,000 1,470,000
The construction accounts are at amounts to-date.

1. What is the gross profit recognized from this long-term contract?


2017 2018

A. P 77,000 P 798,000
B. 77,000 350,000
C. 59,500 448,000
D. 59,500 175,000

Solution:

2017 (427,000 - 367,500) 59,500


2018 (1,274,000 - 427,000 - 672,000) 175,000

PROBLEM 9 (PRTC)

The SKYVIEW CORPORATION started work on three contracts during 2018. Data relating to the
three jobs are:

Contract Contract Contract Contract Contract


Price Price Price Price Price

Contract 1 P400,000 P300,000 - P400,000 P320,000


Contract 2 560,000 80,000 320,000 80,000 40,000
Contract 3 200,000 80,000 80,000 - -

1. Calculate the amount of Construction in Progress to be reported in the year-end balance sheet under
percentage of completion.
A. P132,000 C. P460,000
B. P212,000 D. P628,000
Solution:
Contract 2 Contact 3
Contract Price 560,000 200,000
Actual Cost 80,000 80,000
Est. Cost to Complete 320,000 80,000
Est. Total Cost 400,000 160,000
Gross Profit
160,000 40,000
% 20% 50%

C2 (560,000 x 20%) 112,500


C3 (200,000 x 50%) 100,000

Total 212,500

PROBLEM 10 (PRTC)

RAINBOW, INC., a construction company, has a P8,000,000 contract that was started in 2016. The
following information is provided for the construction activities.
Construction Actual cost Est cost to complete
Years incurred to-date at year-end
2016 P1,024,000 P4,096,000
2017 3,993,600 2,246,400
2018 6,473,600 0

1. Calculate the amount of gross profit to be reported for 2017 under percentage of completion method:
A. P1,126,400 C. P576,000
B. P 550,400 D. P480,000

Solution:
2017 (8,000,000 x 64%) - 3,993,600 1,126,400
2016 (8,000,000 x 20%) - 1,024,000 -576,000
Profit recognized in 2017 550,400
PROBLEM (CRC-ACE)

11

The following information pertain to the building contract of DMCI Construction Company,
wherein the fixed contract price is P80 million.

2016 2017 2018


Estimated costs 20.1 million 30.15 million 16.75 million
Progress billings 10 million 25 million 45 million
Cash collection 8 million 23 million 49 million

Assume that all costs are incurred, all billings to customers are made, and all collections
from customers are received within 30 days of billing, as planned. Under the percentage-
of-completion method revenue recognition is used, how much is the income from
construction for the year 2018?

ANSWER:

Total Contract Price 80,000,000


Total Estimated costs

2,016 20,100,000

2,017 30,150,000

2,018 16,750,000 67,000,000


Estimated gross profit 13,000,000
2018 gross profit:
16750000/67000000 x 13000000 = 3,250,000

PROBLEM 12 (CRC-ACE)

Philip Construction Company started a project with a contract price of P80 million. The cost
incurred to date is P12 million and the estimated cost to complete is still P48 million. Under
the cost to cost basis, how much is the income from construction?

ANSWER:

Contract Price 80,000,000


Total estimated cost
Cost
incurred 12,000,000
PROBLEM (CRC-ACE)

Estimated cost to complete 48,000,000 60,000,000


Estimated gross profit 20,000,000
Percentage of completion (12,000,000/60,000,000) x 20%
Income from construction 4,000,000

13

Cameron Company entered into a contract to build a small bridge for Agdao. The contract
price for the bridge was P7,500,000 and Cameron estimated a total cost of P6,900,000 in
2018. The company incurred P2,300,000 of costs during real costs. The estimated total
cost o project skyrocketed to P7,800,000. Construction cost incurred in 2019 totaled
P4,000,000. The project was completed in 2019 at a final cost of P7,800,000. No progress
billings were made under the contract and no cash was collected by the end of 2019.

The amount of gross profit (loss) that must be recognized in 2019 must be:

ANSWER:

Contract Price (fixed) 7,500,000


Total estimated cost 7,800,000
Anticipated loss to date (300,000)
Add: Gross profit recognized in 2018:
Contract
Price 7,500,000
Total estimated cost 6,900,000
Estimated gross profit 600,000
Percentage of
completion (2.3/6.9) x 1/3 (200,000)
Total loss recognized in 2019 (500,000)

PROBLEM 14 (CRC-ACE)

Clarence Construction has consistently used the percentage-of-completion method. On


January 10, 2018, Clarence began work on P3,000,000 construction contract. At the
inception date, the estimated cost o construction was P2,250,000. The following data relate
to the progress of the contract:

Income recognized at December 31, 2018 300,000


Cost incurred January 10, 2018 through Dec. 31, 2019 1,800,000
Estimated cost to complete, December 31, 2019 600,000
PROBLEM (CRC-ACE)

In its income statement for the year ended Dec. 31, 2019, what amount or gross profit
should Clarence report?

ANSWER:
Gross profit to date:
Contract price 3,000,000
Total estimated costs (1,800,000 + 600,000) (2,400,000)
Estimated gross profit 600,000
Percentage of completion (1.8/2.4) x 75% 450,000
Less: Gross profit in prior year, 2018 (300,000) Gross profit this year, 2019
150,000 15

Jason Construction, Inc. has consistently used the percentage-of-completion method of


recognizing income. During 2019 Jason started work on a P3,000,000 fixed-price
construction contract. The accounting records disclosed the following data or the year
ended Dec. 31, 2019:
Cost incurred 930,000
Estimated cost to complete 2,170,000 Progress billings
1,100,000
Collections 700,000
How much loss should Jason have recognized in 2019?

ANSWER:

Contract price (fixed) 3,000,000

Total estimated costs:

Cost incurred to date 930,000

Add: Estimated cost to complete 2,170,000 ( 3,100,000)

Gross profit (loss) recognized ( 100,000)


PROBLEM (CRC-ACE)
4.3 Franchise Operations

Problem 1 (ReSA)

Frozen Delight, Inc. charges an initial franchise fee of P75,000 for the right to operate as
a franchisee of Frozen Delight. Of this amount P25,000 is collected immediately. The
remainder is collected in four equal annual instalment of P12,500 each. These instalments
have a present value of P41,402. As part of total franchise fee, Frozen Delight also
provides training (with a fair value of P2,000) to help franchisees get the store ready to
open. The franchise agreement is signed of April 1, 2015, training is completed, and the
store opens on July 1, 2015.

1. The amount of revenue from training and franchise on April 1, 2015 to:
a. Zero c. 66,402
b. 64,402 d. 75,000

2. The amount of revenue from training and franchise on July 1, 2015 to:
a. Zero c. 66,402
b. 64,402 d. 75,000

Solution:

1.

April 1, 2015
Cash 25,000
Notes Receivable (75k-25k) 50,000
Unearned Interest Income 8,598
Unearned Service Revenue (training) 2,000
Unearned Service Revenue (franchise) 64,402
(25,000+41,402-2,000)

2.

July 1, 2015
Unearned Service Revenue (training) 2,000
Unearned Service Revenue (franchise) 64,402
Franchise Revenue 64,402
Service Revenue (training) 2,000 Problem 2 (ReSA)

Wynne Inc. charges an initial franchise fee of P1,840,000 with P400,000 paid when the
agreement is signed and the balance in five annual payments. The present value of the
future payments, discounted at 10% is P1,091,744. The franchisee has the option to
purchase P240,000 of equipment for P192,000. Wynne has substantially provided all initial
services required and collectability of the payments is reasonably assured. The amount of
revenue from franchise fees:

a. 400,000 c. 1,491,744
b. 1,443,744 d. 1,840,000

Solution:

(400,000+1,091,744-(240,000-192,000) = 1,443,744

Problem 3 (ReSA)

Pasta Inn charges an initial franchise fee of P1,600,000 for a franchise, with P320,000
paid when the agreement is signed and the balance in four equal annual payments. The
present value of the annual payments, discounted at 10% is P1,014,000. The franchisee
has the right to purchase P60,000 of kitchen equipment and supplies for P50,000. An
additional part of the initial fee is for advertising to be provided by Pasta Inn during the
next five years. The value of advertising is P1,000 a month. Collectability of the payments
is reasonably assured and Pasta Inn has performed all the initial services required by the
contract. How much revenue from franchise fee to be recognized when the agreement is
signed?

a. Zero c. 1,590,000
Total Franchise Fee 1,600,000
Less: Unearned Interest Income
Amount due 1,280,000
Less: Present value of payments 1,014,000 (266,000)
b. 1,264,000 d. 1,600,000

Solution:

Bargain purchase option (60,000-50,000) (10,000)


Advertising (1,000x60 months) (60,000)

Revenue from Franchise Fee 1,264,000 Problem 4 (ReSA)

Pacific Crossburgers Inc. charges an initial franchise fee of P70,000. Upon the signing of
the agreement (which covers 3 years), a payment of P28,000 is due. Thereafter, three
annual payments of P14,000 is required. The credit rating of the franchisee is such that it
would have to pay interest at 10% to borrow money. The franchise agreement signed on
May 1, 2015 and the franchise commences operation on July 1, 2015.
1. The amount of franchise revenue on May 1, 2015 assuming no future services are
required by the franchisor once the franchise starts operations:
a. Zero c. 62,816
b. 28,000 d. 70,000

2. In relation to No. 2, the amount of franchise revenue on July 1, 2015:


a. Zero c. 62,816
b. 28,000 d. 70,000

Solution:

1. May 1, 2015

Cash 28,000
Notes Receivable 42,000
Discount on notes receivable 7,184
Unearned Franchise Revenue 62,416

2.

July 1, 2015
Unearned Franchise Revenue 62,816
Franchise Revenue 62,816

Cash Payment 28,000


Present Value (14,000x2.48685) 34,816

Franchise Revenue 62,816

Problem 5 (ReSA)

TopChop sells hairstyling franchises. TopChop receives a P50,000 from a new franchisee
for providing initial training, equipment and furnishings that have a stand-alone selling
price of P50,000. TopChop also receives P30,000 per year for use of the TopChop name
and for ongoing consulting services (starting on the date of the franchise is purchased).
Carlos became a TopChop franchisee on July 1, 2016 and on August 1, 2016, had
completed training and was open for business. How much revenue in 2016 will TopChop
recognize for its arrangement with Carlos?

a. Zero c. 65,000
b. 10,000 d. 70,000

Solution:

Yearly Fee (30,000x6/12) 15,000


Cash received 50,000
65,000

PROBLEM 6 (PRTC)

On January 1, 2018, MAXX SERVICES, INC. signed an agreement authorizing LALLA


COMPANY to operate as a franchisee over a 20-year period for an initial franchise fee of
P137,500 received when the agreement was signed. LALLA commenced operations on
August 1, 2018, at which date all of the initial services required of MAXX SERVICES had
been performed. The agreement also provides that LALLA must pay annually to MAXX a
continuing franchise fee equal to 5% of the revenue from the franchise. LALLA COMPANY's
franchise revenue for 2018 was P1,100,000.
For the year ended December 31, 2018, how much should MAXX SERVICES record as
revenue fror franchise fees with respect to the LALLA account?

A. P192,500 C. P123,750
B. P137,500 D. P 60,500

Solution:
FR-IFF 137,500
FR-CFF 55,000

Total FR 192,500
PROBLEM 7 (PRTC)

HARRYNAWA PRODUCTIONS has created a franchise based on the hit movie LORD OF PRA
NINGS. Many jumped on the LOPN bandwagon, and several franchise agreements have been
signed. At December 31, 2018 the following franchisees have open accounts with
HARRYNAWA:
YELLOW GREEN BLACK BEIGE
Paid in:
Cash 125,000 125,000 125,000 125,000
Notes (face 750k), unpaid 437,500 250,000 568,750 343,750
Franchise services completed 25% 10% 94% 100%
Probability of collection Likely Unlikely Likely Likely
Continuing franchise fee 1% of NI 1% of NI 1% of NI 1% of NI
Period of refund 1/31/2019 2/28/2019 12/31/2018 12/31/2001

1. Initial franchise fees earned from these four accounts aggregated at Dec. 31, 2018.
A. P 750,000 C. P1,162,500
B. P 1,200,000 D. P1,750,000

Solution:

No revenue shall be recognized for entities Yellow and Green since:


a. There is no substantial performance of initial services having completed 25% and 10%
for Yellow and Green, respectively.
b. Period of refund has not yet expired for both entities, thus amount paid is still
refundable.

Harrynawa Productions can recognize revenue for Black and Beige using ACCRUAL Method
since it is LIKELY that the balance will be collected. Recognition is computed as follows:

DP Notes Total
Black 125,000 750,000 875,000
Beige 125,000 750,000 875,000
Total revenue for initial revenue 1,750,000
PROBLEM 8 (PRTC)

On January 2, 2018, JELLYFISH, INC. entered into a franchise agreement with KOOKIE
COMPANY to sell their products. The agreement provides for an initial franchise fee of
P3,515,625 payable as follows: P984,375 cash to be paid upon signing of the contract and
the balance in five equal annual payments every December 31, starting December 31, 2018.
JELLYFISH signs a 15% interestbearing-note for the balance. The agreement further provides
that the franchisee must pay a continuing franchise fee equal to 3% of its monthly gross
sales. On October 31 the KOOKIE COMPANY completed the initial services required in the
contract at a cost of P1,125,000 and incurred indirect costs of P225,000. The franchise
commenced business operations on November 30, 2018. The gross sales reported to the
franchisor are November sales, P115,312 and December sales, P133,594. The first
installment payment was made on due date. Assume collection of the note is not reasonably
assured.

1. In its income statement for the year ended December 31, 2018, how much is the net income
recognized by KOOKIE COMPANY?
A. P1,216,069 C. P1,059,258
B. P 801,070 D. P 1,175,780

Solution:

DP 984,375
3,515,625 =
Note 2,531,250/5= 50,625

FR-IFF (984,375 + 2,531,250) 3,515,625


Franchise Cost -1,125,000

DGP - Franchises 2,390,625


GRP (2,390,625/3,515,625) 68%

RGP (984,375 + 506,250) x 68% 1,013,625


FR-IFF (248,906 x 3%) 7,467
Interest Revenue (2,531,250 x 15%) 379,688
Expenses -225,000

Net Income 1,175,780


PROBLEM 9 (PRTC)

On January 2, 2018, EXTREME COMPANY signed an agreement to operate as a franchisee of


BASIC PRODUCTS, INC., for an initial franchise fee of P2,500,000 for 10 years. Of this
amount, 40% was paid when the agreement was signed and the balance payable in four
semi-annual payments beginning on June 30, 2018. EXTREME signed a non-interest-bearing
note for the balance. EXTREME's rating indicates that it can borrow money at 24% on a loan
of this type. Assume that substantial services amounting to P617,500 had already been
rendered by BASIC PRODUCTS, INC.

1. If the collection of the note is not reasonably assured, the realized gross profit to be reported
by BASIC for the year ended December 31, 2018 is:

A. P1,057,076 C. P880,856
B. P855,225 D. P1,070,646

Solution:

Down payment 1,000,000


PV (375,000 x 3.04) 1,140,000
DP 1,000,000
2,500,000 =

FR-IFF 2,140,000
Franchise Cost -617,500
Note 1,500,000/4= 375,000

DGP - Franchises 1,522,500


GPR (1,522,500/2,140,000) 71.14%
PROBLEM 10 (PRTC)

RGP (1,000,000 + 238,200 + 266,784) x 71.14%) x 68% 1,070,646


PROBLEM 11

(PRTC)

On January 2, 2018 NAIKEE COMPANY signed an agreement to operate as a franchisee of


CONVERSE PRODUCTS, INC., for an initial franchise fee of P12,500,000 for 10 years. Of this
amount, P2,500,00 was paid when the agreement was signed and the balance payable in four
annual payments beginning on December 30, 2018. NAIKEE signed a non-interest bearing note
for the balance
NAIKEE's rating indicates that it can borrow money at 24% the loan of this type. Present value of
an annuity of 1 for 4 periods at 24% is 2.40.
Assume that substantial services amounting to P1,275,00 had already been rendered by CONVERSE
PRODUCIS. Indirect franchise cost paid amounted to P340,000.

1. Calculate the realized gross profit for 2018 assuming (1) collection of note is reasonably assured
or (2) collection of the note is not reasonably assured

A. (1) P 6,885,000; (2) P4,050,000


B. (1) P 7,225,000; (2) P3,026,000
C. (1) P11,225,000; (2) P4,250,000 D. (1) P 4,725,000; (2) P2,883,600

Solution:

FR-IFF DP 1,600,000
NR @ PV (1,600,000 x 2.4) 3,840,000
DP 1,600,000
2,500,000 =

Total 5,440,000
Franchise Cost -816,000
Note 6,400,000/4= 1,600,000

Gross Profit 4,624,000


GPR (4,624,000/25,440,000) 85.00%
RGP (1,600,000 + (1,600,000 - 921,600) x 71.14%) x 85% 1,936,640

Discount = PV xDR
= 3,840,000 x 24%
= 921600
PROBLEM 11 (CRC-ACE)

On January 1, 2018, Starbucks Company signed an agreement to operate as a franchisee of


perfect Pizza, Inc. for an initial franchise fee of P1,600,000 for a period of (10) years. Of this
amount P600,000 was paid when the agreement was signed and the balance payable in five
annual payments of P200,000 beginning December 31, 2018. Starbucks signed a non interest
bearing note for the balance. Starbucks rating indicates that it can borrow money at 20% for a
loan of this type. In return for the initial fee, the franchisor agrees to make a market studies,
find a location, train the employees, and perform other related services. The following
transactions describe the relationship with perfect pizza, a franchisee:

2018 Jan. 1: Entered into a franchise agreement.


April 1: completed a market study at a cost of P59,436 indirect cost
of services (general expenses), P5,000.
May 15: found suitable location. Service cost P280,000.
Nov. 15: completed training program for employees, cost P20,000.
Dec. 20 franchise outlet opened and business operations started.
Dec. 30: received the first annual payment.
Required: prepare all entries on the books of the franchisor for 2018, assuming the collection of
the note is reasonably assured.

ANSWER:

Down payment 600,000


Notes receivable 598,000 (pv2.99x200,0000)
Total franchise fee 1,198,000

JOURNAL ENTRIES
Jan. 1
Cash 600,000
N/r 1,000,000
Unearned franchise fee 1,198,000
Discount 402,000

Apr-01
Deferred franchise cost 59,436
Expense 5,000
Cash 64,436

May-15
Deferred franchise cost 280,000
Cash 280,000

Nov. 15
Deferred franchise cost 20,000
Cash 20,000

Dec. 20 NO ENTRY

Dec. 31
Collection:
cash 200,000
N/r 200,000

Disc. On N/r 119,600


Int.
income. 119,600

PROBLEM 12 (CRC-ACE)

On September 1, 2018, Goldilocks Company entered into franchise agreements with three
franchisees. The agreement required an initial fee payment of P70,000 plus four (4) P30,000
payments due every 4 months, the first payment due December 31, 2018. The interest rate is
12%. The initial deposit is refundable until substantial performance has been completed. The
following describes each agreement.
Service performed
Probability by Franchiser at Total cost incurred
Full
Franchise collection Dec. 31, 2018 to Dec. 31, 2018
A Likely Substantially 70,000
B Doubtful 25% 20,000
C Doubtful Substantially 100,000

For each franchisee, identify the revenue recognition method that you would recommend
considering the circumstances. Prepare the journal entries on the books of Goldilocks Company
to account the franchise. Assume P100,000 was received from each franchisee during the year.
ANSWER:
A.
Deferred franchise cost 70,000
Cash 70,000

B.
Deferred franchise cost 20,000
Cash 20,000

C.
Deferred franchise cost 100,000
Cash 100,000

Collection:
Cash 30,000
N/R 30,000

Interest:
Discount 4,356(4% x 108,900)
Interest income 4,356

Cash 70,000
Collection 30,000
Interest (4,356)
25,644 x 44.1%
42,179

PROBLEM 13 (CRC-ACE)
On January 2, 2018, REH signed an agreement to operate as a franchisee to SAMGYUPSALAMAT
Corp. for an initial franchise fee of P937,500 for 10 years. Of this amount P187,500 was paid
when the agreement was signed, and the balance was payable in three annual payments
beginning on December 31, 2018. REH signed a noninterest bearing not for the balance. REH’s
rating indicates that h can borrow money at 18% for a loan of this type. Assume that substantial
services amounting to P292,000 had already been rendered by SAGYUMPSALAMAT and that
indirect franchise cost of P25,500 was also incurred. PV is 2.17.

If the collection of the note is not reasonably assured, the net income for the year ended December
31, 2018 is

ANSWER:

Down payment 187,500


Notes Receivable 542,500
Total Franchise Fee 730,000

Installment
Cash 187,500
Collection 250,000
Interest (97,650) -18% x 542,500
399,850
60%
Realized Gross Profit 203,910

Operating Exp (35,500)


Net Income 276,060

PROBLEM 14 (CRC-ACE)

Each of Potter Pie Co’s. 21 new franchisees contracted to pay an initial franchise fee of P30,000.
By December 31, 2017, each franchise had paid a non-refundable P10,000 fee and signed a
note to pay P10,000 principal plus the market rate of interest on December 31,2018 and 2019.
Experience indicates that one franchise will default on the additional payments. Services for the
initial fee will be performed in 2017. What amount of net unearned franchise fees would Potter
report at Dec. 31,2019?

ANSWER:
Down payment (21 x 30,000) 630,000.00

Less: Default (2 additional payments) 20,000.00

Unearned Franchise Fee December 31, 2016 610,000.00

PROBLEM 15 (CRC-ACE)

At the beginning o the year, Zita Eat Haus got the franchise of Max, known steak house of
upscale patronage. The franchise agreement required a P500,000 franchise fee payable
P100,000 upon signing of the franchise and the balance in four annual installments starting the
end of the current year. At present value using 12% as discount rate, the four installments
would approximate P303,735. The fees once paid are not refundable. The franchise may be
canceled subject to the provisions of the agreement. Should there be unpaid franchise fee
attributed to the balance of main fee (P500,000), the same would become due and demandable
upon cancellation. Further, the franchiser is entitled to a 5% fee on gross sales payable monthly
within the first ten days of the following month. The credit investigation bureau rated Zita as
AAA credit rating. The balance of the franchise fee was guaranteed by a commercial bank. The
first year of operations yielded gross sales of P9 million. Max’s earned franchise fees from Zita
for the first year of operation, amounted:

ANSWER:

Franchise fees earned during


the year:

Initial Franchise fee earned:

Down payment 100,000

Installments 303,735

Continuing Franchise fee (5% x 9 million) 450,000 853,735


4.4 Consignment Sales

Problem 1 (ReSA)

On June 1, DD Company shipped twenty five DVD to BB View Store on consignment. The DVD
is to be sold at an advertise price of P200 per item. The cost of each DVD to the consignor is
P100. The consignor paid P75 to ship the merchandise. Commission is to be 25% of sales price.
During the month, two DVD were retuned.

On June 30, BB View Store remitted the amount due to consignor after deducting commission of
P400.

1. The amount remitted by BB View Store is:


a. 1,100 c. 1,200
b. 1,600 d. 2,000

2. The consignment profit is:


a. 370 c. 720
b. 415 d. 800

3. The cost of the inventory on consignment amounted to:


a. 1,400 c. 1,545
b. 1,550 d. 1,500

Solution:

1. Sales price (400/25%) 1,600


Commission 25% (400)

1,200

2.
1,600
Consignment Sales
Consignor's charges:
*Cost (8 units x P100) 800
Freight out (75/25units x 10) 30
Commission 400 (1,230)

Consignment Profit 370

1,600
8 tapes
* # of Units Sold 3. P200 per tape

Cost (15 units x P100) 1,500


Freigh out (75/25unitsx15) 45

1,545
Problem 2 (ReSA)

On August 5, 2015, Famous Furniture shipped 20 dining sets on consignment to Furniture Outlet
Inc. The cost of each dining was P350. The cost of shipping the dining sets amounted to P1,800
and was paid for by Famous Furniture. On December 30, 2014, the consignee reported the sale
of 15 dining sets at P850 each. The consignee remitted payment for the amount due after
deducting a 6% commission, advertising expense of P300 and installation and set up costs of
P390. The amount of cash received by Famous
Furniture is

a. 12,750 c. 11,295
b. 11,985 d. 11,685

Solution:

(15x P850) – (12,750x 6%) – 300- 390 = P11,295

Problem 3 (ReSA)
On August 5, 2015, Famous Furniture shipped 20 dining sets on consignment to Furniture Outlet
Inc. The cost of each dining was P350. The cost of shipping the dining sets amounted to P1,800
and was paid for by Famous Furniture. On December 30, 2014, the consignee reported the sale
of 15 dining sets at P850 each. The consignee remitted payment for the amount due after
deducting a 6% commission, advertising expense of P300 and installation and set up costs of
P390. The total profit on units sold for the consignor is

a. 11,295 c. 6,045
b. 4,695 d. 9,945

Solution:

P11,295 – (15 x P350) = P6,045

Problem 4 (ReSA)

TS Trading consigned 100 beds costing P600 each to PP Company. The advertised selling price
is P1,000 each bed. The consignment agreement provides that the consignee is to be allowed
a commission of 15% on the selling price. Furthermore, PP Company has to draw sight draft for
60% of the cost of the beds; the advanced is to be recovered periodically by monthly deductions
(in proportion to units sold) from the remittance which accompany the account sales. All
expenses of consignee are to be deducted monthly as incurred.

At the end of the first month, the consignee rendered an account sales showing among others the
following charges: Commission, P2,250; Advertising, P1,500; and Delivery expense, P750.

1. The number of units sold by PP Co. is:


a. 10 c. 20
b. 15 d. 25

2. The amount remitted to TS Co. for the month is:


a. 1,500 c. 5,100
b. 4,500 d. 5,500

3. The consignment profit (loss) of TS Co. is?


a. 1,500 c. 3,412.50
b. 2,137.50 d. None of the above

Solution:
1.

Sales (2,250/15%) 15,000


Divided by: Selling price per unit 1,000

No. of units sold 15 units

2.

Sales 15,000
Less: Charges
Commission 2,250
Advertising 1,500
Delivery expense 750 (4,500)

Due to consignor 10,500


Less: Advances
Value of note (100 beds xP600) x 60% 36,000
Multiplied by: Proportional number of beds sold 15/100 (5,400)

Amount remitted 5,100

3.

Sales 15,000
Less: Charges
Cost of beds (600 per bed x 15 beds) (9,000)
Commission 2,250
Advertising 1,500
Delivery expense 750 (4,500)

Consignment net income 1,500


Problem 5 (ReSA)

On October 1, 2014, the NN Company consigned one hundred wall clocks to P&G
Retailer Inc. Each wall clock had a cost of P150. Freight on the shipment was paid by NN
Company for P200. On December 1, 2014, P&G submitted on account sales stating that it had
sold sixty pieces and it was remitting the P12,480 balance due. The remittance was net of the
following deductions from the sales price of the wall clocks sold:

Commission (20% of sales price) ?


Advertising 500
Delivery and installation charges 100

1. What was the total sales price of the wall clocks sold by P&G Retailers Inc.?
a. 13,440 c. 16,800
b. 15,000 d. 17,000

2. What was the cost of the inventory on consignment?


a. 6,000 c. 6,280
b. 6,080 d. 6,320

Solution:

1.

Sales (unknown) x
Less Charges:
Advertising 500
Delivery and installation
charges 100
Commission (unknown) 20%

Remittance 12,840

x- (P500 + P100 + 20%x) = 12,480 x - 20%x = 12,480 + 600

80%x = 13, 440


x = 16,800
2.

Cost (P150 per unit x 40 units) Freight 6,000


Cost of on shipment inventory on
consignment (P200x40/100) 80

6,080

PROBLEM 6 (PRTC)

Passionate Enterprises consigned 15 dozens of fine men's suits with a cost of P800 a suit to
Fashion Treats Company. Passionate incurred freight cost of P35 per dozen. As required by the
agreement, Fashion Treats reported sales of 8 dozens at P1,200 a suit and reimbursable
expenses of P2,500. Fashion Treats remitted the proceeds to Passionate, net of the agreed 15%
commissions on sale.

1. How much cash was remitted by Fashion Treats to Passionate Enterprises?


A. P139,800 C. P 95,420
B. P142,500 D. P142,800

Solution:

Sales (8x12x1,200) 115,200


Reimbursable exp. 2,500
Commissions (115,200x15%) 17,280 (19,780)

Amount remitted 95,420

2. How much was the consignment profit to Passionate Enterprises?


A. P 55,590 C. P 18,430
B. P 58,590 D. P 18,340

Solution:
Orig Cost 15x12x800 144,000
Cap. Cost 15x35 525

Sales (8x12x1,200) 115,200


COS (144,000x525)8/15 -77,080

Gross Profit 38,120


Reimburses exp 2500
Commisions 17280 -19780
Net Income 18,340

PROBLEM 7 (PRTC)

Jessie Corporation consigned 400 dresses to Anne Fashions at a suggested retail price of P500
each. Jessie paid freight charges of P2,000 on the shipment on consignment. Anne paid delivery
charges of P2,100 for units sold, subject to subsequent settlement. Jessie and Anne agreed that
any sales in excess of the suggested retail price will accrue to the latter. Anne submitted an
account sales on the sale of 215 dresses, 40% of which was sold at P580 each and the rest at
P640 each, All these sales were paid in cash. Jessie's cost is P375 each dress, before any
deferred costs on consignment are taken into account.

1. How much should Anne remit to Bryan for the aforementioned sales to customers?
A. P105,400 C. P107,500
B. P130,340 D. P132,440

Solution:
Sales (215x500) 107,500
Delivery Exp. -2,100
Remittance by Anne 105,400

2. How much is the commission earned by Anne from sales of the consigned goods?
A. P 13, 236 C. P 24,940
B. P 49,800 D. P 82,560

215 Dresses 40% 86 @ 580 49,880


60% 129 @ 640 82,560
Solution:

Cash Proceeds 132,440


Sug. Retail price (107,500)

24,940

PROBLEM 8 (PRTC)
Aircon, Inc, consigned 10 one-horse power air conditioner units to Argy Trading and paid P2,000
freight out. Gross margin is 12.5% of sales. The consignee is allowed a commission of 5% on
sales. Argy Trading submitted an account sales on December 31, 2017 as follows:
Sales P 72,000
Less: Advances to consignor P 10,000
Selling expenses 800
Delivery and
Installation cost 1,200
Commission 3,600 15,600

Net remittance P56,400

1. How much is the net profit or loss of Aircon, Inc. in the consignment?
A. P1,400 profit C. P2,200 profit
B. P8,800 loss D. P720 loss

Solution:

Orig Cost (72,000 x 87.5%) 63,000


Cap. Cost 2,000

Total Cost 65,000

Sales 72,000
COS (65,000)

GP 7,000
OPEX (800 +1200+3600) (5600)

Net Income 1,400

PROBLEM 9 (PRTC)
On August 31, 2015, CTC Company consigned to Lovely Company ten ladies handbags which cost
CTC P300 each. CTC paid freight charge of P150 on the shipment.
On September 30, 2015, Lovely Company submitted an account sales reporting that it sold for
cash seven handbags for which it remitted P3,165 representing the net proceeds after
deductions as follows:
Commission 20% of selling price
Advertising placed upon receipt of shipment P120
Delivery of units sold 75
1. The consignee sold the seven handbags for a total of

Remittance 3,165
Charges: Advertising 120
Delivery 75 195
A. P3,956.25 C. P4,200.00
B. P4,087.50 D. P4,387.50
Solution:

Total proceeds from sales, net of 20% commission 3,360


/80%

Total sales price of the 7 handbags 4,200

2. The inventory of unsold handbags at September 30, 2015 was valued at


A. P900 C. P891 B. P949 D. P1,120
Solution:
Cost (3xP300) 900
Freight (3/10 x P150) 45
Advertising (3/10 x P120) 36

Inventory of unsold handbags 981

PROBLEM 10 (PRTC)

The CCN Interior Designers and Manufacturers Corporation consigned 10 sala set to a furniture
dealer. Manufacturing cost is P4,000 per set. Consignment profits are not recorded separately
by the company. At the end of one month, the dealer reported the sale or 4 sets at P7,000 each
and remitted the net sales proceeds after deducting the following: 20% commission on sets
sold and P1,600 freight paid upon receipt of the 10 sets.

1. The entry on the books of CCN Interiors to record the shipment assuming consignment
profits are calculated separately includes:
A. a debit to Consignment Out of P70,000
B. a debit to Consignment In of P40,000
C. a credit to Merchandise Shipment on Consignment of P40,000 D. a credit to
Merchandise Inventory of P70,000

2. Cash remitted to the consignor was


A. P20,000 C. P21,600
B. P20,800 D. P22,400
Solution:
Sales (4 x P7,000) 28,000
Charges: Commission (20% x P28,000) 5,600
Freight 1,600 7,200

Remittance P20,800

3. The balance of the consignor's inventory relative to consigned goods is


A. P19,200 C. P24,640
B. P24,000 D. P24,960

Solution:
Cost (6 x P4,000) 24,000 Freight (6/10 x P1,600) 960
Balance of Merchandise on Consignment account 24,960

4. Net profit on consignment sales was


A. P4,160 C. P5,120
B. P4,800 D. P5,760
Solution:

Sales (4 x P7,000) 28,000


Cost (4 x P4,000) 16,000
Less: Commission 5,600
Freight (4/10 x P1,600) 640 6,240

Net profit on consignment 5,760

PROBLEM 11 (CRC-ACE)

On November 30, Northup Company consigned 90% freezers to Watson Company for sale at
P1,600 each and paid P1,200 in transportation costs. A report of sales was received on
December 30 from Watson reporting the Sale of 20 freezers, together with a remittance of the
P27,200 balance due. The remittance was net of the agreed 15% commission. How much, and
in what month, should Northup recognize as consignment sales revenue?

November December
a. P0 32,000
b. P0 27,200
c. P144,000 0
d. P142,800 0
ANSWER

A sole takes place when there Is a transfer of ownership of goods. A consignment does not
transfer ownership of the goods to another person who is to sell the goods but the owner retains
title to such goods until the consignee makes a bona fide sole. Since the soles of twenty (20)
freezers were mode in December by the consignee (Watson). therefore, the soles revenue
equivalent to the number of freezers sold (i.e.. 20 freezers x P1,600 = P32.000) by the
consignee should be recognized by the consignor.

PROBLEM 12 (CRC-ACE)

On August . I 2016, JBD. Inc. consigned to Mags store 10 ladies handbags costing P3,000 each,
paying freight charge of P3,000. At the end of the month, Mags Store reported sales of 6
handbags at P6,000 each and expenses incurred of 2,500, and remitted the net proceeds due
to JBD after deducting a 20% commission.
How much net income did JBD realize in August on the consignment?
a. 7,500 net income
b. 6,500 net income
c. 6,700 net loss
d. 6,500 net loss

ANSWER

Consignment Soles: P6,000 x 6……………………………………. P36,000


Less: Applicable costs and expenses related to consignment soles:

Consigner:
Cost of goods sold: P3.000 x 6 ................ P16,000
Freight: P3,030 x 6/10 …………………..……….. P 1,800
Consignee:
Expenses ……………………………………………. P2,500
Commission …………………………………………..... P 7,200 P29,500
Net Income ………………………………………………………………………… P6,500
PROBLEM 13 (CRC-ACE)

On November 1, 2016, the Western Appliance Center ships five (5) of its appliances to the ABC
Store on consignment. Each unit is to be sold 'at-P25,000 payable P5,000 in the month of
purchase and PI,000 per month thereafter. The consignee is to be entitled to, 20% of all
amounts collected on consignment sales. ABC Store sells three (3) appliances in November and
one (1) on December. Regular monthly collections are made by the consignee, and appropriate
cash remittances are made to the consignor at the end of each month. The cost of the appliances
shipped by the consignor was P15,500 per unit. The consignor paid shipping costs to the
consignee totaling P5,000.

The cost of inventory on consignment on December 31, 2016?

a. 15,500
b. 16,500
c. 19,600
d. 24,500
ANSWER

The inventory on consignment amounted to P16,500 computed as follows:


Charges Analysis
Sales Inventory Total
4 sets 1 set 5 units
Charges by consignor
Cost of consigned goods
(@ P15,500/set) P62,000 P15,500 P77,500
Shipping cost 4,000 1,000 5,000

Charges by consignee:
Commissions
(20% of sales (25,030 x 4)] P20,000 0 20,000
Total P86,000 P16,500 P102,500

PROBLEM 14 (CRC-ACE)

On June 1 Bruce Company shipped 25 television sets to Lee Inc on consignment. The sets
are to be sold at an advertised price of P20,000. The cost of each set to the consignor was
10,000. The cost of shipment paid by the consignor was P7,500. The consignor agreed to
absorb the consignee’s expenditure for freight and also to allow consignee P1,000 for
delivery and installation of each set. Commission is to be 25% of the sales price. On June
30, Lee submitted the following summary of consignment sales:
Sets received 25
Sets sold 8
Sets returned to consignor (defective) 2 10
Sets on hand 15

June 3-30 Sales, 8 sets at P20,000 Charges:

Freight – In P 5,000
Deliveries and initiation expenses P 8,000
Commissions 25% of sales P 40,000 53,000
107,000
Remittances enclosed 25,000
Balances owned collections fr. customers not yet made

Compute the inventory value of the units unsold in the hands of the
consignee.
a. 150,000 82,000
b. 153,000
ANSWER
c. 154,500
d. 157,500

Charges Analysis
Sales Inventory Total
8 sets 15 sets 25 sets
Charges by consignor
Cost of consigned goods
(@ P10,000/set) 80,000 150,000 230,000
Freight Out 3,000 4,500 7,500

Charges Analysis
Sales Inventory Total
8 sets 15 sets 25 sets
Charges by consignor
Cost of consigned goods
(@ P10,000/set) 80,000 150,000 230,000
Freight Out 3,000 4,500 7,500

Charges by consignee:
Freight In (200 per set) 2,000 2,000 3,000 5,000
Delivery & Installation 8,000 0 8,000
Commissions
(25% of sales (20,000 x 8)] 40,000 0 40,000
Total 133,000 157,500 290,500

PROBLEM 15 (CRC-ACE)

In 2015, CCA Wholesales transferred goods to a retailer on consignment. The goods cost
P90,000 and normally are sold at a 30% markup. In 2014, merchandise costing P24,000
was sold by the consignee at the normal markup, and the balance of the merchandise was
returned to CCA Wholesalers. The consignee withheld a 10% commission from payment

Prepare journal entries to record the transfer of merchandise to the consignee, the sale of
goods by the consignee and the remittance of the amount due the consignor.
ANSWER

Inventory on Consignment 90,000

Merchandise Inventory 90,000

To record transfer of merchandise to consignee

Consignee Receivable (P24,000 x 130%) 31,2000

Consignment Sales Revenue 31,200

To record consignment sales

Cost of Consignment Goods Sold 24,000

Inventory on Consignment 24,000


To record cost of goods sold

Merchandise Inventory 66,000

66,000
Inventory on Consignment

To record return of consigned goods

Commission Expense (P31.200 x 10%) 3,120

Cash 28,080

Consignee Reed able 31,200


ANSWER
P a g e | 33
P a g e | 34

You might also like